Sei sulla pagina 1di 212

Serial Page

No.
PART - IV No.

1. Heat & Thermodynamics 1


PHYSICS

2. Simple Harmonic Motion 89

3. Wave on String and Sound wave 138

CLASS - XI
HEAT & THERMODYNAMICS
1
 Theory

 Try YourSelf
CONTENTS
 Solved Examples

 EXERCISES

 Single Correct Answer

 More than One Correct Questions

 Passage Questions

 Matrix Match Type Questions.

 Integer Type Questions

 Previous Years Questions

 Answer Key
HEAT & THERMODYNAMICS PHYSICS PART - IV

HEAT :
Heat is energy in transit which is transferred from one body to the other, due to difference in temperature,
without any mechanical work involved.
THERMAL EXPANSION OF SOLID :
Expansion due to increase in temperature is called Thermal Expansion.
At any temperature the atoms of solid are vibrating . Figure represents potential energy for two adjacent
atoms in a solid as a function of their intermolecular separation r. When temperature is increased vibrational
energy of atoms increases. Because the curve is asymmetric, the average separation (r1, r2) increases.

r
T 1> T 2

T2
r0
r1
r2

Hence thermal expansion is direct consequence of deviation from symmetry of the characteristic potential
energy curve of solid.
TYPES OF THERMAL EXPANSION:
Coefficient For temperature

of expansion change T ( T is small)

l  l 
Linear  = lim   change in length      0   0T
T0 l0 T
(where l0 is initial length of the
rod,  is Co-efficient of linear expansion)

1  A 
Superficial   lim   change in Area A  A  A0  A0T
0  T 
T 0 A

(where A0 is initial area of the rod, 


is Co-efficient of superficial expansion)

1  V 
Volume   lim   change in volume
T 0 V0  T 
V  V  V0  V0 T

(where V0 is initial volume of the rod,,


 is Co-efficient of volume expansion)

For isotropic solid, 1  2  3   (say). So   2 and   3

For anisotropic solids, 12  1  2 , 23  2  3 , 13  1  3 where 12 , 23 and 31 are coefficients of
superficial expansion in xy, yz and zx plane and   1  2  3 .

Page # 2
HEAT & THERMODYNAMICS PHYSICS PART - IV

Here 1 , 2 and  3 are coefficients of linear expansion in X, Y and Z directions respectively.


y.

For Variable   :
variation of  with distance Let   ax  b
l

Total expansion =  (expansion of length dx)   (ax  b)dxT dx


0 x
variation of  with temperaturee

T2

Let   f  T  l   l0 f  T  dt
T1

Caution : If  is in ºC then put T1 and T2 in ºC


Similarly if  is in K, put T1 and T2 in K.
VARIATION OF TIME PERIOD OF PENDULUM CLOCKS:

l0
T0  2
g

If temperature is increased by t ,

l0 (1  t)
T  2
g

l0 
T  2 (1  t) (by using Binomial expansion)
g 2

 
T  T0 (1  t)  T  T0  T0 t
2 2
T 1
 (t) T  increase in time period
T0 2
VARIATION IN DENSITY:
With increase in temperature, volume increases, so density decreases and vice-versa.

d0
d
1  T 
For solid, values of  are generally small so we can writee

d  d0(1  T ) (using binomial expansion)

THERMAL STRESS:

A rod of length l0 clamped between two fixed wallss

For T change in temperaturee

F l
stress  (area assumed to be constant) , strain   ;   l  l0T
A l0

Page # 3
HEAT & THERMODYNAMICS PHYSICS PART - IV

F / A Fl0 F
so, Y  
l / l0 Al AT or,,

F 1 
or,  Y | T | 
A 2 T

1
Energy stored in rod E   stress × strain × volume
2

For increase in temperature stress will be compressive and for decrease in temperatue stress will be tensile
Illustration:

A pendulum clock with a pendulum made of Invar (  0.7  106 /Cº ) has a period of 0.5 s and is accuratee
at 25º C. If the clock is used in a country where the temperature averages 35ºC, what correction is necessary
at the end of a month (30 days) to the time given by the clock?
Solution :
In time interval t, the clock will become slow (or will lose time) by

1 t
t 
2 T
1 (7  107 )  (30  86400)  (35  25)
So, t    18.2s .
2 0.5
Illustration:
Two metal rods of the same length and area of cross-section, are fixed end to end between rigid supports.
The materials of the rods have Young modulii Y1 and Y2, and coefficients of linear expansion 1 and  2 . For
or
what condition the junction between the rods does not shift if the rods are cooled.
Solution :

If the junction is not to shift, then strain in the first rod should be 1 T and in the second rod, it should be
 2 T . Since stress is same, we have Y1 1 T  Y2 2T  Y1 1  Y2 2

Illustration:

A copper and a tungsten plate having a thickness   2 mm each are riveted together so that at 0ºC they y
form a flat bimetallic plate. Find the average radius of curvature of this plate at T = 200°C. The coefficients
of linear expansion for copper and tungsten are 1  1.7  105 K 1 and 2  0.4  105 K 1

Solution : L0 (1 +  1T) = (R +  /2)  ….. (1)

L0 (1 +  2T) = (R -  /2)  ….. (2)



From (1) and (2), we get RT (  1 -  2) =  + T (  1 +  2)
2
 1 and  2 are very small

RT 1   2  ~
 L R 

 
R  0.769m .
(1  2 )T

Page # 4
HEAT & THERMODYNAMICS PHYSICS PART - IV

Thermal Expansion of Liquid :


Liquids do not have fixed shape and take the shape of container in which it is kept. Therefore, liquids
undergo volume expansion only.
For liquid two types of co-efficient of volume expansion is defined

Apparent change in volume ( V)a


Co-efficient of apparent expansion (  a ) :  a  Initial volume  rise in temp  V T
0

In apparent expansion change in volume of container is not considered .

Real change in volume ( V)r


Co-efficient of real expansion (  r ) :  r  Initial volume  rise in temp  V T
0

For the determination of real change in volume of liquid expansion of container is also taken into account.
Suppose at temp T1 volume of liquid is V0 (Initial volume)
At temp T2 volume of liquid w.r.t. container = V

T  T2  T1
V0 V

V  V0 [1   a T]  (i)

If Vr is real volume of liquid at temp T2

Vr  V0[1   r T]  (ii)

Vr  V[1   c T]  (iii)

Where  c is co-efficient of volume expansion of container..

(i) & (iii) in (iii) 

V0[1   r T]  V0[1   a T][1   c T]

[1   r T]  1   a T  1   a  a

neglecting  a  c T2

 r  a  c

EXPANSION IN ENCLOSED VOLUME:


Increase in height of liquid level in tube when bulb was initially completely filled :

apparent change in volume of liquid


h
area of tube
V0 (  L   g ) T
h ;
A 0 (1  2 g  T )

Note: (i)  for liquids are in order of 10–3

(ii) For water, density increaes from 0 to 40 C so  is -vee  0 to 4 C  4ºC to higher temperature  is
0

+ve. At 4º C density is maximum

Page # 5
HEAT & THERMODYNAMICS PHYSICS PART - IV

VOLUME COEFFICIENT AND PRESSURE COEFFICIENT OF GASES:


Volume Coefficient :
Consider a given mass of a gas is heated through 1º C at constant pressure. Now the ratio of the increase in
volume to the original volume at 0º C is defined as volume coefficient. If VT and V0 be the volumes of a given
mass of gas Tº C and 0º C respectively, then
VT  V0
V 
V0  T
where v is known as volume coefficient..

Pressure coefficient :
Consider a given mass of a gas is heated through 1º C at constant volume Now the ratio of the increase in
pressure to the original pressure at 0º C is defined as the pressure coefficient. If PT and P0 be the pressures
PT  P0
of a given mass of a gas at Tº C and 0º C respectively, then  p  . where p is known as pressuree
P0  T
coefficient.
Illustration:
A sphere of diameter 7cm and mass 266.5 gm floats in a bath of liquid. As the temperature is raised, the
sphere just begins to sink at a temperature of 35ºC. If the density of the liquid at 0ºC is 1.527gm/cm3m, find
the co-efficient of cubical expansion of the liquid. Neglect the expansion of the sphere.
Solution :
The sphere will sink in the liquid at 35ºC, when its density becomes equal to the density of liquid at 35ºC.
266.5
The density of sphere, s = 35  3 (  density of sphere is constant)
4  22   7 
    
3  7  2
35  1.483gm / cm3

Now, 0  35[1  T]

1.527  1.483[1    35]

1.029  1    35

1.029  1
  0.00083/ C
35
Illustration:
A one litre glass flask contains some mercury. It is found that at different temperature the volume of air
inside the flask remains the same. What is the volume of mercury in this flask if coefficient of linear
expansion of glass is 9  106 / C while volume expansion of mercury is 1.8  104 /C ?

Solution:
If V is the volume of flask, VL of mercury and VA of air in it,
V = V L + VA
Now as with change in temperature volume of air remains constant, the expansion of mercury will be
equal to that of the whole flask i.e.,

V  VL

Page # 6
HEAT & THERMODYNAMICS PHYSICS PART - IV

or VG   VL  L  [as V  V ]

Here V  1 litre = 1000 cc and  G  3G  27  106 / C

So VL  (1000  27  106 /1.8  104 )  150 cc.

CALORIE:
The amount of heat needed to increase the temperature of 1 gm of water from 14.5ºC to 15.5ºC at STP is
known as 1 calorie
Specific Heat:
It is heat required to raise temperature by 1º C or 1 K for unit mass of the body.
dQ = mc dT

T2

Q  m  cdT (be careful about unit of temperature, use units according to the given units of c)
T1

MOLAR HEAT CAPACITY:


If instead of unit mass we consider one mole of a substance, the heat required to change the temperature of
one mole of a substance through 1 ºC (or K) is called molar heat capacity or molar specific heat and is
represented by C. If the molecular weight of a substance is M :

Q  Q m
C  Mc   as c  and n  
n T  m T M

Its SI units are (J/mol K)


THERMAL-CAPACITY :
If instead of unit mass we consider the whole body, (of mass m), the heat required to raise the temperature
of a given body by 1 ºC is called its thermal capacity , i.e.,

Thermal capacity  mc  nC  (Q / T)

Thermal capacity of a body depends on the mass and nature of body. It has units (J/K) or  cal/­ºC and
dimensions [ML2T 2K 1 ] .

WATER-EQUIVALENT :
If thermal capacity of a body is expressed in terms of mass of water it is called water-equivalent of the
body, i.e., water-equivalent of a body is the mass of water which when given same amount of heat as to the
body, changes the temperature of water through same range as that of the body, i.e.,
W = (m × c) gram [where : W = mass of water ; m = mass of substance, c = specific heat]
The unit of water equivalent W is g while its dimension [M].
LATENT HEAT:
The amount of heat required to change the phase of 1 gm of a substance to another phase at
melting or boiling point.
Q = mL L = latent heat of substance in cal/gm or in Kcal/kg.
Lice = 80 cal/gm – latent heat of fusion of ice
Lsteam = 540 cal/gm – latent heat of vaporization of water
Page # 7
HEAT & THERMODYNAMICS PHYSICS PART - IV

PRINCIPLE OF CALORIMETRY :
When two bodies (one being solid and other liquid or both being liquid) at different temperature are
mixed, heat will be transferred from body at higher temperature to a body at lower temperature till both
acquire same temperature. The body at higher temperature releases heat while body at lower temperature
absorbs it, so that :
Heat lost = Heat gained,
i.e. principle of calorimetry represents the law of conservation of heat energy.

DETERMINATION OF SPECIFIC HEAT OF LIQUID USING A CALORIMETER:


The principle of calorimetry states that the total heat given by the hot objects equals the total heat received
by the cold objects. Objects at different temperatures are made to come in contact with each other in the
calorimeter. As a result heat is exchanged between the objects as well as with the calorimeter. Neglecting
any heat exchange with the surroundings,
The method is similar to that for determining the specific heat of the solid. In this case, the calorimeter is
half filled with a liquid of unknown specific heat and a solid of known specific heat, which is insoluble in
the liquid is steam heated and then put inside the calorimeter.
Let the mass of the solid = M
Mass of the calorimeter and stirrer = m1
Mass of the liquid = m2
Specific heat capacity of the solid = s1
Specific heat capacity of the material of calorimeter and stirrer = s2
Specific heat capacity of water = s
Initial temperature of the solid = 1
Initial temperature of the calorimeter, stirrer and water = 2
Final temperature of the mixture = 
From the principle of calorimetry

Ms1  1     m1s2    2   m2s    2 

Ms1  1    m1
Heat gained = Heat lost  m     m s2
2 2 2

Illustration:
The temperature of equal masses of three different liquids A, B and C are 12ºC,
19º C and 28º C respectively. The temperature when A and B are mixed is 16º C and when B and C are mixed
is 23º C. What would be the temperature when A and C are mixed?
Solution:
In accordance with principle of calorimetry :
When A and B are mixed

mc A (16  12)  mc B (19  16)  c A  (3 / 4)cB

and when B and C are mixed

Page # 8
HEAT & THERMODYNAMICS PHYSICS PART - IV

mcB (23  19)  mcC (28  23)  cC  (4 /5)cB

Now when A and C are mixed if T is the common temperature of mixture:

mc A (T  12)  mcC (28  T)

Substituting cA and cC from above,


e,

(3/ 4)(T  12)  (4 /5)(28  T)

which on solving gives,

T  20.25ºC .
Illustration:
A solid material is supplied heat at a constant rate. The temperature of the material is changing with the
heat input as shown in figure. Study the graph carefully and answer the following questions
(i) What do the horizontal regions AB and CD represent?
(ii) If CD = 2 BA, what do you infer?
(iii) What does slope DE represent?

(iv) The slope of OA  the slope of BC . What does this indicate?


e?

Solution :
(i) The horizontal portions AB and CD of the graph represent the change of phase.
The portion AB represents the change of phase from solid to liquid at constant temperature and the
portion CD represents the change of phase from liquid to vapour at constant temperature or the portion
CD represents the latent heat of vaporization
. (ii) CD = 2AB,
i.e., latent heat of vaporization is twice the latent heat of fusion.
(iii) The slope DE is equal to dT/dq for vapour, i.e., this gives the rate of increase of temperature of
vapour with heat input.

1
 Slope of DE 
Specific heat of the vapour

1
or, Specific heat of vapour 
Slope of DE
(iv) Slope OA > slope BC

The slope OA is related with specific heat as

1
Specific heat of solid 
Slopeof OA
Now slope OA > slope BC , represents that specific heat of the liquid is more than that of the solid..

Page # 9
HEAT & THERMODYNAMICS PHYSICS PART - IV

TRY YOURSELF
1. A small quantity; mass m, of water at a temperature (inºC) is poured on to a large mass M of ice which is
at its melting point. f c is the specific heat capacity of water and L the specific latent heat of fusion of ice,
then the mass of ice melted is given by :

ML mc Mc mc


(A) (B) (C) (D)
mc ML L L

2. A metal ball of specific gravity 4.5 and specific heat 0.1 cal/gm-°C is placed on a large slab of ice at 0°C.
Half of the ball sinks in the ice. The initial temperature of the ball is :-
(Latent heat capacity of ice = 80 cal/g, specific gravity of ice = 0.9)
(A) 100 °C (B) 90 °C (C) 80 °C (D) 70 °C
3. Specific heat of a substance can be :-
(A) finite (B) infinite (C) zero (D) negative
4. When two samples at different temperatures are mixed, the temperature of the mixture can be :
(A) lesser than lower or greater than higher temperature
(B) equal to lower or higher temperature
(C) greater than lower but lesser than higher temperature
(D) average of lower and higher temperatures
5. When m g of water at 10°C is mixed with m g of ice at 0°C, which of the following statements are false?
(A) The temperature of the system will be given by the equation m × 80 + m × 1 × (T – 0) = m × 1 ×
(10 – T)
(B) Whole of ice will melt and temperature will be more than 0°C but lesser than 10°C
(C) Whole of ice will melt and temperature will be 0°C
(D) Whole of ice will not melt and temperature will be 0°C
6. Two large holes are cut in a metal sheet. If this is heated, distances AB and BC, (as,shown)
If this is heated, distances AB and BC, (as shown)

(A) both will increase (B) both will decrease


(C) AB increases, BC decreases (D) AB decreases, BC increases
7. Two rods having length 1 and 2, made of materials with the linear expansion coefficient 1 and 2, were
soldered together. The equivalent coefficients of linear expansion for the obtained rod :-

 1 2   2 1  1 1   2 2  1 1   2 2  2 1   1 2
(A) 1   2 (B) 1  2 (C) 1   2 (D) 1  2

8. A steel scale is to be prepared such that the millimeter intervals are to be accurate within 6 × 10 –5
mm.m The maximum temperature variation from the temperature of calibration during the reading of
the millimeter marks is (  = 12 × 10–6 k–1)
(A) 4.0°C (B) 4.5°C (C) 5.0°C (D) 5.5°C

Page # 10
HEAT & THERMODYNAMICS PHYSICS PART - IV

9. The volume thermal expansion coefficient of an ideal gas at constant pressure is

1 1
(A) T (B) T2 (C) (D)
T T2
(Here T = absolute temperature of gas)
10. A liquid with coefficient of volume expansion  is filled in a container of a material having the coefficient of
linear expansion  .f the liquid overflows on heating, then.
(A)  > 3 (B)  < 3 (C)  = 3 (D) none of these
11. Expansion during heating –
(A) occurs only in a solid (B) increases the density of the material
(C) decreases the density of the material (D) occurs at the same rate for all liquids and solids.
12. A metal ball immersed in water weighs w1 at 0ºC and w2 at 50ºC. The coefficient of cubical expansion of
metal is less than that of water. Then
(A) w1 > w2 (B) w1 < w2
(C) w1 = w2 (D) data is insufficient
13. A steel rod 25 cm long has a cross-sectional area of 0.8 cm 2. Force that would be required to stretch
this rod by the same amount as the expansion produced by heating it through 10ºC is:
(Coefficient of linear expansion of steel is 10-5/ºC and Young’s modulus of steel is 2 × 1010 N/m2.)
(A) 160 N (B) 360 N (C) 106 N (D) 260 N
14. Two rods of different materials having coefficients of thermal expansion 1 , 2 and Young’s moduli Y1, Y2
respectively are fixed between two rigid massive walls. The rods are heated such that they undergo the
same increase in temperature. There is no bending of the rods. If 1 : 2 = 2 : 3, the thermal stresses
developed in the two rods are equal provided Y1 : Y2 is equal to
(A) 2 : 3 (B) 1 : 1 (C) 3:2 (D) 4:9
15. If  is the moment of inertia of a solid body having -coefficient of linear expansion then the change
in  corresponding to a small change in temperature T is
1
(A) T (B) T (C) 2 T (D) 3 T
2
16. Two identical beakers are filled with water to the same level at 4°C. If one say A is heated while the
other B is cooled, then :
(A) water level in A will rise (B) water level in B will rise
(C) water level in A will fall (D) water level in B will fall
17. Bimetal strips are used in :
(A) metal thermometers (B) relays for opening or closing electrical circuits
(C) thermostats (D) all of these.
18. f a bimetallic strip is heated , it will .
(A) bend towards the metal with lower thermal expansion coefficient.
(B) bend towards the metal with higher thermal expansion coefficient.
(C) twist itself into helix.
(D) have no bending.

Page # 11
HEAT & THERMODYNAMICS PHYSICS PART - IV

HEAT–TRANSFER:
Conduction:
Heat energy is transferred (usually through solids) from one part of the material medium to other without
transferring the material particles.
(i) Steady State : In this state heat absorption stops and temperature gradient throughout the rod becomes
dT
constant i.e.  constant
dx
(ii) Before steady State : Temparture of rod at any point changes

LAW FOR THERMAL CONDUCTION IN STEADY STATE:

L
T1 A Q T2
x=0 x=L

In steady state heat passing through a bar of length L and cross-section A in time t when its ends are at
(T1  T2 )
temperatures T1 and T2 (  T1 ) is given by:
y: Q  KA t … (i)
L

dQ dT
So rate of flow of heat will be  KA … (ii)
dt dx

The quantity (dT / dx) is called temperature gradient (minus sign indicates that with increase in x ,
temperature  decreases) and the constant K depends on the nature of metal and is called coefficient of
thermal conductivity or simply thermal conductivity and is a measure of the ability of a substance to
conduct heat through it. The dimensions of coefficient of thermal conductivity are [MLT3K 1 ] while its SI
units are W/mK.
dQ T1  T2
Let the two ends of rod of length  is maintained at temperature T1 and T2Thermal current dt  R
Th
l
Where thermal resistance th R 
kA

T1 l T2
(a) Two rods joined
R1 R2
dQ T1  T2
 A B (T1>T2)
dt R1  R2 T1 T T2

(b) Three rods joined to a common point


K2 20ºC
T  100 T  20 T  5
  0
1 2 3 K1 l2

K1A K2A K3A 100ºC l1 K3


T
l3

5ºC

Page # 12
HEAT & THERMODYNAMICS PHYSICS PART - IV

SERIES AND PARALLEL CONNECTION OF RODS IN STEADY STATE:


Series Connection
K1 = thermal conductivity of A K2 = thermal conductivity of B T1 > T2 length l and cross section area A of
 
both rods are same R1  K A R 2  K A
1 2

Thermal current is same in both the rods


 (T  T2 )R 1  (T1  T)R 2
TR 1  T2R 1  T1 R 2  TR 2
 T(R 1  R 2 )  T1 R 2  T2R 1

T1R 2  T2R1
T
R1  R2

T1  T2 T1  T2
 
R R1  R 2
R1, l R2, l
 R  R1  R2 T1 A B T2
K1 T K2
Two rods together is equivalent to a single rod of thermal resistance R1  R2

PARALLEL CONNECTION R1 K1
B
Q2 T1  T2 T1 C T2
 ;  i2  t  R i  i1  i2  ; R2 K2
2
1 1 1
The system of the two rods is equivalent to a single rod of thermal resistance R given by R  R  R
1 2

CONDUCTION BEFORE STEADY STATE:


Differential form : T T - dT

dT dQ dT
 temperature gradient  KA
dx dt dx
dx

CONDUCTION IN A SECTION OF MEDIUM BEFORE STEADY STATE:


dQ1 dQ2 (T1  T2 )
dQ  dQ 1  dQ 2 (In steady state dQ  0 , i.e. dQ1  dQ2 )
T1 T2
dT1 dT
 mS dT  KA dt  KA 2 dt
dx dx

dT KA  dT1 dT2 
 
dt mS  dx dx 
dT = increase in temperature of the section in time dt.

Page # 13
HEAT & THERMODYNAMICS PHYSICS PART - IV

GROWTH OF ICE:
Consider a layer of ice of thickness x. The air temperature is
- º C and water temperature below the ice is 0º C.

Considering unit cross-section area of ice, if a layer of thickness dx grows in time dt .

Then heat given by this layer = mass  latent heat  1  dx    L

  density of ice L  latent heat of fusion of ice.

If this quantity of heat is conducted upwards through the ice layer of area A in time dt .

 0  (  )
Adx    L  K dt
x
x
L 2 L
time taken t   x  dx  (x 22  x21 )
K x1 2K 

 dx K  
Rate of increase of thickness of the ice layer   ,
 dt Lx 

Illustration:
Three cylindrical rods A, B and C of equal lengths and equal diameters are joined in series as shown in
figure. Their thermal conductivities are 2K, K and 0.5K respectively. In steady state, if the free ends of rods
A and C are at 100º C and 0º C respectively, calculate the temperature at the two junction points. Assume
negligible loss through the curved surface. What will be the equivalent thermal conductivity?
L AB BC
100ºC A B C 0ºC
2K K 0.5K

Solution:

As the rods are in series, R eq  R A  R B  R C with R  (L / KA)

L L L 7L
i.e., R eq     … (i)
2KA KA 0.5KA 2KA
dQ  (100  0) 200KA
And hence, H   
dt R (7L / 2KA) 7L

Now in series, rate of flow of heat remains same, i.e., H  HA  HB  HC .

 dQ   dQ  (100  AB )2KA 200KA


So for rod A,      i.e., 
 dt  A  dt  L 7L
Page # 14
HEAT & THERMODYNAMICS PHYSICS PART - IV

or,  A B  100  (100 / 7)  (600 / 7)  85.7º C

 dQ   dQ  (BC  0) 0.5KA 200KA


And for rod C ,      i.e.,  BC  55.55º C
 dt C  dt  L 7L

L  L  L 7L
or, R eq   [from equation (i)]
K eq A 2KA

i.e., K eq  (6 /7)K .

Illustration:
A 2m long wire of resistance 4 ohm and diameter 0.64 mm is coated with plastic insulation of thickness
0.06 mm. When a current of 5 ampere flows through the wire, find the temperature difference across the
insulation in steady state if [ K  0.16  102 cal / cm  Cs]
2
dr
1
r a b

Solution :
Considering a concentric cylindrical shell of radius r and thickness dr as shown in figure. The radial rate of
flow of heat through this shell in steady state will be

dQ d
H  KA
dt dr

Negative sign is used as with increase in r,  decreases

Now as for cylindrical shell A = 2r L


b 
d dr 2LK 2
a r   d
H 1
H  2rLK or
dr
which on integration and simplification gives

dQ 2LK(1  2 )
H 
dt ln b
a  …(i)

I2R (5)2  4 cal


Here, H    24
4.2 4.2 s
L  2m  200 cm

r1  (0.64 / 2) mm= 0.032 cm and R 2  r1  d  0.032  0.006  0.038 cm

( 1  2 ) 
24  ln 38  32 
 2o C
So
2  3.14  200  .16  102

24  2.3026[log10 38  log10 32]



3.14  0.64

Page # 15
HEAT & THERMODYNAMICS PHYSICS PART - IV

55 [1.57  1.50]


or (1  2 )   2 ºC
2
Illustration:
Consider a lake that is getting frozen at an atmospheric temperature of -10 ° C. Assuming that most of the
heat that is lost comes from the latent heat of fusion released when the water freezes. Find the rate at
which the thickness of ice increases as a function of time. Take the conductivity of ice as K and the density
of ice 
~ density of water  

Solution:

The water just beneath the ice is almost at 0°C. Assume that the thickness of ice at time t is x  t  , that the
area of the lake is A0 and that the density of ice is  .
If the latent heat of ice is L, then

dQ LA0dx KA0 10KA0


  0   10  
dt dt x x

dx 10K
or, 
dt xL

10K
 xdx  dt
L 
or,

x 2 10K
or  t  constant
2 L

At t  0 , we assume that x  0 : i.e. initially the lake is not frozen.

20K
Therefore, x2  t or x  t   20K t  C t,
L L

20K
where C is a constant..
L

CONVECTION:
Heat energy is transferred (usually through liquids and gases) by mass movement of molecules from one
point to another. (Due to gravity & buoyant force).
RADIATION:
Heat energy is transferred by electromagnetic waves even in absence of medium.

Page # 16
HEAT & THERMODYNAMICS PHYSICS PART - IV

Absorptive Power a
Absorptive power of a body is defined as the fraction of the incident radiation that is absorbed by the body.
Energy absorbed
Absorptive power a  Energy incident
Emissive Power ‘e’
The emissive power denotes the energy radiated per unit area per unit time per unit solid angle along the
normal to the area.

Emissivity 
Emissivity of a surface is the ratio of the emissive power of the surface to the emissive power of black
body at the same temperature.

(Emissive power of the surface) e


Emissivity,  
(Emissive power of black body at the same temperature) E
Black body
A perfectly black body is one which absorbs completely all the radiation, of whatever wave-length, incident
on it. (a = 1)
KIRCHHOFF’S LAW
It states that the ratio of the emissive power to the absorptive power for radiation of a given wave length
is the same for all bodies at the same temperature, and is equal to the emissive power of a perfectly black
e
body at that temperature. E
a

STEFAN’S LAW OF RADIATION :


The total radiant energy emitted E per unit time by a black body of surface area A is proportional to the
fourth power of its absolute temperature and surface area A of the black body

E  AT4
or, E  AT4 (   Stefan’s constant = 5.67 × 10-8 kg sec-2 k-4)

For a body which is not a black body

E  AT 4 where e is emissivity of the body

Using Krichoff’s law

E( body )
a
E(black body )

AT4
or, a or, a
AT4
Emissivity and absorptive power have the same value. So, good absorbers are good radiators and bad
absorbers are bad radiators.
NET LOSS OF THERMAL ENERGY:

If a body of surface area A is kept at absolute temperature T in a surrounding of temperature T0 (T0  T ) .


Then energy emitted by the body per unit time

Page # 17
HEAT & THERMODYNAMICS PHYSICS PART - IV

E  AT4

And energy absorbed per unit time by the body

E0  AT04

Net, loss of thermal energy per unit time.

E  E  E0  A(T4  T04 ) .

NEWTON’S LAW OF COOLING:


For a small temperature difference between a body and its surrounding, the rate of cooling of the body is
directly proportional to the temperature difference.
If a body of surface area A is kept at absolute temperature T in a surrounding of temperature T0 (T0  T) .
Then net loss of thermal energy per unit time.

dQ
 A(T4  T04 )
dt
If the temperature difference is small

 T  T0  T
4
  T  
4 4
4 4
 A{(T0  T)  T0 }  A T
 0 1    T0 
  T0  
 T T   4AT03 T
 AT04 1  4  higher powers of  1 … (i)
 T0 T0 

dQ dT
Now, rate of loss of heat at temperature T  ms … (ii)
dt dt
From equation (1) and (2), we get

dT 3 dT 4AT03
 ms  4AT0 (T  T0 )  ;   (T  T0 )  
dt dt ms
dT
 k(T  T0 ) . .. (iii)
dt

4AT03
where k 
ms

dT
i.e.,  (T  T0 ) .
dt
dT
From equation (iii), T  T  Kdt
0

T t
dT T  T0  Kt  Kt
By integrating this equation  T  T    Kdt ; T  T  e  T  T0   T1  T0  e
T1 0 0 1 0

Page # 18
HEAT & THERMODYNAMICS PHYSICS PART - IV

WEIN’S BLACK BODY RADIATION :

At every temperature (>0K) a body radiates energy in the form of electromagnetic wave of all wavelengths.

According to Wein’s displacement law if the wavelength corresponding to maximum energy is  m

Then  m T  b where b is a constant (Wein’s constant)


T3
T = temperature of body I
T1 < T2 < T3
T2
Intensity at a specific temp. T Im T5 T1

This is wein’s fifth power law.


m3 m2 m1 

Illustration:
One end of a rod of length 20 cm is inserted in a furnace at 800 K. the sides of the rod are covered with an
insulating material and the other end emits radiation like a blackbody. The temperature of this end is 750 K
in the steady state. The temperature of the surrounding air is 300 K. Assuming radiation to be the only
important mode of energy transfer between the surrounding and the open end of the rod, find the thermal
conductivity of the rod. Stefan constant   6.0  108 W / m2  K 4
Solution :
Quantity of heat flowing through the rod in steady state

dQ K.A.d 750K
 …(i) Furnace
dt x 800K Air temp.
300K
20cm

Quantity of heat radiated from the end of the rod in steady state

dQ
dt

 A T 4  T04  …(ii)

From (i) and (ii)


K.d
x

  T 4  T04 
K  50
 6.0  108 [(7.5)4  (3)4 ] 108
0.2
Or K = 74 W/m – K.
Illustration:
A body initially at 80°C cools to 64°C in 5 minutes and to 52°C in 10 minutes. What will be the temperature
of the body after 15 minutes and what is the temperature of the surroundings ?

Page # 19
HEAT & THERMODYNAMICS PHYSICS PART - IV

Solution :
If T0 is the temperature of the surroundings, then
We have
 80  T0 
log e    5K ….. (1)
 64  T0 
 64  T0 
and log e    5K ….. (2)
 52  T0 

Equating Equations (1) and (2), we get


80  T0 64  T0

64  T0 52  T0
which gives
T0 = 16°C
If T temperature after 15 minutes, then

 52  T0   52  16 
log e    5K or log e    5K ….. (3)
 T  T0   T  16 
From equations (1) and (3), we get

80  16 52  16

64  16 T  16
which gives T  43C

Illustration:
The intensity of solar radiation, just outside the earth’s atmosphere, is measured to be 1.4 kW/m2. If the
radius of the sun 7×108 m, while the earth-sun distance is 150 × 106 km, then find
(i) the intensity of solar radiation at the surface of the sun,
(ii) the temperature at the surface of the sun assuming it to be a black body,
(iii) the most probable wavelength in solar radiation,
Solution:
(i) The radiation emitted from the solar surface per unit time is spread over the surface of a sphere having
a radius equal to earth-sun distance where it is received on the earth (just outside the atmosphere)

 W  4R 2S  I0  4DSe
2

where DSe is the distance between the sun and the earth, and I 0 is the intensity outsidee

the earth’s atmosphere.


2
R 
I0  W   S 
 DSe 
2
3  7  108  49
 1.4  10  W   9 
 W  104 or W  6.4  107 W/m2
 150  10  225

Page # 20
HEAT & THERMODYNAMICS PHYSICS PART - IV

(ii) Assuming the sun to be a blackbody, W  T04


6.4  107  T04  5.67  108 T04 
6.4
 T04   1015 or T0  0.58  104 K  5800K
5.67
(iii) Using Wien’s displacement law,
2.0  103
 mp T0  0.29cm-K  2.9  103 m-K or  mp   5  107 m  5000A
[Note :  mp is also referred to as  max ] 5800

TRY YOURSELF
19. A sphere of ice at 0°C having initial radius R is placed in an environment having ambient temperature > 0°C.
The ice melts uniformly, such that shape remains spherical. After a time ‘t’ the radius of the sphere has
reduced to r. Assuming the rate of heat absorption is proportional to the surface area of the sphere at any
moment, which graph best depicts r (t).

(A) (B) (C) (D)

20. A rod of length L and uniform cross-sectional area has varying thermal conductivity which changes linearly
from 2K at end A to K at the other end B. The ends A and B of the rod are maintained at constant temperature
100°C and 0°C, respectively. At steady state, the graph of temperature : T = T(x) where
x = distance from end A will be

(A) (B) (C) (D)

21. Two sheets of thickness d and 2 d and same area are touching each other on their face. Temperature TA, TB,
TC shown are in geometric progression with common ratio r = 2. Then ratio of thermal conductivity of
thinner and thicker sheet are

(A) 1 (B) 2 (C) 3 (D) 4


22. The wall with a cavity consists of two layers of brick separated by a layer of air. All three layers have the
same thickness and the thermal conductivity of the brick is much greater than that of air. The left layer is at
a higher temperature than the right layer and steady state condition exists. Which of the following graphs
predicts correctly the variation of temperature T with distance d inside the cavity?

(A) (B) (C) (D)

Page # 21
HEAT & THERMODYNAMICS PHYSICS PART - IV

23. A wall has two layer A and B each made of different material, both the layers have the same thickness. The
thermal conductivity of the material A is twice that of B. Under thermal equilibrium the temperature
difference across the wall B is 36°C. The temperature difference across the wall A is
(A) 6°C (B) 12°C (C) 18°C (D) 72°C
24. A ring consisting of two parts ADB and ACB of same conductivity k carries an amount of heat H. The ADB
part is now replaced with another metal keeping the temperatures T1 and T2 constant. The heat carried

ACB
increases to 2H. What should be the conductivity of the new ADB part? Given = 3:
ADB

7 5
(A) k (B) 2k (C) k (D) 3k
3 2
25. Three conducting rods of same material and cross-section are shown in figure. Temperatures of A, D and C
are maintained at 20°C, 90°C and 0°C. The ratio of lengths of BD and BC if there is no heat flow in AB is:

(A) 2/7 (B) 7/2 (C) 9/2 (D) 2/9


26. Three rods made of the same material and having same cross-sectional area but different lengths 10cm, 20
cm and 30 cm are joined as shown. The temperature of the joint is:

(A) 20°C (B) 23.7°C (C) 16.4°C (D) 18.2°C


27. Twelve conducting rods form the riders of a uniform cube of side 'l'. If in steady
state, B and H ends of the rod are at 100°C and 0°C. Find the temperature of the
junction 'A'.

(A) 80°C (B) 60°C (C) 40°C (D) 70°C


28. Six identical conducting rods are joined as shown in figure. Points A and D are maintained at temperature of
200°C and 20°C respectively. The temperature of junction B will be:

(A) 120°C (B) 100°C (C) 140°C (D) 80°C

Page # 22
HEAT & THERMODYNAMICS PHYSICS PART - IV

29. A metallic rod of cross-sectional area 9.0 cm2 and length 0.54 m, with the surface insulated to prevent heat
loss, has one end immersed in boiling water and the other in ice-water mixture. The heat conducted through
the rod melts the ice at the rate of 1 gm for every 33 sec. The thermal conductivity of the rod is
(A) 330 Wm–1 K–1 (B) 60 Wm–1 K–1 (C) 600 Wm–1 K–1 (D) 33 Wm–1 K–1
30. A hollow sphere of inner radius R and outer radius 2R is made of a material of thermal conductivity K. It is
surrounded by another hollow sphere of inner radius 2R and outer radius 3R made of same material of thermal
conductivity K. The inside of smaller sphere is maintained at 0°C and the outside of bigger sphere at 100°C. The
system is in steady state. The temperature of the interface will be :
(A) 50°C (B) 70°C (C) 75°C (D) 45°C
31. The ends of a metal bas of constant cross-sectional area are maintained at temperatures T1 and T2 which are
both higher than the temperature of the surroundings. If the bar is unlagged, which one of the following
sketches best represents the variation of temperature with distance along the bar?

(A) (B) (C) (D)

32. Three identical rods AB, CD and PQ are joined as shown. P and Q are mid points of AB and CD respectively.
Ends A, B, C and D are maintained at 0°C, 100°C, 30°C and 60°C respectively. The direction of heat flow in PQ
is

(A) from P to Q (B) from Q to P


(C) heat does not flow in PQ (D) data not sufficient
33.. The temperature drop through each layer of a two layer furnace wall is shown in figure. Assume that the
external temperature T1 and T3 are maintained constant and T1 > T3. If the thickness of the layers x1 and
x2 are the same, which of the following statements are correct.

(A) k1 > k2
(B) k1 < k2
(C) k1 = k2 but heat flow through material (1) is larger then through (2)
(D) k1 = k2 but heat flow through material (1) is less than that through (2)
34. Two rods A and B of different materials but same cross section are joined as in figure. The free end of A is
maintained at 100°C and the free end of B is maintained at 0°C. If l2 = 2l1, K1 = 2K2 and rods are thermally
insulated from sides to prevent heat losses then the temperature  of the junction of the two rods is

Page # 23
HEAT & THERMODYNAMICS PHYSICS PART - IV

(A) 80°C (B) 60°C (C) 40°C (D) 20°C


35. A composite rod made of three rods of equal length and cross-section as shown in the fig. The thermal
conductivities of the materials of the rods are K/2, 5K and K respectively. The end A and end B are at
constant temperatures. All heat entering the face A goes out of the end B there being no loss of heat from the
sides of the bar. The effective thermal conductivity of the bar is
A B

K/2 5K K

(A) 15K/16 (B) 6K/13 (C) 5K/16 (D) 2K/13.


36. A rod of length L with sides fully insulated is of a material whose thermal conductivity varies with temperature

as K= , where  is a constant. The ends of the rod are kept at temperature T1 and T2. The temperature T at
T
x, where x is the distance from the end whose temperature is T1 is
x
 T L x T2 T2 x
T2  T1
(A) T1  2  (B) ln (C) T1L (D) T1  x
L T1 T1e L
 T1 
37. The power radiated by a black body is P and it radiates maximum energy around the wavelength 0. If the
temperature of the black body is now changed so that it radiates maximum energy around wavelength 3/
40, the power radiated by it will increase by a factor of
(A) 4/3 (B) 16/9 (C) 64/27 (D) 256/81
38. A black metal foil is warmed by radiation from a small sphere at temperature ' T ' and at a distance ' d ' . It is
found that the power received by the foil is P . If both the temperature and distance are doubled, the power
received by the foil will be :
(A) 16 P (B) 4P (C) 2P (D) P
39. Star S1 emits maximum radiation of wavelength 420 nm and the star S2 emits maximum radiation of wavelength
560 nm, what is the ratio of the temperature of S1 and S2 :
(A) 4/3 (B) (4/3)1/4 (C) 3/4 (D) (3/4)1/2
40. Spheres P and Q are uniformly constructed from the same material which is a good conductor of heat and
the radius of Q is thrice the radius of P. The rate of fall of temperature of P is x times that of Q when both are
at the same surface temperature. The value of x is :
(A) 1/4 (B) 1/3 (C) 3 (D) 4
41. An ice cube at temperature – 20°C is kept in a room at temperature 20°C. The variation of temperature of
the body with time is given by

(A) (B) (C) (D)

Page # 24
HEAT & THERMODYNAMICS PHYSICS PART - IV

42. The spectral emissive power E for a body at temperature T1 is plotted against the wavelength and area
under the curve is found to be A. At a different temperature T2 the area is found to be 9A. Then 1/2 =

(A) 3 (B) 1/3 (C) 1 3 (D) 3


43. The intensity of radiation emitted by the Sun has its maximum value at a wavelength of 510 nm and that
emitted by the North Star has the maximum value at 350 nm. If these stars behave like black bodies then the
ratio of the surface temperature of the Sun and the North Star is
(A) 1.46 (B) 0.69 (C) 1.21 (D) 0.83
44. Two bodies P and Q have thermal emissivities of P and Q respectively. Surface areas of these bodies are
same and the total radiant power is also emitted at the same rate. If temperature of P is
P kelvin then temperature of Q i.e. Q is

1/ 4 1/ 4 1/ 4 4
 Q   P   Q  1  Q 
(A)   P (B)   P (C)    (D)    P
 
 P   Q  P  P  P 

45. A black body calorimeter filled with hot water cools from 60°C to 50°C in 4 min and 40°C to 30°C in 8 min.
The approximate temperature of surrounding is :
(A) 10°C (B) 15°C (C) 20°C (D) 25°C
46. The rate of emission of radiation of a black body at 273°C is E, then the rate of emission of radiation of this
body at 0°C will be

E E E
(A) (B) (C) (D) 0
16 4 8
IDEAL GAS
Any gas which obey gas law (PV = nRT, where p is pressure, V is volume, T is temperature) is called ideal gas,
where R = universal gas constant and its value is equal to 8.31 J/mol K. An ideal gas is a simple theoretical
model of a gas. No real gas is truly ideal. Real gas approaches the ideal gas behavior for low pressures and
high temperatures.
KINETIC THEORY OF GASES
Gases are made-up of tiny particles, consisting of molecules, atoms or even ions (sometimes) which retain
the chemical properties of the sample of which they are composed.
The kinetic theory of gases develops a model of the molecular behavior which should result in the observed
behavior of an ideal gas.
Assumptions
(i) Molecules (Whose size is extermely small in comparision to the separation between them) are moving
randomly in all direction.
(ii) Molecules exert no appreciable force on one another or on the walls of the container except during
collision.
(iii) All collisions between the molecules of with the wall of the container are perfectly elastic.

Page # 25
HEAT & THERMODYNAMICS PHYSICS PART - IV

(iv) The duration of a collision is negligible in comparsion to the time spend by a molecue between
collisions.
(v) The molecules obey Newton’s Laws of motion.
The assumptions of kinetic theory are close to the situation at low densities.

PRESSURE OF AN IDEAL GAS


Consider an ideal gas enclosed in a cubical vessel of edge L.
Considering a molecule moving with velocity Y

     Face 1
V  V x  V y  V z ;|V| v2x  Vy2  Vz2
Vy
Face 1
Vz Vx
X
Z

The change in momentum of the molecule on collision with face 1

P  ( mVx )  (mVx )  2mVx

Thus, momentum imparted to the wall = 2mVx

2L
Time between two successive collisions of face 1, t 
Vx
Rate at which momentum is imparted to the wall , F  P/ t  mVx2 / L

Total force on the face 1 due to all the molecules

F   mVx2 / L  (m / L)  mVx2 ...........(1)

As  Vx2   Vy2   Vz2

1 1
 Vx2    Vx2  Vy2  Z2z    V 2 ……………… (2)
3 3

1m 1 mN  v 2
F  V2  ,
3L 3 L N

where N = total number of molecules in the sample.

force 1 mN  V 2
Now, Pressure = 
area 3 L3 N

1  V2 mN
P  , where   3
3 N L

1   V2 
P  V 2 , V 2  is the mean square speed.
3  N 

Page # 26
HEAT & THERMODYNAMICS PHYSICS PART - IV

RMS Speed
1/2
1/2
 V 2dN 
V    
 N V 2  N2V22  ..... 
 1 1
It is defined as rms 
  dN 
   N1  N2  ...... 

Here, N1 molecules have speed V1, N2 molecules have speed V2, and so on.

1m 2
Since P  Vrms
3V

1 2
If we take n mole gas, PV  n.MVrms
3

1 2
 nRT  n.MVrms , where R = universal gas constant = 8.314 J/mol-K
3

3RT
or v rms 
M
Kinetic interpretation of temperature
Kinetic energy of radom motionis the internal energy, given by

1 2 1 3RT
K.E.int  m.Vrms  n.M
2 2 M

K .E.int  T

This means temperature of a body depends on its internal KE. At absolute zero temperature, K.E.int becomes
zero.
DEGREES OF FREEDOM
The degree of freedom of a particle is the number of independent motions which the particle can undergo.
LAW OF EQUIPARTITION OF ENERGY
For a system in equilibrium at absolute temperature T, the average energy per molecule associated with
1
each degree of freedom is kT, where k is Boltzmann’s constant..
2
The internal energy of an ideal gas is entirely the kinetic energy of its molecules.
Thus, internal energy of one mole of an ideal gas, having ‘f’ degrees of freedom

1 1
U  N  f  kT  fRT , N = Avogadro’s Number, R = kN
2 2
dU fR
The specific heat of the gas at constant volume , CV  
dT 2
f f 
 CP  C V  R  R  R    1  R
2 2 

Page # 27
HEAT & THERMODYNAMICS PHYSICS PART - IV

f 
  1  R
C 2
  P  2  1
Thus, ratio of the two specific heats, CV f f
R
2
 A molecule of a monatomic gas has only 3 (translational) degrees of freedom, i.e. f = 3.
f 3
 CV  R  R
2 2
CP  (f /2  1)R  5/2R;   1  2/ f  5/3  1.66

 A molecule of a diatomic gas has 5 degrees of freedom (3-translational and 2-rotational) at ordinary
atmospheric temperatures (Because the vibrational modes are not excited). The moment of inertia about
the line joining the two atoms is negligibly small. Hence, the roational energy about that axis is zero.
F=5
f 5
 CV  R  R
2 2
CP  (f /2  1)R  7/2R
  1  2/ f  7/5  1.40
Note : At very hight temperatures, if vibrational mode is also considered, then f = 7
A molecule of a triatomic or polyatomic gas has 6 degrees of freedom (3 translational and 3 rotational )
f=6

f
Cv  R  3R
2
f 
Cp    1  R  4R
2 
2 4
  1    1.33
f 3
However, if the atoms of the molecule are arranged in one line (like the molecule of CO2), then the degrees
of freedom are only 5 and its Cv, Cp and  values will be similar to those of the diatomic gases.

Illustration:
An air bubbles starts rising from the bottom of a lake. Its diameter is 3.6 mm at the bottom and 4 mm at the
surface. The depth of the lake is 250 cm and the temperature at the surface is 40°C. What is the temperature
at the bottom of the lake ? Given atmospheric pressure = 76 cm of Hg and g = 980 cm/sec2. (Specific gravity
of mercury = 13.6)
Solution: At the bottom of the lake, volume of the bubble

4 4
V1  r13  (0.18)3 cm3
3 3
Pressure on the bubble P1 = Atmospheric pressure + Pressure due to a column of 250 cm of water.
= 76 × 13.6 × 980 + 250 × 1 × 980 = (76 × 13.6 + 250) 980 dyne / cm2
T1 = Temperature at the bottom
At the surface of the lake, Volume of the bubble

4 4
V2  r23  (0.2)3 cm3
3 3

Page # 28
HEAT & THERMODYNAMICS PHYSICS PART - IV

Pressure on the bubble P2 = atmospheric pressure


= 76 × 13.6 × 980 dyne/cm2
T2 = 273 + 40°C = 313 K

P1 V1 P2V2
Now, T  T
1 2

(76  13.6  250)980  (4 /3)(0.18)3 (76  13.6) 980(4 /3)(0.2)3


or 
T1 313

1283.6  (0.18)3 1033.6(0.2)3


or 
T1 313

1283.6  (0.18)3  313


T1   287.37K
1033.6(0.2)3
 T1 = 283.37 – 273 = 10.37°C

Illustration:

Given : Avogardo’s Number N = 6.02 × 1023 and Boltzmann’s constant

k = .38 × 10–23 J/K.

(i) Calculate the average kinetic energy of translation of the molecules of an ideal gas at
0°C and at 100°C.

(ii) Also calculate the crossesponding energies per mole of the gas.

Solution:

(i) According to the kinetic theory, the average kinetic energy of translation per molecule of an
ideal gas at Kelvin temperature T is (3/2) kT, where k is Boltzmann’s constant.

At 0°C (T = 273 K)

The kinetic energy of translation = 3/2 kT

= (3/2) × (1.38 × 10–23) × 273 = 5.65 × 10–23 J/molecule

At 100°C (T = 373 K),

(ii) 1 mole of gas contains N (=6.02 × 1023 ) molecules.

Therefore, at 0°C, the kinetic energy of translation of 1 mole of the gas

= (5.65 × 10–21) (6.02 × 1023)  3401 J/molee

And at 100°C, the kinetic energy of translation of 1 mole of gas

= (7.72 × 10–21) (6.02×1023)  4647 J/mole.

Illustration:

How many degrees of freedom does the gas molecule have if under standard conditions the gas density is 1.3
kg/m3 and the velocity of sound in it is v = 330 m/s.

Page # 29
HEAT & THERMODYNAMICS PHYSICS PART - IV

Solution:

P v 2
As v 
 P
2 2
If f be the number of degree of freedom, then f    1  v 2 / P   1
 
P = 1.013 × 105 N/m2

 = 1.3 kg/m3 , v = 330 m/s.  f = 5


INTERNAL ENERGY OF AN IDEAL GAS

In case of an ideal gas, no internal force of interaction exists. Hence, the internal potential energy is zero.
The interrnal energy of an ideal gas comprises of molecular kinetic energy.

f
U  nRT
2

WORK DONE
Work done by the gas (or system) over the surroundings can be calculated as

P
p B
dW = P dV A

vf
 W   P dV dV V
vi

Here, dW is elemental work done by pressure P, of the system during elemental change in voluem dV.

Work done in the process AB is equal to the area under the curve AB and V-axis.

Mechanical Equivalent of Heat:

Whenever mechanical work is transformed into heat or heat into mechanical work, there is a constant
ratio between the work and the amount of heat. This ratio is called “mechanical equivalent of heat” and is
denoted by J. Thus, if W be the amount of work done and Q the amount of heat prouduced, we havee

W
J, W  JQ
Q

If Q = 1 unit then J = W. Therefore, J is numerically equal to the mechanical work required to produce one
unit of heat.

FIRST LAW OF THERMODYNAMICS :

It is the consequence of conservation of energy for gaseous system.

Heat supplied to the gas = Increase in internal energy + work done by the gas.

Q  U  W Q = +ve  heat is supplied to the gas

in differential form dQ  dU  dW Q = –ve  heat is taken out from the gas


as

Page # 30
HEAT & THERMODYNAMICS PHYSICS PART - IV

and dQ  nCdT C = molar specific heat

C  CP (constant pressure); C  CV =(constant volume)

INDICATOR DIAGRAM:
This is graph between pressure and volume of a system under going operation,
P

(1) Every point of Indicator Diagram represents a unique state (P, V, T) of gases.
(2) Every curve on Indicator Diagram represents a unique process.
Isochoric Process (V = constant) dV  0  dW  0
By First Law of Thermodynamic dQ  dU  nCV dT
Isobaric Process (P = constant) dP  0
f
By First Law of Thermodynamics dQ  dU  dW nCp (T2  T1 )    nR(T2  T1 )  nR(T2  T1 )
2
* Be careful if V  0 then not necessarily an Isochoric Process.

* If P  0 then not necessarily an Isobaric Process.


Isothermal Process (T = constant):

dU  0 ( dT  0) PV  K

By First Law of Thermodynamics  dQ  dW   dQ  PdV

V2 P
W  nRTln  nRT ln 1 .
V1 P2
Adiabatic Process :

dQ = 0 but if Q  0 it is not necessarily adiabatic.

dW  dU by First Law of Thermodynamics


T2
nRdT nR(T1  T2 ) P1 V1  P2 V2
W   
T1   1  1  1

How to get the process Equation for adiabatic


(i) First Law of Thermodynamics with process condition
nRdT
dU  dW  …(i)
 1
(ii) Differential form of gas law d(PV)  d(nRT)

PdV  VdP  nRdT


nRdT
But dW  PdV 
 1

Page # 31
HEAT & THERMODYNAMICS PHYSICS PART - IV

So PdV  VdP  (   1)PdV …(ii)


VdP  ( PdV)
dP dV
 
P V
lnP   ln V  lnC

PV   Const.
TV 1  Const.
T  P1  Const.

dP dP
For Adiabatic Process PV   constant 
dV adiabatic dV isothermal

Slope of adiabatic curve is more in magnitude in comparison to the slope of the isothermal curve.

P P
Bulk Modulus of Gases:     V
(V)/ V V

dP  P 
Isothermal bulk modulus of Elasticity ET   dV / V  V  V 
 Isothermal

dP  P  adia E
Adiabatic bulk modulus of Elasticity Eadia   dV / V  V  V  ; E   .
 adia T

CYCLIC PROCESS :
It is the combination of various processes such that initial and final state is same. Therefore initial and final
temperature is equal i.e., T  0  U  0  Q  W

Efficiency of a Cyclic Process

U  0  no rise in internal energy Q  W

Qin
P 1 2
Qout

work done by gas


Efficiency  
heat input

Molar Specific Heat For Polytropic Process PV n  K

R R R R
molar heat capacity of polytropic process C    
 1 1n  1 n 1

So C is constant for polytropic process


Work done on gas in some process :

Page # 32
HEAT & THERMODYNAMICS PHYSICS PART - IV

V  0but dV  0 W can be zero For clockwise W   vee

work done = + ve but dW  0 For anticlockwise W   vee


Work done is least for monoatomic gas in expansion :

Isothermal
Polyatomic
Diatomic Adiabatic
Monatomic Process
V1 V2 V

Illustration:

Three moles of an ideal gas being initially at a temperature T0  273 K were isothermally expanded   5.0
time its initial volume and then isochorically heated so that the pressure in the final state became equal to
that in the initial state. The total amount of heat transferred to the gas during the process equals Q  80 KJ.
Find the ratio   CP /CV for this gas.

Solution : In isothermal process, the heat transferred to the gas is given by

Q1  nRT0ln(V2 / V1 )  nRT0ln …(i)

[   (V2 / V1 )  (P1 / P2 )]

In isochroric process, Q2  U (W = 0)

 Q2  nCV T  n{R /(   1)}T …(ii)

P2 T0 P 
Now P  T or T  T0  1   T0
1  P2 

 T  T0  T0  (  1)T0 …(iii)

 R 
substituting the value of T from equation (iii) in equation (ii), we get Q 2  n   (   1)T0
  1 

 R 
 Q  nRT0ln  n   (   1)T0
  1 
Q  1 
or  ln    
nRT0   1 

Page # 33
HEAT & THERMODYNAMICS PHYSICS PART - IV

 1
 1 
or Q
 ln
nRT0

 1
  1
Q
 ln
nRT0

(5  1)
Substituting given values, we get   1  Solving we get   1.4
80  103
 ln5
3  8.3  273

Illustration:
If 2 moles of an ideal monoatomic gas at temperature T0 is mixed with 4 moles of another ideal monoatomic
gas at temperature 2T0, then find the temperature of the mixture?
Solution :
Let T be the temperature of the mixture. Then U =U1 + U2

f f f
or (n1  n2 )RT  (n1 )RT0  (n 2 )(R)(2T0 )
2 2 2

5
or (2 + 4) T = 2T0 + 8T0 (n1 = 2, n2 = 4) or T  T0
3

Illustration:
In a given gas during a process one third of heat supplied is used to raise internal energy of gas. Find molar
specific heat of the gas and their process.
Solution: Heat supplied = nCdT C = molar specific heat
nCdT f 3 fR 3R  dQ 
 nRdT  C     dU  .
3 2 2  1  3 

Illustration:
An ideal gas is taken through a process in which the pressure and the volume are changed according to the
equation P = KV. Show that the molar heat capacity of the gas for the process is given by C = CV +R/2.
Solution : PV = nRT . . . . . (i)
P = KV . . . . . (ii)
KV2 = nRT From (i) and (ii),
Differentiating 2 KVdV = nRdT
PdV = nRdT/2 dQ = dU + PdV
nCdT = nCVdT + nRdT/2
C = CV + R/2

Page # 34
HEAT & THERMODYNAMICS PHYSICS PART - IV

Illustration:
One mole of monoatomic ideal gas is taken through the cycle shown in figure. A  BAdiabatic expansion
B  C Cooling at constant volume C  D Adiabatic compression D  A Heating at constant volume. The
pressure and temperature at A, B etc., are denoted by PA ,TA ;PB , TB etc/ respectively. Given
TA  1000K, PB  (2 /3)PA and PC  (1/3)PA . Calculate (a) The work done by the gas in the process A  B
(b) The heat lost by the gas in the process B  C and (c) Temperature TD given (2/3)2/5  0.85 and R  8.31
J/mol K.

A
B

P
D C

V
Solution :

 1
T  TB   PB  5
(a) As for adiabatic change 1  constant so      wheree  
P  TA   PA  3

1
1 2/5
2   2
i.e. TB  TA    1000    850K
3 3

R[Ti  Tf ] 1  8.31[1000  850]


so WAB  
[   1] [(5 /3)  1]

i.e. WAB  (3/ 2)  8.31  150  1869.75 J

(b) For B  C, V  constant so W  0

so from first law of thermodynamics Q  U  W  CV T  0

3 
or Q  1   R (TC  850)
2 
3
as Cv  R
2
Now along path BC, V = constant; P  T

PC TC (1/3)PA T 850
i.e.  , TC   TB  B   425 K…(ii)
PB TB (2/3)PA 2 2

3
So Q  1   8.31(425  850)  5297.625 J [Negative heat means, heat is lost by the system]
2

PD TD TD
(c) D  A process is isochoric P  T , i.e. PD  PA
TA
A A

  1 1
 TD   PD  P T 
But C and D are on the same adiabatic       A D 
 TC   PC   PC TA 

Page # 35
HEAT & THERMODYNAMICS PHYSICS PART - IV

1
1
1/   P  
or  TD   TC  A  , i.e.
 PC TA 

3/5
 1  2 2/3   3  2/5
i.e. TD      1000   i.e. TD  500 K
 2  3    1000 

Illustration:
An ideal gas is taken through a cyclic thermodynamical process through four steps. The amount of heat
involved in these steps are Q1  5960 J ; Q2  5585 J Q3  2980 J ; and Q 4  3645 J respectively. The
corresponding works involved are W1  2200J; W2  825 J; W3  1100 J and W4 respectively.
y.

(a) Find the value of W4

(b) What is the efficiency of the cycle?


Solution :
(a) According to the given problem

Q  Q1  Q2  Q3  Q 4  5960  5585  2980  3645

Q  9605  8565  1040 J

W  W1  W2  W3  W4  2200  825  1100  W4  275  W4

and as for cyclic process UF  U I , U  UF  U I  0

So from first law of thermodynamics, i.e., Q  U  W, we havee

1040  (275  W4 )  0, i.e. W4  765 J

(b) Efficiency of a cycle is defined as

Network W Q
  
Input heat (Q1  Q 4 ) (Q1  Q 4 )

1040
  0.1082  10.82%
9605

TRY YOURSELF
47. When an ideal gas is compressed isothermally then its pressure increases because :
(A) its potential energy increases
(B) its kinetic energy increases and molecules move apart
(C) its number of collisions per unit area with walls of container increases
(D) molecular energy increases
48. The r.m.s. speed of hydrogen molecules is v when at a temperature of 300 K. What is its
value at a temperature of 450 K ?

Page # 36
HEAT & THERMODYNAMICS PHYSICS PART - IV

v v
(A) (B) (C) v 1.5 (D) 1.5 v
1.5 1 .5

49. If vrms = root mean square speed of molecules,


vav = average speed of molecules.
vmp = most probable speed of molecules,
vs = speed of sound in a gas
Then, identify the correct relation between these speeds.
(A) vrms > vav > vmp > vs (B) vav > vmp > vrms > vs
(C) vmp > vav > vrms > vs (D) vrms > vav > vs > vmp
50. Three closed vessels A, B, and C are at the same temperature T and contain gases
which obey the Maxwellian distribution of velocities. Vessel A contains only O 2, B
only N2 and C a mixture of equal quantities of O 2 and N2. If the average speed of O 2
molecules in vessel A is V1, that of the N2 molecules in vessel B is V2 ,the average
speed of the O 2 molecules in vessel C will be :

(A) (V1 + V2 )/2 (B) V 1 (C) (V 1 V 2 ) 1/2 (D) 3kT / M

51. A vessel contains a mixture of one mole of oxygen and two moles of nitrogen at 300 K.
The ratio of the average rotational kinetic energy per O2 molecule to that per N2 mol-
ecule is :
(A) 1:1 (B) 1 : 2 (C) 2:1
(D) depends on the moments of inertia of the two molecules
52. In the following figures (1) to (4), variation of volume by change of pressure is shown. A
gas is taken along the path ABCDA. The change in internal energy of the gas will be:

(1) (2) (3) (4)

(A) positive in all cases from (1) to (4)


(B) positive in cases (1), (2) and (3) but zero in case (4)
(C) negative in cases (1), (2) and (3) but zero in case (4)
(D) zero in all the four cases.
53. The pressure p and volume V of an ideal gas both increase in a process.
(A) Such a process is not possible
(B) The work done by the system is positive
(C) The temperature of the system must increase
(D) heat supplied to the gas is equal to the change in internal energy.

Page # 37
HEAT & THERMODYNAMICS PHYSICS PART - IV

54. Pressure versus temperature graph of an ideal gas is as shown in figure.

Corresponding density () versus volume (V) graph will be :

(A) (B) (C) (D)

55. In the cyclic process shown on the P – V diagram the magnitude of the work done is :
2 2
 P  P1   V  V1 
(A)  2  (B)  2 
 2   2 


(C) (P – P1) (V2 – V1) (D)  (P2V2 – P1V1)
4 2
56. A fixed mass of an ideal gas undergoes the chage represented by XYZX below
(Fig.)Which one of the following sets could describe these of changes ?

XY YX ZX
(A) isothermal adiabatic comperssion at
expansion comperssion constant pressure
(B) adiabatic isothermal pressure reduction
expansion comperssion constant volume
(C) isothermal adiabatic comperssion at
comperssion expansion constant pressure
(D) adiabatic isothermal comperssion at
comperssion expansion constant pressure
57. Consider two processes on a system as shown in fig. The volumes in the initial states
are the same in the two porcesses and the volumes in the final states are also the
same. Let W1 and W2 be the work done by the system in the processes A and B
respectively.

Page # 38
HEAT & THERMODYNAMICS PHYSICS PART - IV

(A) W1 > W2

(B) W1 = W2

(C) W1 < W2

(D) Nothing can be said about the relation between W1 and W2
58. The first law of thermodynamics may be witten
U = Q + W
where U is the increase in internal energy of the system Q is the heat transfer to
the system and W is the external work done on the system.
Which of the following is correct for the case of isothermal expansion of an ideal
gas ?
(A) W < 0 (B) W = 0 (C) Q = 0 (D) U > 0
59. A certain amount of an ideal gas is taken form state A to state B, one time by process I
and another time by process II. If the amount of heat absorbed by the gas are Q1 and Q2
respectively, then :
(A) Q1 = Q 2 (B) Q1 < Q2

(C) Q1 > Q 2 (D) data insufficient

60. Figure shows a cylinder containing gas and closed by a movable piston. The cylinder
is submerged in an ice-water mixture. The piston is quickly pushed down from
position (1) to position (2). The piston is held at position (2) until the gas is again at
0°C and then is slowly raised back to position (1).
P-V diagram for the above process will be

(A) (B) (C) (D)

61. When an ideal diatomic gas is heated at constant pressure , the fraction of the
heat energy supplied which increases the internal energy of the gas is .

2 3 3 5
(A) (B) (C) (D)
5 5 7 7

62. For a solid with a small expansion coefficient,


(A) Cp - Cv = R (B) Cp - Cv = R
(C) Cp is slightly greater than Cv (D) Cp is slightly less than Cv

Page # 39
HEAT & THERMODYNAMICS PHYSICS PART - IV

63. In the following P–V diagram of an ideal figure, two adiabates cut two isothermals
at T1 and T2 . The value of VB/VC is
(A) = VA /VD (B) < VA / VD

(C) > VA / VD (D) cannot say

64. A gas kept in a container of finite conductivity is suddenly compressed. The process
(A) must be very nearly adiabatic (B) must be very nearly isothermal
(C) may be very nearly adiabatic (D) may be very nearly isothermal
65. Three identical adiabatic containers A, B and C contain helium, neon and oxygen
respectively at equal pressure. The gases are pushed to half their original volumes.
(A) The final temperature in the three containers will be the same.
(B) The final pressures in the three containers will be the same.
(C) The pressure of helium and neon will be the same but that of oxygen will be
different.
(D) The temperature of helium and neon will be the same but that of oxygen will be
different
66. Four curves A, B, C and D are drawn in the Fig. for a given amount of gas. The
curves which represent adiabtic and isothermal changes are
(A) C and D respectively

(B) D and C respectively


(C) A and B respectively
(D) B and A respectively
67. The stopes of isothermal and addiabatic curves are related as
(A) Isothermal curve slope = adiabatic curve slope
(B) Isothermal curve slope =  × adiabatic cureve slope
(C) Adiabatic curve slope =  × isothermal curve slope

1
(D) Adiabatic curve slope = × isothermal curve slope.
2

68. The adiabatic elasticity of hydrogen gas ( = 1.4) at NTP is :


(A) 1 × 105 N/m2 (B) 1 × 10–5 N/m2 (C) 1.4 N/m2 (D) 1.4 × 105 N/m2
69. A given quantity of a gas is at pressure P and absolute temperature T. The isothermal
bulk modulus of the gas is:

2 3
(A) P (B) P (C) P (D) 2P
3 2

Page # 40
HEAT & THERMODYNAMICS PHYSICS PART - IV

 dv 
70. Which of the following graphs correctly represents the variation   dP  with P for an
 
v
ideal gas at constant temperature :

(A) (B) (C) (D)

71. A and B are two adiabatic curves for two different gases. Then A and B corresponds to :
(A) Ar and He respectively (B) He and H2 respectively

(C) O2 and H2 respectively (D) H2 and He respectively


72. A thermally insulated chamber of volume 2V 0 is divided by a frictionless piston of
area S into two equal parts A and B.Part A has an ideal gas at pressure P 0 and
temperature T0 and in part B is vacuum. A massless spring of force constant K is
connected with the piston and the wall of the container as shown. Initially the
spring is unstretched.The gas in chamber A is allowed to expand. Let in
equillibrium the spring is compressed by x 0 . Then :

Kx 0
(A) final pressure of the gas is
S

1 2
(B) work done by the gas is Kx0
2

1 2
(C) change in internal energy of the gas is Kx 0
2

(D) temperature of the gas is decreased


73. For an ideal gas :
(A) the change in internal energy in a constant pressure process from temperature
T1 to T2 is equal to nC v (T2 – T1 ), where C v is the molar specific heat at constant
volume and n the number of moles of the gas.
(B) the change in internal energy of the gas and the work done by the gas are
equal in magnitude in an adiabatic process.
(C) the internal energy does not change in an isothermal process.
(D) no heat is added or removed in an adiabatic process.
74. The relation between internal energy U, pressure P and volume V of gas in a adiabatic
process is :
U = a + b PV
where a and b are constants. What is the value of the ratio of the specific heats ?
a b 1 a 1 b
(A) (B) (C) (D)
b b a a

Page # 41
HEAT & THERMODYNAMICS PHYSICS PART - IV

75. A monoatomic ideal gas, initially at temperature T1, is enclosed in a cylinder fitted with
a frictionless piston. The gas is allowed to expand adiabatically to a temperature T2 by
releasing the piston suddenly. If L1 and L2 are lengths of the gas column before and
T1
after expansion respectively, then T is given by :
2

2/3 2/3
 L1   L1  L2  L2 
(A)   (B)  L  (C) L1 (D)  
 L2   2  L1 
76. Two different ideal diatomic gases A and B are initially in the same state. A and B are
then expanded to same final volume through adiabatic and isothermal process respec-
tively. If PA , PB and TA , TB represents the final pressure and temperatures of A and B
respectively then:
(A) PA < PB and TA < TB (B) PA > PB and TA > TB
(C) PA > PB and TA < TB (D) PA < PB and TA > TB
77. An ideal gas undergoes a process in which T = T0 +aV3 , where T0 and ‘a’ are positive
constants and V is molar volume. The volume for which pressure will be minimum
is

1/3 1/3 2/3 2/3


 T0   T0   a   a 
(A)  2a  (B)   (C)   (D)  
   3a   2T0   3T0 

78. Three moles of an ideal monoatomic gas perform a cycle shown in figure. The gas
te mp e r a tur e s in d i ff e r e nt s ta te s are T1 = 40 0 K,
T2 = 800 K, T3 = 2400 K, and T4 = 1200 K. The work done by the gas during the cycle
is :-
(A) 5 kJ (B) 10 kJ

(C) 15 kJ (D) 20 kJ

Page # 42
HEAT & THERMODYNAMICS PHYSICS PART - IV

SOLVED EXAMPLES
1. If hot air rises, why is it cooler at the top of a mountain than near sea level?
Sol. Pressure decreases with height. Therefore if hot air rises, it suffers adiabatic expansion.
Therefore form first law of thermodynamics
Q = U + W
U = – W (as Q = 0)
This causes a decrease in internal energy and hence a fall of temperature. That is why it is cooler at the top
of a mountain than at sea level.
2. A diatomic ideal gas undergoes a thermodynamic change according to the
P-V diagram shown in the figure. The total heat given of the gas is nearly
P –
(A) 2.5 P0V0 (B) 1.4 P0V0
B
(C) 3.9 P0V0 (D) 1.1 P0V0 2P0 Isothermal
Sol. (C).Process AB : Q AB   U AB  WAB
P0 C
WAB  0 A

f f 5
 U AB  nRT  ( PV)  UAB  ( PV) V
2 2 2 V 0 2V 0
Q AB  2.5P0 V0

Process BC : Q BC  UBC  WBC

Q BC  0  2P0V0 log2  1.4 P0V0

Qnet  Q AB  Q BC  3.9 P0 V0
3. A diatomic ideal gas is heated at constant volume until the pressure is doubled and again heated at con-
stant pressure until volume is doubled. Find The average molar heat capacity for whole process.
Sol. Let initial pressure , volume, temperature be P0, V0, T0 indicated by state A in P-V diagram. The gas is then
isochorically taken to state B (2P0, V0, 2T0) and then taken from state B to state C (2P0, 2V0, 4T0)
isobarically.
Total heat absorbed by 1 mole of gas 1

5 7 19
Q  Cv (2T0  T0 )  Cp (4T0  2T0 ) = RT0  R  2T0  T0
2 2 2
Total change in temperature from state A to C is T = 3T0.

19
RT
Q 2 0 19
Molar heat capacity =   R
T 3T0 6

4. A cylinder with a movable piston contains 3 moles of hydrogen at standard temperature and pressure. The
walls of the cylinder are made of a heat insulator, and the piston is insulated by having a pile of sand on it.
By what factor does the pressure of the gas increase if the gas is compressed to half its original volume ?
Sol. The process in adiabatic

 P1 V1  P2 V2

V1 2 7 P2
Given, V  1 and   (for diatomic gas)   (2)7/5 i.e., P2 = (2)7/5P1
2 5 P1

Page # 43
HEAT & THERMODYNAMICS PHYSICS PART - IV

5. Find the amount of work done to increase the temperature of one mole of an ideal gas by 30°C if it is
expanding under the condition V  T2/3 .
Sol. PV = RT for 1 mole

RT
W   PdV   dV
V

dV 2 1/3 dV 2 dT
V  CT2/3   CT or 
dT 3 V 3 T

T2
 2  dT 2
 W   RT    R (T2  T1 )  166.2 J
T 3 T 3
1

6. An ideal gas undergoes an expansion from a state with temperature T1 and volume V1 to V2 through three
different polytropic processes A, B and C as shown in the P-V diagram. If | E A | , | E B | and | E C | be the
magnitude of changes in internal energy along the three paths respectively, then –
P
(A) | E A |  | EB |  | E C | if temperature in every process decreases
(B) | E A |  | EB |  | E C | if temperature in every process decreases
(C) | E A |  | EB |  | E C | if temperature in every process increases A
B
(D) | E B |  | E A |  | E C | if temperature in every process increases C
Sol. (AC). V
V1 V2
Initial state is same for all three processes (say initial internal energy = E0)
In the final state, VA = VB = VC.
and PA  PB  PC  PA VA  PB VB  PC VC  E A  E B  EC
if T1 > T2 then E0 > Ef for all three processes
and hence (E0  E A )  (E0  EB )  (E0  EC )  |  E A |  | EB |  | EC |
If T1 < T2 then E0 < Ef for all three processes
and (E A  E0 )  (EB  E0 )  (EC  E0 )  |  E A |  |  EB |  |  EC |
7. Two moles of helium gas ( = 5/3) are initially at tem-
P
perature 27°C and occupy a volume of 20 litres. The gas PV
, P1,V 1

is first expanded at constant pressure until the volume is a b


doubled. Then, it undergoes an adiabatic change until the
temperature returns to the initial value.
(i) Sketch the process on a P-V diagram. P2,V 2
(ii) What are the final volume and pressure of the gas?
(iii) What is the work done by the gas? –3 –3
V
20 × 10 40 × 10
Sol. For a perfect gas, PV = nRT

nRT 2  8.31  (273  27)


P1   (i) Fig. shows the sketch of the process
v 20  103
or P1 = 2.5 × 105 N/m²

V V1 V 
(ii) At constant pressure, T  T  T1   1  T  2  300  600 K
1 V

Page # 44
HEAT & THERMODYNAMICS PHYSICS PART - IV

(iii) The gas now undergoes an adiabatic change.


T1 = 600 K, T2 = 300 K, V1 = 40 lit., V2 = ?,  – 1 = 5/3 – 1 = 2/3
T1 V1  – 1 = T2 V2  – 1

600 (40)2/3 = 300 (v2)2/3


(2)3/2 × 40 = V2 or V2 = 112.4 lit.

But, P1V1 = P2V2 

5/3
 40 
P2     2.5  105 = 0.45 × 105 N/m²
 112.4 

(iv) Work done under isobaric process,


W1 = P(V1 – V) = 2.5 × 105 (40 × 10–3 – 20 × 10–3) = 2.5 × 105 × 20 × 10–3 = 50 × 102 J
Work done under adiabatic process,

R 2  8.31
W2  (T2  T1 )  (600  300) = 7442 J
 1 2 /3
W = W1 + W2 = 5000 + 7442 = 12442 J

8. A steam engine delivers 5.4×108J of work per minute and services 3.6 × 109J of heat per minute from its
boiler. What is the efficiency of the engine? How much heat is wasted per minute?
Sol. Output i.e. useful work done per min = 5.4 × 108 J
Input i.e. heat absorbed per min = 3.6 × 109 J

output 5.4  108


Efficiency =   0.15 = 0.15 × 100% = 15%
input 3.6  109
Heat energy wasted per minute = Heat absorbed per minute – useful work done per minute
= 3.6 × 109 – 5.4 × 108 = 109 (3.6 – 0.54) = 3.06 × 109 J

9. Logarithms of readings of pressure and volume for an


ideal gas were plotted on a graph sa shown in figure. By
measuring the gradient, it can be shown that the gas 2.40
may be –
2.30
logP(kPa)

(A) monoatomic and undergoing an adiabatic change


2.20
(B) monoatomic and undergoing an isothermal change
(C) diatomic and undergoing an adiabatic change 2.10
(D) triatomic and undergoing an isothermal change
Sol. (C). log P = m log V (where m is slope) 1.10 1.20 1.30
logV(dm³)
2.38  2.10
m  1.4
1.1  1.3
log P = – 1.4 log V ; log PV1.4 = 0 ; PV1.4 = k

Page # 45
HEAT & THERMODYNAMICS PHYSICS PART - IV

10. One mole of an ideal gas at pressure P0 and temperature T0 is expanded isothermally to twice its volume
and then compressed at constant pressure to (V0/2) and the gas is brought back to original state by a
process in which P  V (Pressure is directly proportional to volume). The correct representation of
process is –

P P P
V
P0
P0 V0

(A) P0 (B) (C) P0/2 (D) V0/2


V V 0/2 V0 V T T0
T
V0/2 V0 2V0 2V0 T0/4 T0

Sol. (C). P
Process AB is isothermal expansion,
A
BC is isobaric compression and in process CA P0
nRT C
P  P2  T P0/2 B
P

T/4
0 T0

11. A partition divides a container having insulated walls into two compartments I and II. The same gas fills
the two compartments whose initial parameters are given. The partition is a conducting wall which can
move freely without friction. Which of the following statements is/are correct, with reference to the
final equilibrium position –
(A) The pressure in the two compartments are equal

3V P,V,T 2P,2V,T
(B) Volume of compartment I is I
5 II

12V 5P
(C) Volume of compartment II is (D) Final pressure in compartment I is
5 3
Sol. (ABCD). In the equilibrium position the net force on the partition will be zero.
Hence pressure on both sides are same. Hence, (A) is correct.
Initially, PV = nRT

P1 V1 PV (2P)(2V) PV
n1   and n2  4  n2  4n1
RT1 RT RT RT
Moles remains conserved.
Finally, pressure becomes equal in both parts.
Using, P1V1 = n1RT1, P2V2 = n2RT2  P1 = P2 and T1 = T2
V1 n1 1
    V2  4V1
V2 n2 4
3 12
Also,V1 + V2 = 3V  V1 + 4V1 = 3V  V1  V and V2  V
5 5
Hence (B) and (C) are correct.
In compartment (I) : P1V1  n1RT1

 3V   PV  5PV 5
P1     RT  R (T) ; P1   P Hence (D) is also correct
 5    3V 3

Page # 46
HEAT & THERMODYNAMICS PHYSICS PART - IV

12. An ideal gas expands from state (P1, V1) to state (P2, V2) where V2 = 2V1. Path of the gas is expressed by

  V  V 2 
 1 
the following relation P = P1 1   V   . Find the work in this process .
  1  

2v1  2 2v 1
2v1
 V  V1   v 2  v12  2vv1 
W=  Pdv =  P1 1     dv   1   dv
v1 v1   V1    W = P1 v1

 v12  v

 

2V
V3 2V2 
1

 W = P1 2V  2  2V   W = 4/3 P1V1
 3V1 1
 V1

13. An electric heater supplies heat to a system at a rate of 100W. If system performs work at a rate of 75
joules per second. At what rate is the internal energy increasing?
Sol. Heat supplied, Q = 100 W = 100 J/s
Useful work done, W = 75 J/s
Increase in internal energy per sec, U = ?
As, Q = U + W U = Q – W = 100 – 75 = 25 J/S
14. P-T diagram is shown below the choose the corresponding V-T diagram.

P
A C

B D
T
V V V
V
D B C D
A D B
B
(A) (B) (C) (D) C
B C A C A D A
T T T T

Sol. (D). BC is isochoric. VB  VA , VB  VC , VD  VC

15. geyser heats water flowing at the rate of 3.0 litres per minute from 27 °C to 77 °C. If the geyser operates
on a gas burner, what is the rate of consumption of the fuel if its heat of combustion is 4.0 × 104 J/g ?
Sol. Here, m = mass of water flowing per unit time = 3000g /min.
t = 77 – 27 = 50°C
Heat of combustion = 4 × 104 J/g
3000g 4.2 J
Heat required to heat the water , Q  ms t =   50C = 63 × 104 J/min.
mm Cg
4
Fuel required per min 63  10
Rate of consumption of the fuel = =  15.675 g / min.
Heat of combustion 4  104

Page # 47
HEAT & THERMODYNAMICS PHYSICS PART - IV

16. One mole of an ideal monatomic gas is caused to go through the cycle
shown in figure. Then the change in the internal energy in expanding the
gas from a to c along path abc is Tc
2p0

Pressure
c
(A) 3P0V0 (B) 6RT0
T0
p0 b
(C) 4.5RT0 (D) 10.5 RT0 a

pv
Sol. (D).  T = nR = constant v0 4v0
Volume
For any state of an ideal gas. Therefore
Pa Va Pc Vc P0 V0 2P0 4V0
Ta
= T or T0
= T ; Tc = 8T0
C c
3 21
Thus change in internal energy, U = nCvT = 1 × × R × 7T0 = RT
T0 = 10.5 RT
T0
2 2
17. An ideal gas is taken through a cycle A B  C  A as shown in figure. If the heat supplied in the cycle is
5J, then work done by the gas in the process C  A is–
(A) –5J (B) –10J (C) –15J (D) –20J
(m³)
Sol. (C).
2 C B
Since in cyclic process U = 0
Hence according to first law of thermodynamics
 (  Q)   (  W)
1
A
 ( W)  WAB  WAB  WCA (Pa)
(0,0) 20
WAB  PV (isobaric)  20(2  1)  20J
WBC = 0 (isochoric), so
5  20  WCA or WCA = – 15 J
18. A sample of an ideal gas is in a cylinder fitted with a piston. As 5.76 kJ of heat is supplied to the gas to raise
its temperature, the pressure of gas adjusted so that the state of the gas changes from point A to point B
along the semicircle in P-V graph shown in figure. Find the change in internal energy of the gas (in kJ).
Take  = 22/7
Sol. Work done by the gas P(kPa)

 500
W  [200  2.8]  (300  5.6)
2 400
300 A B
W = 2560 J = 2.56 kJ 200
By first law of thermodynamics 100

U  Q  W  (5.76  2.56)kJ  3.20 kJ 1.4 4.2 7.0

19. A horizontally insulated cylindrical vessel of length 2 is separated by a thin insulating piton into two
equal parts each of which contains n moles of an ideal monoatomic gas at temperature T. The piston is
connected to the end faces of the vessel by undeformed springs of force constant k each.
The left part is in contact with a thermostat (a device which maintains a constant temperature). When an
amount of heat Q is supplied to the gas in the right part, the piston is displaced to left by a distance x = /
2. Determine the heat Q’ given away at the temperature T to the thermostat by the left part of the piston.
Sol. Total heat given to the system is Q – Q’. So from first law of thermodynamics,
Q – Q’ = total work done by the gas in both the chambers (W) + change in internal energies of both the
gases (U) .......... (1)
Page # 48
HEAT & THERMODYNAMICS PHYSICS PART - IV

Here W = sum of potential energies stored in the springs


 1   2  k2
W 2 k    .............. (2)
 2  2   4
Since the temperature of left part remains constant (piston does not conduct heat), internal energy of left
part does not change. U of right part can be given as
3
U  nC V T  nRT .............. (3)
2
T can be found from the condition of equilibrium at the end of the process.
pressure on right side = pressure on left side
nR(T  T) nRT 2K( / 2)
or  
A(   / 2) A(   / 2) A
simplifying this we get :
3k2
or T   2T ............. (4)
2nR
From equations (1), (2), (3) and (4), we get
k2 3  3k2  5
Q  Q   nR   2T  or Q  Q  k2  3nRT
4 2  2nR 
 2
20. A piston divides a closed gas cylinder into two parts. Initially the piston is kept pressed such that one part
has a pressure P and volume 5V and the other part has pressure 8P and volume V, the piston is now left
free. Find the new pressure and volume for the isothermal and adiabatic process. [ = 1.5].
Sol. Final pressure will be same on both sides. Let it be P’, with volume V’, in the left side and (6V – V’) in the
right side
(A) If the change is isothermal :
For the gas enclosed in the left chamber, P 8P
P × 5V = P’V’ .......... (1) 5V L
V
while for the gas in the right chamber,
8P × V = P’ (6V – V’) .......... (2)
Solving these for V’ and P’, we get P' P'
V' L (6V–V')
30 13
V  V and P  P
13 6
48
and (6V  V)  V
13
(B) If the change is adiabatic : For the gas in the left chamber,

P(5V )  P (V  ) .......... (3)


and for the gas in the right chamber.

8P(V )  P (6V  V ) .......... (4)


Dividing (4) by (3)
3/2
 6V  V   8 6V 4 10
 V   3/2 or 1 i.e., V   V
  5 V  5 3

Substituting it in eq. (3),


3/2
 5V  3  3 3 10 8
P  P    P  1.84P So P' = 1.84 P, V  V and (6V  V )  V
 10V  2 2 3 3

Page # 49
HEAT & THERMODYNAMICS PHYSICS PART - IV

EXERCISES
ONLY ONE CHOICE IS CORRECT
THERMAL EXPANSION
1. Two iron spheres of same diameter are heated to the same temperature. One is solid and other is hollow.
Which will expand more.
(A) hollow (B) solid
(C) same for hollow and solid (D) data insufficient
2. Two rods of lengths l1 and l2 are made of materials whose coefficients of linear expansion are 1 and  2 .
If the difference between the two lengths is independent of temperature, then

l1 1 l1 2 12 22
(A) l   (B) 
l2 1 (C) 2 2
l 1  l 2 (D) 
2 2
2 1 l1 l2
3. The radius of a ring is R and its coefficient of linear expansion is  . If the temperature of ring increases
by  , then its circumference will increase by y
(A) R (B) 2R (C) R /2 (D) R / 4
4. A cylindrical metal rod of length L0 is shaped into a ring with a small gap as shown. On heating the system

(A) x decreases, r and d increase (B) x and r increase, d decrease


(C) x, r and d all increase (D) Data insufficient to arrive at a conclusion
5. What is specific heat of ice melting at 0°C?
(A) zero (B) infinity (C) 2100 J / Kg 0C (D) 4200 J / Kg 0C
6. Equal masses of three liquids A, B and C have temperatures 10°C, 25°C and 40°C respectively. If A and B
are mixed, the mixture has a temperature of 15°C. If B and C are mixed the mixture has a temperature of
30°C. If A and C are mixed, the mixture will have a temperature of
(A) 16°C (B) 20°C (C) 25°C (D) 29°C
7. If there is no heat losses, the heat released by the condensation of x grams of steam at 100°C into water at
100°C converts y grams of ice at 0°C into water at 100°C. The ratio y/x is
(A) 1 : 1 (B) 2 : 1 (C) 3: 1 (D) 4:1
8. Latent heat of steam = 536 calories per gm. Latent heat of ice = 80 cal/gm. Specific heat of
ice = 0.5 cal/gm degree °C. The heat required to convert 10 gm of ice at –20°C into steam at 100°C is
(A) 100 calories (B) 900 calories (C) 9900 calories (D) 7260 calories
9. One mole of a mono atomic ideal gas is mixed in the one mole of a diatomic gas. What is molar specific
heat of mixture at constant volume?
(A) 3R (B) 2R (C) 4R (D) 5R
10. A solid and a hollow sphere of copper of same radius R are heated to the same temperature and allowed
to cool in the same environment which of them cool faster?
(A) hollow (B) solid
(C) same for hollow and solid (D) data insufficient

Page # 50
HEAT & THERMODYNAMICS PHYSICS PART - IV

11. Two bodies are at temperature 27°C and 927°C. The heat energy radiated by them will be in the ratio
(A) 1 : 256 (B) 1 : 64 (C) 1:4 (D) 1 : 16
12. Three rods of the same dimensions have thermal conductivities 3k, 2k, and k. They are arranged as
shown, with their ends at 100°C, 50°C and 0°C. The temperature of their junctions is

50°C
2k
100°C
3k
k
0°C

200 100
(A) 75°C (B) C (C) 40°C (D) C
3 3
13. Two spheres P and Q of same colour having radii 8 cm and 2 cm are maintained at temperature 127°C
and 527°C respectively. The ratio of energy radiated by P and Q is
(A) 0.054 (B) 0.0034 (C) 1 (D) 2
14. Two rods of equal length and diameter but of thermal conductivities 2 and 3 units respectively are
joined in series. The thermal conductivity of the combination is
(A) 6 (B) 5 (C) 1 (D) 2.4
15. What is the effect on the temperature of a gas when its container is moved at a constant speed?
(A) increases (B) decreases (C) no change (D) data insufficient
16. What is change in internal energy of an ideal gas in adiabatic free expansion?
(A) zero (B) work done by ideal gas
(C) negative work done by ideal gas (D) data insufficient
17. Two gases have same initial pressure, volume & temperature. They expand to same final volume. One
adiabatically and other isothermally. In which case is final pressure greater ?
(A) isothermal (B) adiabatic (C) same (D) data insufficient
18. A cyclic process ABCD is shown in the p  V diagram. Which of the following curves represent the same
process ?

A B

P PV=Constant

D C
V

C B A B
A B
D
(A) (B) B (C) A (D)
D V p C V
T A D C
C D
p T T T

19. Two containers of equal volume contain the same gas at pressure p1 and p2 and absolute temperatures T1
and T2 respectively. On joining the vessels, the gas reaches a common pressure p and a common
temperature T. The ratio p/T is equal to

Page # 51
HEAT & THERMODYNAMICS PHYSICS PART - IV

p1 p2 1  p1 p2  p1T2  p2T1 p1T2  p2T1


(A)  (B)    (C) (D)
T1 T2 2  T1 T2  T1  T2 T1  T2
20. In the process PV = constant, pressure (P) versus density () graph of an ideal gas is
(A) a straight line parallel to P-axis (B) a straight line parallel to -axis
(C) a straight line passing through origin (D) a parabola
21. A cyclic process is shown in the p-T diagram. Which of the curves show the same process on a p-V
diagram ?

C B

p
A
O T

C B C B C B
(A) (B) (C) (D)
p p p p
A A A A C
B
V V V V

22. An ideal monoatomic gas is taken round the cycle ABCDA as shown in following P-V diagram. The work
done during the cycle is
(3P, V) (3P, 3V)
C D

P
B A
(P, V) (P, 3V)
O V
(A) PV (B) 2PV (C) 4PV (D) zero.
23. The internal energy of a system remains constant when it undergoes
(A) Isobaric process (B) an isothermal process
(C) an adiabatic process (D) Isochoric Process
24. Internal energy of n1 moles of hydrogen at temperature T is equal to the internal energy of n2 moles of
helium at temperature 2T. Then the ratio n1/n2 is

3 2 6 3
(A) (B) (C) (D)
5 3 5 7
25. Ideal gas is taken through the process shown in the figure : P
(A) in process AB, work done by system is positive C
B
(B) in process AB, heat is rejected
(C) in process AB, internal energy increases
A
(D) in process AB internal energy decreases and in process
T
BC, internal energy increases
26. The volume thermal expansion coefficient of an ideal gas at constant pressure is
1 1
(A) T (B) T 2 (C) (D) .
T T2

Page # 52
HEAT & THERMODYNAMICS PHYSICS PART - IV

27. The specific heat of solids at low temperatures varies with absolute temperature T according to the
relation S = AT3, where A is a constant. The heat energy required to raise the temperature of a mass m of
such a solid from T = 0 to T = 20 K is :
(A) 4 × 104 mA (B) 2 × 103 mA (C) 8 × 106 mA (D) 2 × 106 mA.
28. The temperature of an isolated black body falls from T1 to T2 in time t. Let c be a constant, then find t?
1 1 1 1 1 1 1 1
(A) t  c    (B) t  c  2
 2 (C) t  c 3  3  (D) t  c 4  4 
 2 T1 
T  2 T1 
T  T2 T1   T2 T1 
29. A body with an initial temperature i is allowed to cool in a surrounding which is at a constant temperature
of 0 (0  i ) . Assume that Newton’s law of cooling is obeyed. Let k = constant. The temperature of the
body after time t is best expressed by

(A) (i  0 )e  kt (B) (i  0 )ln (kt) (C) 0  ( i  0 )e  kt (D) i e kt  0


30. Three conducting rods of same material and cross–section are shown in figure. Temperatures of A, D
and C are maintained at 20ºC, 90 ºC and 0 ºC. The ratio of lengths of BD and BC if there is no heat flow in
AB is
(A) 2/7 (B) 7/2 (C) 9/2 (D) 2/9.
32. Certain amount of an ideal gas are contained in a closed vessel. The vessel is moving with a constant
velocity v. The molecular mass of gas is M. The raise in temperature of the gas when the vessel is suddenly
stopped is : ( (   Cp / C v )

Mv 2 Mv 2(   1) Mv 2 Mv 2 
(A) (B) (C) (D) .
2R(   1) 2R 2R 2R(   1)
33. A horizontal cylinder has two sections of unequal cross-sections, in which two pistons can move freely.
The pistons are joined by a string. Some gas is trapped between the pistons. If this gas is heated, the
pistons will

(A) move to the left (B) move to the right


(C) remain stationary (D) either (a) or (b) depending on the initial pressure of the gas
34. Pressure versus temperature graph of an ideal gas of equal number of moles of different volumes are
plotted as shown in figure. Choose the correct alternative :
P
4
2
3
1
T

(A) V1 = V2, V3 = V4 and V2 > V3 (B) V1 = V2, V3 = V4 and V2 < V3


(C) V1 = V2 = V3 = V4 (D) V4 > V3 > V2 > V1
35. Volume versus temperature graph of two moles of helium gas is as shown in figure. The ratio of heat
absorbed and the work done by the gas in process 1–2 is
V
2

(A) 3 (B) 5/2 (C) 5/3 (D) 7/2.

Page # 53
HEAT & THERMODYNAMICS PHYSICS PART - IV

36. In the P–V diagram shown in figure ABC is a semicircle. The work done in the process ABC is
P(atm)
3 C
B
1 A

V(L)
1 2

 
(A) zero (B) atm  L (C)  atm  L (D) 4 atm–L.
2 2
37. A thermodynamic system undergoes cyclic process ABCDA as shown in figure. The work done by the
system is
P
C B
3P0
2P0 O
P0 D
A
V0 V
2V0

P0 V0
(A) P0 V0 (B) 2 P0 V0 (C) (D) zero.
2
38. In a cyclic process shown in the figure an ideal gas is adiabatically taken from B to A, the work done on
the gas during the process B to A is 30 J, when the gas is taken from A to B the heat absorbed by the gas
is 20 J. The change in internal energy of the gas in the process A to B is :
P
A
20J

30J B

V
(A) 20 J (B) –30 J (C) 50 J (D) –10 J
39. An ideal monoatomic gas undergoes a cyclic process ABCA as shown in the figure. The ratio of heat
absorbed during AB to the work done on the gas during BC is
V
B

A
V0 C

T
T0 2T0

5 5 5 5
(A) (B) (C) (D) .
2ln2 3 4ln2 6
40. A gas is expanded from volume V0 to 2V0 under three different processes. Process 1 is isobaric, process
2 is isothermal and process 3 is adiabatic. Let U1 , U 2 and U 3 be the change in internal energy of
the gas in these three processes. Then :

P
1
P0
2

3
V
V0 2V0

Page # 54
HEAT & THERMODYNAMICS PHYSICS PART - IV

(A) U1  U2  U3 (B) U1  U2  U3

(C) U2  U1  U3 (D) U2  U3  U1 .


41. P–V diagram of an ideal gas is as shown in figure. Work done by the gas in the process ABCD is :
P
C D
2P0

P0 A
B
V
V0 2V0 3V0

(A) 4 P0 V0 (B) 2 P0 V0 (C) 3 P0 V0 (D) P0 V0.


42. P–V diagram of a diatomic gas is a straight line passing through origin. The molar heat capacity of the gas
in the process will be :

4R
(A) 4 R (B) 2.5 R (C) 3 R (D) .
3
43. The figure shows two paths for the change of state of a gas from A to B. The ratio of molar heat capacities
in path 1 and 2 is :
P
2
A B
1 4
V 0
3
(A) > 1 (B) < 1 (C) 1 (D) data insufficient.
45. 3 moles of an ideal mono atomic gas performs a cycle as shown in fig. If gas temperature TA  400 K
TB  800K, TC  2400 K, and TD = 1200K. Then total work done by gas is

P B C

A
D

(A) 2400 R (B) 1200 R (C) 2000 R (D) Zero

46. When a block of iron floats in mercury at 0°C, a fraction k1 of its volume is submerged, while at the
temperature 60°C, a fraction k2 is seen to be submerged if the coefficient of volume expansion of iron is
 Fe and that of mercury is  Hg , then the ratio k1 / k2 can be expressed as

1  60 Fe 1  60 Fe 1  60 Fe 1  60 Hg


(A) 1  60 (B) 1  60 (C) 1  60 Hg (D)
Hg Hg 1  60 Fe

P0
P 2
47. One mole of an ideal gas undergoes a process V 
1   0  . Here, P0 and V0 are constants. Change in
V
temperature of the gas when volume is changed from V = V0 to V = 2V0 is :

2P0 V0 11P0 V0 5P0V0


(A)  (B) (C)  (D) P0 V0 .
5R 10R 4R

Page # 55
HEAT & THERMODYNAMICS PHYSICS PART - IV

48. Pressure versus temperature graph of an ideal gas is shown in figure. Density of the gas at point A is 0 .
Density at B will be

P
3P0 B

P0 A

T
T0 2T0

3 3
(A) 0 (B) 0 (C) (D) 20 .
4 2
49. In a vertical U-tube containing a liquid, the two arms are maintained at different temperatures, t1 and t2.
The liquid columns in the two arms have heights l1 and l2 respectively. The coefficient of volume expansion
of the liquid is equal to

t12

t1 L1
L2

l1  l2 l1  l2 l1  l2 l1  l2
(A) (B) (C) (D)
l2 t 1  l1 t 2 l1 t 2  l2 t 2 l2t 1  l1 t 2 l1 t 1  l2t 2
50. Three rods of equal length are joined to form an equilateral triangle ABC as shown in figure. D is the
midpoint of AB. The coefficient of linear expansion is 1 for AB, and  2 for AC and BC. If the distance DC
remains constant for small changes in temperature, then
A D 1 B

2 3

C
1
(A) 1   2 (B) 1  2 2 (C) 1  4 2 (D) 1  2
2
51. A and B are two points on a uniform metal ring whose centre is C. The angle ABC =  . A and B aree
maintained at two different constant temperatures. When  = 180°, the rate of total heat flow from A to
o
B is 1.2 W. When  = 90°, this rate will be
(A) 0.6 W (B) 0.9 W (C) 1.6 W (D)1.8 W
52. One end of a uniform rod of length 1 m is placed in boiling water while its other end is placed in melting
ice. A point P on the rod is maintained at a constant temperature of 800°C. The mass of steam produced
per second is equal to the mass of ice melted per second. If latent heat of steam is 7 times the latent heat
of ice, the distance of P from the stem chamber must be
1 1 1 1
(A) m (B) m (C) m (D) m
7 8 9 10

Page # 56
HEAT & THERMODYNAMICS PHYSICS PART - IV

53. An ideal gas is expanding such that PT2 = constant. The coefficient of volume expansion of the gas is
1 2 3 4
(A) (B) (C) (D) .
T T T T
54. During an experiment, an ideal gas is found to obey a condition p2/  = constant. The gas is initially at
temperature T, pressure P and density  . The gas expands such that density changes to  /2 and
(A) The pressure of the gas changes to 2P

(B) The pressure of the gas changes to 2T


(C) The graph of the above process on the P-T graph is parabola
(D) The graph of the above process on the P-T graph is hyperbola
55. A metal block of density 5000 kg/m3 and mass 2 kg is suspended by a spring of force constant 200 N/m.
The spring block system is submerged in water vessel. Total mass of water in vessel is 300 gm and in
equilibrium the block is at a height 40 cm above the bottom of vessel. The specific heat of material of
block is 250J/kg/k and that of water is 4200 J/kg/k. Neglect the heat capacities of vessel end the spring.
If the support is broken the rise in temperature of water, when block reaches bottom of vessel is
(A) 0.0012°C (B) 0.0049°C (C) 0.0028°C (D) 0.0°C
56. A vessel of water equivalent W kg contains m kg of water of specific heat S. When water evaporates at
the rate of  kgS-1, the temperature of the vessel and water in it falls from T1 C to T2 C in t s. If m>>  t
and a fraction Eof the heat needed for evaporation is taken from the vessel and the water then average
rate of fall of temprature is
(L = average latent heat of vapourisation in J kg–1)

 L   L  L
(A) ms  w C /S (B) L(ms  w)
C / s (C) C / s (D) C / s
w ms

57. A small spherical body of radius r is falling under gravity in a viscous medium and due to friction the
medium gets heated. When body attains terminal velocity then rate of heating is proportional to

(A) r (B) r 3 / 2 (C) r5 (D) r1/ 2

58. One mole of Argon undergoes a process given by PV3 / 2  const. If heat obtained by gas is Q and molar

specific heat of gas in the process is C then which of the following is correct if temperature of gas
changes by -26 K (assume Argon as an ideal gas)

(A) C = 0.5 R, Q = 13 R (B) C = -0.5 R, Q = 1.3 R

(C) C = -0.5 R, Q = 13 R (D) C = 0, Q = 13 R

Page # 57
HEAT & THERMODYNAMICS PHYSICS PART - IV

ONE OR MORE THAN ONE CHOICE MAY BE CORRECT


1. A metal rod is shaped into a ring with a small gap. If this is heated,
(A) the length of the rod will increase
(B) the gap will decrease
(C) the gap will increase
(D) the diameter of the ring will increase in the same ratio as the length of the rod

cp
2. A gas with   goes from an initial state (p , V , T ) to a final state (p , V , T ) through an adiabatic
cV 1 1 1 2 2 2

process. The work done by the gas is

nR(T1  T2 ) p1 V1  p2 V2 p1 V1  p2 V2
(A) (B) (C) (D) nR(T1  T2 )
 1  1  1
3. The internal energy of a system remains constant when it undergoes
(A) a cyclic process
(B) an isothermal process
(C) an adiabatic process
(D) any process in which the heat given out by the system is equal to the work done on the system
4. The molar heat capacity for an ideal gas
(A) is zero for an adiabatic process
(B) is infinite for an isothermal process
(C) depends only on the nature of the gas for a process in which either volume or pressure is constant
(D) is equal to the product of the molecular weight and specific heat capacity for any process
5. A system undergoes a cyclic process in which it absorbs Q1 heat and gives out Q2 heat. The efficiency of
the process is h and the work done is W

W Q1 Q2
(A) W  Q1  Q2 (B)   (C)  (D) 1
Q1 Q2 Q1

mkT
6. The quantity of an ideal gas depends on (m = mass of the gas) :
V
(A) temperature of the gas (B) volume of the gas
(C) pressure of the gas (D) nature of the gas.
7. Which of the following quantities is independent of the nature of the gas at same temperature ?
(A) the number of molecules in 1 mole (B) the number of molecules in equal volume
(C) the translational kinetic energy of 1 mole (D) the kinetic energy of unit mass.
8. Which of the following quantities depend on temperature only for a given ideal gas ?
(A) internal energy of the gas (B) product PV of the gas
(C) the ratio of pressure and density of the gas (D) root mean square speed of the gas.
9. Temperature versus pressure graph of an ideal gas is shown in figure. During the process AB :
(A) internal energy of the gas remains constant T

(B) volume of the gas is increased A B

(C) work done by the atmosphere on the gas is positive


P
(D) pressure is inversely proportional to volume.

Page # 58
HEAT & THERMODYNAMICS PHYSICS PART - IV

10. Figure shows the P-V diagram of a cyclic process. If dQ is the heat energy supplied to the system, dU is
change in the internal energy of the system and dW is the work done by the system, then which of the
following relations is/are correct

C
B
P
D

(a) dQ = dU - dW (b) dU = 0 (c) dQ = dW (d) dQ = - dW


11. The first law of Thermodynamics is based on
(A) the law of conservation of energy (B) the law of conservation of heat
(C) the law of conservation of work (D) the equivalence of heat and work
12. A metal rod of length L0, made of material of Young’s modulus Y, area A is fixed between two rigid supports. The coefficient
of linear expansion of the rod is  . The rod is heated such that the compressive force in the rod is T
(A) T  L0 (B) T  1/  (C) T  A (D) T Y
13. A gas expends such that its initial and final temperatures are equal. Also, the process followed by the gas
traces a straight line on the p–V diagram
(A) the temperature of the gas remains constant throughout
(B) the temperature of the gas first increases and then decreases
(C) the temperature of the gas first decreases and then increases
(D) the straight line has a negative slope
14. The following are the p–V diagrams for cyclic processes for a gas. In which of these processes is heat
absorbed by the gas ?

(A) p (B) V (C) p (D) V

V p V p

15. Two gases have the same initial pressure, volume and temperature. They expand to the same final
volume, one adiabatically and the other isothermally
(A) the final temperature is greater for the isothermal process
(B) the final pressure is greater for the isothermal process
(C) the work done by the gas is greater for the isothermal process
(D) all the above options are incorrect
16. In the previous question, if the two gases are compressed to the same final volume
(A) the final temperature is greater for the adiabatic process
(B) the final pressure is greater for the adiabatic process
(C) the work done on the gas is greater for the adiabatic process
(D) all the above options are incorrect
17. A spherical black body of radius r radiates power P, and its rate of cooling is R

1
(A) P  r (B) P  r2 (C) R  r2 (D) R
r

Page # 59
HEAT & THERMODYNAMICS PHYSICS PART - IV

18. The curves A and B in the figure shown on P-V graphs. Then choose the correct a statement :

(A) A is adiabatic process (B) A is isothermal process


(C) B is adiabatic process (D) B is isothermal process
19. Two rods of length L1 and L2 are made of materials of coefficients of linear expansions 1 and  2
respectively such that L11  L 2  2 . The temperature of the rods is increased by T and correspondingly
ly
the change in their respective lengths be L1 and L 2

(A) L1  L2 (B) L1  L2


(C) The difference in the length (L1 – L2) is a constant and is independent of rise of temperature
(D) Data is insufficient to arrive at a conclusion
20. Two spheres A and B have same radius but the heat capacity of A is greater than that of B. The surfaces
of both are painted black. They are heated to the same temperature and allowed to cool. Then :
(A) a cools faster than B]
(B) both A and B cool at the same rate
(C) at any temperature the ratio of their rates of cooling is a constant
(D) B cools faster than A.
21. P–V diagram of a cyclic process ABCA is as shown in figure. Choose the correct statement (s) :
P
A

B
C

(A) Q A B  negative (B) U BC  positive

(C) UCA  negative (D) WCAB  negative .


22. During the process A–B of an ideal gas :
P
B

(A) work done on the gas is zero


(B) density of the gas is constant
(C) slope of line AB from the T–axis is inversely proportional to the number of moles of the gas
(D) slope of line AB from the T–axis is directly proportional to the number of moles of the gas.
23. 1 kg of ice at 0ºC is mixed with 1.5 kg of water at 45ºC [latent heat of fusion = 80 cal/g]. Then
(A) the temperature of the mixture is 0ºC (B) mixture contains 156.25 g of ice
(C) mixture contains 843.75 g of ice (D) the temperature of the mixture is 15ºC.

Page # 60
HEAT & THERMODYNAMICS PHYSICS PART - IV

24. In the arrangement shown in figure. Gas is thermally insulated. An ideal gas is filled in the cylinder having
pressure P0 (> atmospheric pressure Pa ). Spring of force constant k is initially unstretched. Piston of
mass m and area S is frictionless. In equilibrium piston rises up a distance x0, then :

kx 0 mg
(A) final pressure of the gas is Pa  
S S k

m, S
1 2
(B) work done by the gas is kx 0  mgx 0 P0
2

1 2
(C) decrease in internal energy of the gas is kx 0  mg x0  PaS x0
2

1 2
(D) work done by the gas is kx 0 .
2
25. One mole of an ideal monoatomic gas is taken from A to C along the path ABC. The temperature of the gas
at A is T0. For the process ABC :
(A) work done by the gas is RT0

11
(B) change in internal energy of the gas is RT0
2 P
2P0 C

11 P0 B
(C) heat absorbed by the gas is RT0 A
2 V
V0 2V0

13
(D) heat absorbed by the gas is RT0 .
2
(R = universal gas constant)
26. The temperature drop through a two layer furnace wall is 900ºC. Each layer is of equal area of cross
section. Which of the following actions will result in lowering the temperature  of the interface ?
(A) by increasing the thermal conductivity of outer layer Inner layer Outer layer

(B) by increasing thermal conductivity of inner layer


1000ºC

100ºC

(C) by increasing thickness of outer layer


(D) by increasing thickness of inner layer. 

27. In a thermodynamic process helium gas obeys the law TP–2/5 = constant. If temperature of 2 moles of the
gas is raised from T to 3T, then :
(A) heat given to the gas is 9RT (B) heat given to the gas is zero
(C) increase in internal energy is 6RT (D) work done by the gas is –6RT.
2
28. A gas is found to obey the law P V = constant. The initial temperature and volume are T0 and V0. If the gas
expands to a volume 3V0, then :
(A) final temperature become (B) internal energy of the gas will increase

T0
(C) final temperature becomes (D) internal energy of the gas decreases.
3

Page # 61
HEAT & THERMODYNAMICS PHYSICS PART - IV

PASSAGE BASED QUESTIONS


PASSAGE-I
A source of heat supplies heat at constant rate to a solid cube. The variation of temperature of the cube
with heat supplied is shown in figure

1. The portion DE of the graph represents conversion of


(A) Solid into liquid (B) Liquid into vapour
(C) Solid into vapour (D) Vapour into liquid
2. The reciprocal of the slope of the portion EF of the graph shown in figure represents
(A) Latent heat of fusion (B) Latent heat of vaporisation
(B) Thermal capacity of the liquid (D) Thermal capacity of the vaporisation
3. In figure it is observed that DE = 3 BC. This means that
(A) The thermal capacity of the vapour is 3 times that of the liquid
(B) The specific heat of the vapour is 3 times that of the liquid
(C) The latent heat of vaporisation of the liquid is 3 times that of latent heat of fusion of the solid
(D) The latent heat of fusion of the solid is 3 times that of latent heat of vaporization of the liquid

PASSAGE-II
A certain amount of ice is heated at a constant rate for 14 minutes. For the first 4 minutes the temperature
rises uniformly with time. Then it remains constant for 8 minutes and again the temperature rises at
uniform rate for the last 2 minutes. (Latenet heat of ice = 80cal , specific heat of ice = 0.5 cal , specific of
water =1 cal )
Choose the correct answer
4. What is the initial temperature of ice ?
(A) –40°C (B) –80°C (C) 0°C (D) –20°C
5. The temperature at the end of 14 minutes is
(A) 120°C (b) 40°C (c) 20°C (d) 100°C
6. If the mass of ice is 6 gm then the rate at which constant power is supplied
(a) 1 W (b) 1 cal/s (c) 4W (d) 0.5 W

PASSAGE-III
The rectangular box shown in the figure has a partition which can slide without friction along the length
of the box. Initially each of two chambers of the box have one mole of a mono-atomic ideal gas (g = 5/3)
at a pressure P0, volume V0 and temperature T0. The chamber on the left is slowly heated by an electric
heater. The walls of box and partition are thermally insulated. Heat loss through lead wire of heater is
negligible. The gas in left chamber expands, pushing the partition until the final pressure in both chambers
becomes 243p0/32.

Page # 62
HEAT & THERMODYNAMICS PHYSICS PART - IV

7. Final temperature of the gas in right chamber is


(A) 2.25 T0 (B) 4.5 T0 (C) 8.75 T0 (D) 12.93 T0
8. Final temperature of the gas in left chamber is
(A) 2.25 T0 (B) 4.5 T0 (C) 8.75 T0 (D) 12.93 T0
9. The work done by the gas in the right chamber is
(A) 5.5 T0 J (B) 10.5 T0 J (C) 25.5 T0 J (D) None of these

PASSAGE-IV
Two mole of an ideal monatomic gas are confined within a cylinder by a mass less spring loaded with a
frictionless piston of negligible mass and crossectional area 4 × 10-3m2. The spring is initially in ill relaxed
state. Now the gas is heated by a heater for some time. During this time the gas expands and does 50J of
work in moving the piston through a distance of 0.01m. The temperature of gas increases by 50k.

10. The force constant of spring is


(A) 189.6 N m-1 (B) 18.96 N m-1 (C) 1896 N m-1 (D) 2896 N m-1
11. Change in internal energy of the gas is
(A) 1246.5 J (B) 124.65 J (C) 200 J (D) 12.46 J
12. Heat supplied by heater during this process is
(A) 129.65 J (B) 1296.5 J (C) 12.96 J (D) 250 J

Page # 63
HEAT & THERMODYNAMICS PHYSICS PART - IV

MATCH THE COLUMN TYPE QUESTIONS

1. The figure shows a cyclic process ABCDA.

Column I (Process) Column II


A. AB (p) W < 0
B. BC (q) Q > 0
C. CD (r) W > 0
D. DA (s) Q < 0
2. Whenever a liquid is heated in a container, expansion in liquid as well as container takes place. If  is the
volume expansion coefficient of liquid and  is coefficient of linear expansion of the container match
the entries of Column I and Column II
Column I Column II
(i) Liquid level rises with respect to container (A)   2
(ii) Liquid level remains same with respect to container (B) 2    3
(iii) Liquid level drops with respect to container (C)   3
(iv) Liquid level remains same with respect to ground (D)   3
3. Match the entries of column I and II.

Column I Column II

(i) (a)

(ii) (b)

(iii) (c)

Page # 64
HEAT & THERMODYNAMICS PHYSICS PART - IV

(iv) (d)

4. Match the entries of column I and II


Column I Column II

(i) (a)

(ii) (b)

(iii) (c)

(iv) (d)

INTEGER TYPE QUESTIONS

1. A lead ball at 30°C is dropped from a height of h. The ball is heated due to the air resistance and it
completely melts just before reaching the ground. The molten substance falls slowly on the ground.
Latent heat of fusion of lead is 22200 J/kg. Specific heat capacity of lead = 126 J/kg–°C and melting point
of lead = 330°C. Assume that all mechanical energy lost is used to heat the ball.
Find the value of h in km ? (Use g = 10 m/s2)
2. One mole of an ideal monatomic gas at temperature T0 expands slowly according to the law P/V =
constant. If the final temperature is 2T0, heat supplied to the gas is xRT0. Then find value of x ?
3. Three identical rods A, B and C of equal lengths and equal diameters are joined in series as shown in
figure. Their thermal conductivities are same as shown. The temperature at junction of A and B is 100k
°C. Find the value of k ?
4 00°C A B C 100° C
K K K

4. A steel rod of length 5 m is fixed between two support. The coefficient of linear expansion of steel is 12.5
× 10–6/°C. Calculate the stress (in 108 N/m2) in the rod for an increase in temperature of 40°C. Young’s
modulus for steel is 2 × 1011 N/m2.

Page # 65
HEAT & THERMODYNAMICS PHYSICS PART - IV

5. 1/R (R is universal gas constant) moles of an ideal gas (  1.5) undergoes a cyclic process (ABCDA) as
shown in figure. Assuming the gas to be ideal. If the net heat exchange is 10x Joules, find the value of x ?
[ln 2 = 0.7]

2×10 3 atm A B

1×10 3 atm C
D
T
300K 400K

6. A metal rod AB of length 10x has its one end A in ice at 0°C and the other end B in water at 100°C . If a
point P on the rod is maintained at 400°C, then it is found that equal amounts of water and ice evaporate
and melt per unit time. The latent heat of evaporation of water is 540 cal/g and latent heat of melting of
ice is 80 cal/g. If the point P is at a distance of  x from the ice end A, find the value of  . [Neglect any
y
heat loss to the surrounding]

7. An ideal gas absorbs 100J of heat and performs 25J of work at constant pressure. Find the number of
degrees of freedom of the gas ?
8. An ideal diatomic gas under goes a process in which its internal energy changes with volume as given
U  cV2/5 where c is constant. Find the ratio of molar heat capacity to universal gas constant R ?
9. An ideal gas is taken through a cyclic thermodynamic process through four steps. The amount of heat
involved in four steps are Q1  6000J,Q2  5000J,Q3  3000J andQ4  4000J

respectively. If efficiency of cycle is 10x% then find value of x?

Page # 66
HEAT & THERMODYNAMICS PHYSICS PART - IV

Questions asked in previous AIEEE / JEE MAINS


PART - I: HEAT TRANSFER

1. Infrared radiations are detected by (AIEEE-2002)


(A) spectrometer (B) pyrometer (C) nanometer (D) photometer
2. Which of the following is more close to a black body ? (AIEEE-2002)
(A) Black board paint (B) Green leaves (C) Black holes (D) Red roses
3. Which of the following radiations has the least wavelength ? (AIEEE-2003)
(A) -rays (B) -rays (C) -rays (D) X-rays
4. If the termpreature of the sun were to increase from T to 2T and its radius from R to 2R, then the ratio
of the radiant energy received on earth to what it was previously will be- (AIEEE-2004)
(A) 4 (B) 16 (C) 32 (D) 64
5. The temperature of the two outer surfaces of a composite slab, consisting of two materials K and 2K and
thickness x and 4x, respectively, are T2 and T1(T2 > T1). The rate of heat transfer through the slab, in a
 A(T2  T1 )K 
steady state is   f with f equal to– (AIEEE-2004)
 x 
x 4x

T2 T1
K 2K

(A) 1 (B) 1/2 (C) 2/3 (D) 1/3


6. The figure shows a system of two concentric spheres of radii r1 and r2 and kept at temperature T1 and T2,
respectively. The radial rate of flow of heat in a substance between the two concentric spheres is propor-
tional to : [AIEEE-2005]

r1 T1

T2
r2

(r2  r1 ) (r )
2
r1r2
(A) (r r ) (B) ln (r ) (C)  r2  r1  (D) (r2 – r1)
1 2 1

7. Assuming the sun to be a spherical body of radius R at a temperature of T K, evaluate the total radiant
power, incident on Earth, at a distance r from the Sun. (earth radius = r0) (AIEEE-2006)

R 2T 4 4 r02 R 2 T 4 r02 R 2 T 4 r02 R 2 T 4


(A) (B) (C) (D)
r2 r2 r2 4 r 2
8. One end of a thermally insulated rod is kept at a temperature T1 and the other at T2. The rod is composed
of two sections of lengths L1 and L2 and thermal conductivities k1 and k2 respectively. The temperature
at the interface of the sections is (AIEEE-2007)

Page # 67
HEAT & THERMODYNAMICS PHYSICS PART - IV

(K 2 L2 T1  K 1 L1 T2 ) (K 2 L1 T1  K 1 L 2T2 ) (K 1 L2 T1  K 2 L1T2 ) (K 1 L1 T1  K 2 L2 T2 )
(A) (K 1L1  K 2L 2 ) (B) (K 2L1  K 1L 2 ) (C) (K 1L 2  K 2L1 ) (D) (K 1L1  K 2L 2 )

9. A long metallic bar is carrying heat from one of its ends to the other end under steady-state. The variation
of temperature  along the length x of the bar from its hot end is best described by which of the following
figures (AIEEE-2009)

(A) (B) (C) (D)

10. A liquid in a beaker has temperature q(t) at time t and q0 is temperature of surroundings, then accord-
ing
to Newton’s law of cooling the correct graph between loge (q – q0) and t is (AIEEE-2012)

11. The above p-v diagram represents the thermodynamic cycle of an engine, operating with an ideal
monoatomic gas. The amount of heat extracted from the source in a single cycle is
(AIEEE-2012)

 13   11 
(A)  2 p 0 v 0 (B)  2 p 0 v0 (C) 4p 0 v 0 (D) p0v0
   
12. Three rods of Copper, Brass and Steel are welded together to from Y-shaped structure. Area of cross-
section of each rod = 4 cm2. End of copper rod is maintained at 100°C where as ends of brass and steel
are kept at 0°C. Lengths of the copper, brass and steel rods are 46, 13 and 12 cms respectively. The rods
are thermally insulated from surroundings except at ends. Thermal conductive of copper, brass and
steel are 0.92, 0.26 and 0.12 CGS units respectively. Rate of heat flow through copper rod is :
(JEE Mains-2014)
(A) 4.8 cal/s (B) 6.0 cal/s (C) 1.2 cal/s (D) 2.4 cal/s
13. The temperature of an open room of volume 30 m3 increases from 17°C to 27°C due to sunshine.
The atmospheric pressure in the room remains 1 × 105 Pa. If ni and nf are the number of molecules in the
room before and after heating, then nf - ni will be :- (JEE MAIN 2017)
(A) 2.5 × 1025 (B) –2.5 × 1025 (C) –1.61 × 103 (D) 1.38 × 1023

Page # 68
HEAT & THERMODYNAMICS PHYSICS PART - IV

14. A copper ball of mass 100 gm is at a temperature T. It is dropped in a copper calorimeter of mass 100 gm,
filled with 170 gm of water at room temperature. Subsequently, the temperature of the system is
found to be 75°C. T is given by : (Given : room temperature = 30° C, specific heat of copper = 0.1 cal/
gm°C (JEE MAIN 2017)
(A) 1250°C (B) 825°C (C) 800°C (D) 885° C

PART - II: KTG & THERMODYNAMICS


(Questions asked in previous AIEEE / JEE MAINS)

1. Which statement is incorrect ? [AIEEE - 2002]


(A) All reversible cycles have same efficiency
(B) Reversible cycle has more efficiency than an irreversible one
(C) Carnot cycle is a reversible one
(D) Carnot cycle has the maximum efficiency in all cycles
2. Cooking gas containers are kept in a lorry moving with uniform speed. The temperature of the gas
molecules inside will : [AIEEE - 2002]
(A) increase (B) decrease
(C) remain same (D) decrease for some, while increase for others
3. At what temperature is the rms velocity of a hydrogen molecule equal to that of an oxygen molecule at 47º C?
[AIEEE - 2002]
(A) 80 K (B) –73 K (C) 3K (D) 20 K
4. Even Carnot engine cannot give 100% efficiency because we cannot : [AIEEE - 2002]
(A) prevent radiation (B) find ideal sources
(C) reach absolute zero temperature (D) eliminate friction
5. 1 mole of a gas with  = 7/5 is mixed with 1 mole of a gas with = 5/3, then the value of  for the resulting
mixture is : [AIEEE - 2002]
(A) 7/5 (B) 2/5 (C) 24/16 (D) 12/7
6. "Heat cannot be itself flow from a body at lower temperature to a body at higher temperature” is a
statement or consequence of : [AIEEE - 2003]
(A) second law of thermodynamics (B) conservation of momentum
(C) conservation of mass (D) first law of thermodynamics
7. During an adiabatic process, the pressure of a gas is found to be proportional to the cube of its absolute
temperature. The ratio Cp/Cv for the gas is : [AIEEE - 2003]
(A) 4/3 (B) 2 (C) 5/3 (D) 3/2
8. Which of the following parameters does not characterise the thermodynamic state of matter?
[AIEEE - 2003]
(A) Temperature (B) Pressure (C) Work (D) Volume
9. A Carnot engine takes 3 × 106 cal of heat from a reservoir at 627ºC and gives it to a sink at 27ºC. The work
done by the engine is : [AIEEE - 2003]
6 6 6
(A) 4.2 × 10 J (B) 8.4 × 10 J (C) 16.8 × 10 J (D) zero
10. One mole of ideal monoatomic gas ( = 5/3) is mixed with one mole of diatomic gas ( = 7/5). What is  for
the mixture?  denotes the ratio of specific heat at constant pressure, to that at constant volume.
[AIEEE - 2004]
(A) 3/2 (B) 23/15 (C) 35/23 (D) 4/3

Page # 69
HEAT & THERMODYNAMICS PHYSICS PART - IV

11. Which of the following statements is correct for any thermodynamic system? [AIEEE - 2004]
(A) The internal energy changes in all processes
(B) Internal energy and entropy are state functions
(C) The change in entropy can never be zero
(D) The work done in an adiabatic process is always zero
12. Two thermally insulated vessels 1 and 2 are filled with air at temperature (T1, T2), volume (V1, V2) and
pressure (P1, P2) respectively. If the valve joining the two vessels is opened, the temperature inside the
vessel at equilibrium will be : [AIEEE - 2004]
T1 T2 (P1 V1  P2 V2 ) T1 T2 (P1 V1  P2 V2 )
(A) T1 + T2 (B) (T1 + T2)/2 (C) P1 V1 T2  P2 V2 T1 (D) P1 V1 T1  P2 V2 T2
13*. Which of the following is incorrect regarding the first law of thermodynamics? [AIEEE - 2005]
(A) It is not applicable to any cycle process
(B) It is a restatement of the principle of conservation of energy
(C) It introduces the concept of the internal energy
(D) It introduces the concept of the entropy
Cp
14. A gaseous mixture consists of 16 g of helium and 16 g of oxygen. The ratio C of the mixture is :
v

[AIEEE - 2005]
(A) 1.59 (B) 1.62 (C) 1.4 (D) 1.54
15. A system goes from A to B via two processes  and  as shown in figure. If U1 and U2 are the changes in
internal energies in the processes  and  respectively, then : [AIEEE - 2005]
P
(A) U1 = U2
(B) relation between U1 and U2 cannot be determined

(C) U2 > U1
A B
(D) U2 < U1 

16. The temperature-entropy diagram of a reversible engine cycle is V


given in the figure. Its efficiency is : [AIEEE - 2005]

1 1 1 2
(A) (B) (C) (D)
2 4 3 3
17. Two rigid boxes containing different ideal gases are placed on a table. Box A contains one mole of nitrogen at
temperature To, while box B contains one mole of helium at temperature (7/3)To. The boxes are then put into
thermal contact with each other, and heat flows between them until the gases reach a common final tem-
perature. (Ignore the heat capacity of boxes). Then, the final temperature of the gases, Tf in terms of T0 is :
[AIEEE - 2006]
3 5
(A) Tf  3 T0 (B) Tf  7 T0 (C) Tf  T0 (D) Tf  T0
7 3 2 2

Page # 70
HEAT & THERMODYNAMICS PHYSICS PART - IV

18. The work of 146 kJ is performed in order to compress one kilo mole of a gas adiabatically and in this
process the temperature of the gas increases by 7 oC. The gas is (R = 8.3 J mol-1 K-1)
[AIEEE - 2006]
(A) diatomic (B) triatomic
(C) mixture of monoatomic and diatomic (D) monoatomic
19. A Carnot engine, having an efficiency of  = 1/10 as heat engine, is used as a refrigerator. If the work done
on the system is 10 J, the amount of energy absorbed from the reservoir at lower temperature is
[AIEEE - 2007]
(A) 99 J (B) 90 J (C) 1J (D) 100 J
20. If C p and C V denote the specific heats of nitrogen per unit mass at constant pressure and constant
volume respectively, then [AIEEE - 2007]
(A) Cp – C v = R / 28 (B) C p – C v = R / 14

(C) Cp – C v = R (D) C p – C v = 28R

21. When a system is taken from state i to state f along the path iaf, it is found that Q = 50 cal and W = 20 cal. Along the path
ibf Q = 36 cal. W along the path ibf is: [AIEEE - 2007]
(A) 6 cal (B) 16 cal (C) 66 cal (D) 14 cal
22. An insulated container of gas has two chambers separated by an insulating partition. One of the cham-
bers has volume V1 and contains ideal gas at pressure p1 and temperature T1. The other chamber has
volume V2 and contains ideal gas at pressure p2 and temperature T2. If the partition is removed without
doing any work on the gas, the final equilibrium temperature of the gas in the container will be -
[AIEEE - 2008]
T1 T2 (p1 V1  p2 V2 ) p1 V1 T1  p 2 V2 T2
(A) p1 V1 T2  p2 V2 T1 (B) p1 V1  p 2 V2

p1V1T2  p 2 V2 T1 T1T2 (p1V1  p 2 V2 )


(C) p1V1  p 2 V2
(D) p1V1T1  p 2 V2 T2

Directions : Question number 23, 24 and 25 are based on the following paragraph.
Two moles of helium gas are taken over the cycle ABCDA, as shown in the P-T diagram.
[AIEEE - 2009]

23. Assume the gas to be ideal the work done on the gas in taking it from A to B is :
(A) 200 R (B) 300 R (C) 400 R (D) 500 R
24. The work done on the gas in taking it from D to A is
(A) –414 R (B) + 414 R (C) – 690 R (D) + 690 R
25. The net work done on the gas in the cycle ABCDA is:
(A) Zero (B) 276 R (C) 1076 R (D) 1904 R
26. One kg of a diatomic gas is at a pressure of 8 × 10 N/m . The density of the gas is 4 kg/m3. What is the
4 2

energy of the gas due to its thermal motion? [AIEEE - 2009]


(A) 5 × 10 J 4 4
(B) 6 × 10 J (C) 7 × 10 J4 (D) 3 × 104J

Page # 71
HEAT & THERMODYNAMICS PHYSICS PART - IV

27. A diatomic ideal gas is used in a Carnot engine as the working substance. If during the adiabatic expansion
part of the cycle the volume of the gas increases from V to 32 V, the efficiency of the engine is :
[AIEEE - 2010]
(A) 0.5 (B) 0.75 (C) 0.99 (D) 0.25
28. 100g of water is heated from 30ºC to 50ºC ignoring the slight expansion of the water, the change in its
internal energy is (specific heat of water is 4184 J/Kg/K) : [AIEEE - 2011]
(A) 4.2 kJ (B) 8.4 kJ (C) 84 kJ (D) 2.1 kJ
1
29. A Carnot engine operating between temperatures T1 and T2 has effeiciency . When T2 is lowered by 62
6

1
K, its efficiency increases to . Then T1 and T2 are, respectively : [AIEEE - 2011]
3
(A) 372 K and 310 K (B) 372 K and 330 K (C) 330 K and 268 K (D) 310 K and 248 K
30. Three perfect gases at absolute temperature T1,T2 and T3 are mixed. The masses of molecules are m1,m2
and m3 and the number of molecules are n1,n2 and n3 respectively. Assuming no loss of energy, the final
temperature of the mixture is : [AIEEE - 2011]

(T1  T2  T3 ) n 1 T1  n 2T2  n 3T3


(A) (B) n 1  n2  n 3
3

n 1 T12  n 2 T22  n 3 T32 n 21 T12  n 22 T22  n 32 T32


(C) (D)
n 1 T1  n 2 T2  n 3 T3 n 1 T1  n 2 T2  n 3 T3

31. A thermally insulated vessel contains an ideal gas of molecular mass M and ratio of specific heats . It is
moving with speed v and is suddenly brought to rest. Assuming no heat is lost to the surroundings, its
temperature increases by : [AIEEE - 2011]
(   1)
Mv2K
(   1) 2
Mv K
Mv 2 (   1) 2
(A) (B) (C) K (D) Mv K
2(   1)R 2R 2R 2R
32. A container with insulating walls is divided into equal parts by a partition fitted with a valve. One part is
filled with an ideal gas at a pressure P and temperature T, whereas the other part is completely evacuated.
If the valve is suddenly opened, the pressure and temperature of the gas will be : [AIEEE 2011]
P T T P
(A) , (B) P, T (C) P, (D) ,T
2 2 2 2
33. Helium gas goes through a cycle ABCDA (consisting of two isochoric and isobaric lines) as shown in
figure. Efficiency of this cycle is nearly : (Assume the gas to be close to ideal gas)
[AIEEE 2012]

(A) 15.4% (B) 9.1% (C) 10.5% (D) 12.5%


34. A Carnot engine, whose efficiency is 40%, takes in heat from a source maintained at a temperature of
500K. It is desired to have an engine of efficiency 60%. Then, the intake temperature for the same exhaust
(sink) temperature must be : [AIEEE 2012]
(A) efficiency of carnot engine cannot be made larger than 50%
(B) 1200 K
Page # 72
HEAT & THERMODYNAMICS PHYSICS PART - IV

(C) 750 K (D) 600 K


35. One mole of diatomic ideal gas undergoes a cyclic process ABC as shown in figure. The process BC is
adiabatic. The temperatures at A, B and C are 400 K, 800 K and 600 K respectively. Choose the correct
statements : [JEE MAINS 2014]

(a) The change in internal energy in the process AB is – 350 R.


(b) The change in internal energy in the process BC is – 500 R.
(c) The Change in internal energy in whole cyclic process is 250 R.
(d) The change in internal energy in process CA is 700 R.
36. Consider a spherical shell of radius R at temperature T. The black body radiation inside it can be consid-
ered as an ideal gas of photons with internal energy per unit volume u = [U/V)  T4 and pressure p = [(1/
3)(U/V)]. If the shell now undergoes an adiabatic expansion the relation between T and R is :
[JEE MAINS 2015]
(a) T  e–3R (b) T  (1/R) (c) T  (1/R3) (d) T  eR
37. Consider an ideal gas confined in an isolated closed chamber. As the gas undergoes an adiabatic expansion,
the average time of collision between molecules increases as Vq, where V is the volume of gas. The value of
q is : [   (Cp / Cv )] [JEE MAINS 2015]

(a) [(3  5) / 6] (b) [(  1) / 2] (c) [(  1)/ 2] (d) [(3  5) / 6]

Questions asked in previous IIT JEE/JEE ADVANCED


ONLY ONE CHOICE IS CORRECT
1. Steam at 100°C is passed into 1.1 kg of water contained in a calorimeter of water equivalent 0.02 kg at
15°C till the temperature of the calorimeter and its contents rise to 80°C. The mass of the steam condensed
in kilogram is [IIT - 1986]
(A) 0.130 (B) 0.065 (C) 0.260 (D) 0.135
2. A cylinder of radius R made of a material of thermal conductivity K1 is surrounded by a cylindrical shell
of inner radius R and outer radius 2R made of a material of thermal conductivity K2. The two ends of the
combined system are maintained at two different temperatures. There is no loss of heat across the
cylindrical surface and the system is in steady state. The effective thermal conductivity of the system is
: (IIT - 1988)
K 1K 2 K1  3K2  3K1  K2 
(A) K1 + K2 (B)  K  K  (C) (D)
1 2 4 4
3. Three closed vessels A, B and C at the same temperature T and contain gases which obey the Maxwellian
distribution of velocities. Vessel A contains only O2 , B only N 2 and C a mixture of equal quantities of

O2 and N 2 . If the average speed of the O2 molecules in vessel A is v1 , that of the N 2 molecules in vessel
essel
B is v2 , the average speed of the O2 molecules in vessel C is : (IIT - 1992)
1/2 1/2
(A)  v1  v2  (B) v1 (C)  v1v2  (D) 3kT / M

Page # 73
HEAT & THERMODYNAMICS PHYSICS PART - IV

4. Two metallic spheres S1 and S2 are made of the same material and have got identical surface finish. The
mass of S1 is thrice that of S2. Both the spheres are heated to the same high temperature and placed in the
same room having lower temperature but are thermally insulated from each other. The ratio of the
initial rate of cooling of S1 to that S2 is : (IIT - 1995)
13
1 1 3 1
(A) (B) (C) (D)  
3 3 1 3
5. The intensity of radiation emitted by the sun has its maximum value at a wavelength of 510 nm and that
emitted by the north star has the maximum value at 350 nm, If these stars behave like blackbodies, then
the ratio of the surface temperature of the sun and the north star is : (IIT - 1997)
(A) 1.46 (B) 0.69 (C) 1.21 (D) 0.83
6. The average translational kinetic energy of O2 (molar mass 32) molecules at a particular temperature is
0.048 eV. The translational kinetic energy of N 2 (molar mass 28) molecules in eV at the same temperature
e
is : (IIT - 1997)
(A) 0.0015 (B) 0.003 (C) 0.048 (D) 0.768
7. A black body is at a temperature of 2880 K. The energy of radiation emitted by this object with wavelength
between 499 nm and 500 nm is U1, between 999 nm and 1000 nm is U2 and between 1499 nm and 1500
nm is U3. The Wein constant, b = 2.88 × 106 nm-K. Then : (IIT - 1998)
(A) U1 = 0 (B) U3 = 0 (C) U1 > U2 (D) U2 > U1
8. Two idnetical containers A and B with frictionless pistons contain the same ideal gas at the same
temperature and the same volume V. The mass of the gas in A is mA and that in B is mB . The gas in each
cylinder is now allowed to expand isothermally to the same final volume 2V. The changes in the pressure
in A and B are found to be P and 1.5P respectively. Then : (IIT - 1998)

(A) 4mA  9mB (B) 2m A  3mB (C) 3m A  2mB (D) 9mA  4mB
9. An ideal gas is initially at temperature T and volume V. It volume is increased by V due to an increase
in temperature T , pressure remaining constant. The quantity  = V / V T varies with temperature ure
as : (IIT - 2000)

(A) (B) (C) (D)

10. A block of ice at –10°C is slowly heated and converted to steam at 100°C. which of the following curves
represents the phenomenon qualitatively ? [IIT - 2000]

T T T T
(A) (B) (C) (D)
Heat supplied Heat supplied Heat supplied Heat supplied

11. P-V plots for two gases during adiabatic processes are shown in the figure. Plots 1 and 2 should correspond
respectively to : (IIT - 2001)

Page # 74
HEAT & THERMODYNAMICS PHYSICS PART - IV

(A) He and O2 (B) O2 and He (C) He and Ar (D) O2 and N 2


dV dP
12. Which of the following graphs correctly represent the variation of β = – with P for an ideal gas
as
V
at constant temperature? (IIT - 2002)

(A) (B)

(C) (D)

13. 2 kg of ice at –20°C is mixed with 5 kg of water at 20°C in an insulating vessel having a negligible heat
capacity. Calculate the final mass of water remaining in the container. It is given that the specific heats of
water and ice are 1 kcal/ kg°C and 0.5 kcal/kg°C while the latent heat of fusion of ice is 80 kcal/kg
[IIT - 2003]
(A) 7kg (B) 6kg (C) 4 kg (D) 2 kg
14. Two rods, one of aluminum and the other made of steel, having initial lengths l1 and l2 are connected
together to form a single rod of length l1 + l2. The co-efficients of linear expansion for aluminum and
steel are a and s respectively. If the length of each rod increases by the same amount when their
temperatures are raised by t°C, then find the ratio l1 / (l1 + l2) [IIT - 2003]
(A) s /  a (B)  a / s (C)  s /( s   a ) (D)  a /( a   s )
15. Three discs, A, B and C having radii 2 m, 4 m and 6 m respectively are coated with carbon black on their
outer surfaces. The wavelengths corresponding to maximum intensity are 300 nm, 400 nm and 500 nm
respectively. The power radiated by them are QA, QB and QC respectively : (IIT - 2004)
(A) QA is maximum (B) QB is maximum (C) QC is maximum (D) QA = QB = QC
16. On heating a liquid of coefficient of cubical expansion  in a container having coefficient of linear

expansion , the level of liquid w.r.t container will
3
(A) Rise (B) Fall
(C) Remain stationary (D) It is difficult to say
17. An ideal gas expands isot hermally from a volume V1 and V2 and then compressed to original volume V1
adiabatically. Initial pressure is P1 and final pressure is P3. The total work done is W. Then (IIT -2004)
(A) P3 > P1, W > 0 (B) P3 < P1, W < 0 (C) P3 > P1, W < 0 (D) P3 = P1, W = 0

Page # 75
HEAT & THERMODYNAMICS PHYSICS PART - IV

18. Variation of radiant energy emitted by sun, filament of tungsten


lamp and welding arc as a function of its wavelength is shown
in figure. Which of the following option is the correct match?
(A) Sun-T1, tungsten filament-T2, welding arc-T3
(B) Sun-T2, tungsten filament-T1, welding arc-T2
(C) Sun-T3, tungsten filament-T2, welding arc-T1
(D) Sun-T1, tungsten filament-T3, welding arc-T2 (IIT 2005)
19. In which of the following process, convection does not take place primarily ? (IIT- 2005)
(A) Sea and land breeze (B) Boiling of water
(C) Warming of glass bulb due to filament (D) heating air around a furnace

20. An ideal gas is expanding such that PT2= constant . The coefficient of volume expansion of the gas is
[IIT JEE-2008]

1 2 3 4
(A) (B) (C) (D)
T T T T
21. A real gas behaves like an ideal gas if its [IIT JEE-2010]
(A) pressure and temperature are both high (B) pressure and temperature are both low
(C) pressure is high and temperature is low (D) pressure is low and temperature is high
22. One mole of an ideal gas in initial state A undergoes a cyclic process ABCA, as shown in the figure. Its
pressure at A is P0. Choose the correct option(s) from the following [IIT JEE-2010]

(A) Internal energies at A and B are the same (B) Work done by the gas in process AB is P0 V0 n 4

P0 T0
(C) Pressure at C is (D) Temperature at C is
4 4
23. 5.6 liter of helium gas at STP is adiabatically compressed to 0.7 liter. Taking the initial temperature to be
T1, the work done in the process is : [JEE, 2011]
9 3 15 9
(A) RT1 (B) RT1 (C) RT1 (D) RT1
8 2 8 2
24. Three very large plates of same area are kept parellel and close to each other . They are considerd as ideal
black surface and have very high thermal conductivity . The first and third olates are mainted at
temperatures 2T and 3T respectively . The temperature of the middle(i.e., second)plane under steady
state condition is [IIT-JEE-2012]
1/4 1/4 1/4
 65   97   97 
(A)   T (B)   T (C)   T (D) (97)1/4 T
 2   4   2 

Page # 76
HEAT & THERMODYNAMICS PHYSICS PART - IV

25. A mixture of 2 moles of helium gas (atomic mass = 4 amu), and 1 mole of argon gas (atomic mass = 40
 v rms (helium) 
amu) is kept at 300 K in a container. The ratio of the rms speeds  v (arg on)  is : [IIT-JEE-2012]
 rms 
(A) 0.32 (B) 0.45 (C) 2.24 (D) 3.16
26. Two moles of ideal helium gas are in a rubber balloon at 30° C .The balloon is fully expandable and can be
assumed to require no energy in its expansion. The temperature of the gas in the balloon is slowly changed
to 35°C.The amount of heat required in raising the temperature is nearly (take R = 8.31 J/mol.K)
[IIT-JEE-2012]
(A) 62J (B) 104 J (C) 124 J (D) 208 J
27. Two rectangular blocks, having identical dimensions, can be arranged either in configuration I or in
configuration II as shown in the figure. One of the blocks has thermal conductivity K and the other 2K.
The temperature difference between the ends along the x-axis is the same in both the configurations. It
takes 9 s to transport a certain amount of heat from the hot end to the cold endin the configuration I. The
time to transport the same amount of heat in the configuration II is [JEE Advanced-2013]

(A) 2.0 s (B) 3.0 s (C) 4.5 s (D) 6.0 s


28. One mole of mono-atomic ideal gas is taken along two cyclic processes E  F  G  E and E  F 
H  E as shown in the PV diagram. The processes involved are purely isochoric, isobaric, isothermal or
adiabatic. [JEE Advanced-2013]

Match the paths in List I with the magnitudes of the work done in List II and select the correct answer
using the codes given below the lists.
List I List II
P. G  E 1. 160 P0V0 ln2
Q. G  H 2. 36 P0V0
R. F  H 3. 24 P0V0
S. F  G 4. 31 P0 V0
Codes:
P Q R S
(A) 4 3 2 1
(B) 4 3 1 2
(C) 3 1 2 4
(D) 1 3 2 4

Page # 77
HEAT & THERMODYNAMICS PHYSICS PART - IV

29. Two non-reactive monoatomic ideal gases hace their atomic masses in the ratio 2 : 3 . The ratio of their
partial pressure , when enclosed in a vessel kept at a constant temperature , is 4 : 3 . The ratio of their
densities is [JEE Advanced-2013]
(a) 1:4 (b) 1:2 (c) 6:9 (d) 8:9
30. Parelle rays of light of intensity I=912 Wm-2 are incident on a spherical black body kept in surroundings
of t emperature 300 K. Take Stefan Boltzmann constant   5.7  108 Wm 2K 4 and assume that the en-
ergy exchange with the surrounding is only through radiation . The final steady state temperature of the
black body is close to [JEE Advanced-2014]
(A) 33. K (B) 660 K (C) 990 K (D) 1550 K
ONE OR MORE THAN ONE CHOICE MAY BE CORRECT
1. A bimetallic strip is formed out of two identical strips one of copper and the other of brass. The co-
efficients of linear expansion of the two metals are C and  B . On heating, the temperature of the strip
goes up by T and the strip bends to form an arc of radius of curvature R. Then R is [IIT - 1999]
(A) proportional to T (B) inversely proportional to T
(C) proportional to |  B   C | (D) Inversely proportional to |  B   C |
2. Initially a black body at absolute temperature T is kept inside a closed chamber at absolute temperature
T0. Now the chamber is slightly opened to allow sun rays to enter. It is observed that temperatures T and
T0 remains constant. Which of the following statements is /are true (IIT - 2006)
(A) The rate of emission of energy from the black body remains the same
(B) The rate of emission of energy from the black body increases
(C) The rate of absorption of energy by the black body increases
(D) The energy radiated by the black body equals the energy absorbed by it
3. The figure shows the P-V plot of an ideal gas taken through a cycle ABCDA. The part ABC is a semi-circle
and CDA is half of an ellipse. Then, [IIT-JEE-2009]

(A) The process during the path A  B is isothermal


(B) Heat flows out of the gas during then path B  C  D
(C) Work done during the path A  B  C is zero o
(D) Positive work is done by the gas in the cycle ABCDA
4. CV and CP denote the molar specific heat capacities of a gas at constant volume and constant pressure,
respectively, Then [IIT-JEE-2009]
(A) CP – CV is larger for a diatomic ideal gas then for a monoatomic ideal gas
(B) CP + CV is larger for a diatomic ideal gas then for a monoatomic ideal gas
(C) CP / CV is larger for a diatomic ideal gas then for a monoatomic ideal gas
(D) CP . CV is larger for a diatomic ideal gas then for a monoatomic ideal gas
5. For an ideal gas :
(A) the changes in internal energy in a constant pressure process from temperature T1 and T2 is
equal to nCv  T2  T1  , where Cv is the molar heat capacity at constant volume and n the
number of moles of the gas.
(B) the change in internal energy of the gas and the work done by the gas are equal in magnitude
in an adiabatic process
(C) the internal energy does not change in an isothermal process
(D) no heat is added or removed in an adiabatic process (IIT - 1989)

Page # 78
HEAT & THERMODYNAMICS PHYSICS PART - IV

6. Two bodies A and B have thermal emissivities of 0.01 and 0.81 respectively. The outer surface areas of
the two bodies are the same. The two bodies emit total radiant power at the same rate. The wavelength
B corresponding to maximum spectral radiancy in the radiation from B shifted from the wavelength
corresponding to maximum spectral radiancy in the radiation from A, by 1.00  m . If the temperaturee
of A is 5802 K : (IIT - 1994)
(A) the temperature of B is 1934 K (B) B  1.5 m
(C) the temperature of B is 11604 K (D) the temperature of B is 2901 K

7. Let v , vrms and v p respectively denote the mean speed, root mean square speed and most probablee
speed of the molecules in an ideal monoatomic gas at absolute temperature T. The mass of molecule is m.
Then : (IIT - 1998)
(A) no molecule can have speed greater than 2v rms

(B) no molecule can have speed less than v p / 2

(C) v p  v  vrms

3 2
(D) the average kinetic a energy of molecule is mvp
4
8. Cv and Cp denote the molar specific heat capacities of a gas at constant volume and constant pressure,
respectively. Then [IIT JEE-2009]
(A) Cp –Cv is larger for a diatomic ideal gas than for a monoatomic ideal gas.
(B) Cp + Cv is larger for a diatomic ideal gas than for a monoatomic ideal gas.
(C) Cp/Cv is larger for a diatomic ideal gas than for a monoatomic ideal gas.
(D) Cp. Cv is larger for a diatomic ideal gas than for a monoatomic ideal gas.
9. Heater of an electric kettle is made of a wire of length L and diameter d. It takes 4 minutes to raise the
temperature of 0.5 kg water by 40 K. This heater is replaced by a new heater having two wires of the
same material, each of length L and diameter 2d. The way these wires are connected is given in the
options. How much time in minutes will it take to arise the temperature of the same amount of water by
40K ? [JEE ADVANCED-2014]
(a) 4 if wires are in parallel (b) 2 if wires are in series
(c) 1 if wires are in series (d) 0.5 if wires are in parallel
10. A container of fixed volume has a mixture of one mole of hydrogen and one mole of helium in equilibrium
at temperature T, Assuming the gases are ideal the correct statement(s) is(are) : [JEE ADV. 2015]
(a) The verage energy per mole of the gas mixture is 2RT
(b) The ratio of speed of sound in the gas mixture to that in helium gas is (6 /5)
(c) The ratio of the rms speed of helium atoms to that of hydrogen molecules is 1/2
(d) The ratio of the rms speed of helium atoms to that of hydrogen molecules is 1/ 2
11. An ideal monoatomic is gas is confines in a horizontal cylinder by a spring loaded piston (as shown in the
figure).Initially the gas us at tenperature T1, pressure P1 and volume V1 and the spring is in its relaxed
state . The gas is then heated very slowly to temperature T2 , pressure P2 by a distance x. Ignoging the
friction between the piston and the cylinder , the correct statement (s) is (are ): [JEE ADV. 2015]

Page # 79
HEAT & THERMODYNAMICS PHYSICS PART - IV

1
(a) If V2=2V1 and T2=3T, then the energy stored in the spring is PV.
4 1 1
(b) If V2=2V1 and T2= 3T1 , then the change in internal energy is 3P1V1.

7
(c) If V2=3V1 and T2=4T1, then the work done by the gas is PV
3 1 1.

17
(d) If V2=3V1 and T2=4T1 then the heat supplies to the gas is PV.
6 1 1
12. A human body has a surface area of approximately 1 m2. The normal body temperature is 10 K
above the surrounding room temperature T0. Take the room temperature to be T0 = 300 K. For T0 = 300
K, the value of T04  460 Wm 2 (where  is the Stefan-Boltzmann constant). Which of the following
options is/are correct ? [JEE ADV 2017]
(a) If the body temperature rises significantly then the peak in the spectrum of electromagnetic radiation
emitted by the body would shift to longer wavelengths
(b) If the surrounding temperature reduces by a small amount ? T0 <<T0, then to maintain the same
body temperature the same (living) human being needs to radiate W  4 T03 T0 more energy per unit
time
(c) The amount of energy radiated by the body in 1 second is close to 60 Joules
(d) Reducing the exposed surface area of the body (e.g. by curling up) allows humans to maintain the
same body temperature while reducing the energy lost by radiation

PASSAGE BASED QUESTIONS


PASSAGE :
A fixed thermally conducting cylinder has a radius R and height L0. The cylinder is open at its bottom and
has a small hole at its top. A piston of mass M is held at a distance L from the top surface, as shown in the
figure. The atmospheric pressure is P0

1. The piston is now pulled out slowly and held at a distance 2L from the top. The pressure in the cylinder
between its top and the piston will then be: (2007; 4M)

P0 P0 Mg P0 Mg
(a) P0 (b) (c)  (d) 
2 2 R 2 2 R 2
2. While the piston is at a distance 2L from the top, the hole at the top is sealed. The piston is then released,
to a position where it can stay in equilibrium. In this condition, the distance of the piston from the top is
: (2007; 4M)

Page # 80
HEAT & THERMODYNAMICS PHYSICS PART - IV

 2P0 R 2   P0 R 2  Mg 
(A)  2   2L  (B)  2   2L 
 R P0  Mg   R P0 

 P0 R 2  Mg   P0R 2 
(C)  2   2L  (D)  2   2L 
 R P0   R P0  Mg 
3. The piston is taken completely out of the cylinder. The hole at the top is sealed. A water tank is brought
below the cylinder and put in a position so that the water surface in the tank is at the same level as the top
of the cylinder as shown in the figure. The density of the water is  . In equilibrium, the height H of the
water column in the cylinder satisfies : (2007; 4M)

2 2
(A)  g  L0  H   P0  L 0  H   L0P0  0 (B)  g  L0  H   P0  L0  H   L 0P0  0

2 2
(C)  g  L0  H   P0  L 0  H   L 0P0  0 (D)  g  L0  H   P0  L0  H   L 0P0  0

PASSAGE:

 5
A small spherical monoatomic ideal gas bubble     is trapped inside a liquid of density  (see
 3
figure). Assume that the bubble does not exchange any heat with the liquid. The bubble contains n moles
of gas. The temperature of the gas when the bubble is at the battom is T0 the height of the liquid is H and

the atmospheric pressure is P0 (Neglect surface tension). (IIT - 2008)

0
Liquid

4. As the bubble moves upwards, besides the buoyancy force the following forces are acting on it.
(A) Only the force of gravity
(B) The force due to gravity and the force due to the pressure of the liquid.
(C) The force due to gravity, the force due to the pressure of the liquid and the force due to viscosity
of the liquid.
(D) The force due to gravity and the force due to viscosity of the liquid.

Page # 81
HEAT & THERMODYNAMICS PHYSICS PART - IV

5. When the gas bubble is at a height y from the bottom, its temperature is
2 2
 P  l g H  3  P  l g H  y   5
(A) T0  0  (C) T0  0 
 P0   l g y   P0   l g H 

3 3
 P  l g H  5  P  l g H  y  5
(B) T0  0  (D) T0  0 
 P0   l g H   P0   l g H 
6. The buoyancy force acting onthe gas bubble is (Assume R is the universal gas constant)

2
 P0   l g H   l nRgT0
 l n RgT0 5 2 3
(A) 7 (B)
 0   l g y  5  P0   l g H  5  P0   l g  H  y   5

3
 P0   l g H   l n RgT0
5
(C)  l nRgT0 8 (D) 3 2

 P0   l g y  5  P0   l g H 5  P0   l g  H  y  5

Passage (7 to 8)
In the figure a container is shown to have a movable (without friction) piston on top. The container and
the piston are all made of perfectly insulating material allowing no heat transfer between outside and
inside the container. The container is divided into two compartments by a rigid partition made of a
thermally conducting material that allows slow transfer of heat. The lower compartment of the container
is filled with 2 moles of an ideal monatomic gas at 700 K and the upper compartment is filled with 2 moles
of an ideal diatomic gas at 400 K. The heat capacities per mole of an ideal monatomic gas
3 5 5 7
are CV  R,Cp  R , and those for an ideal diatomic gas are CV  R,Cp  R
2 2 2 2
[JEE ADVANCED - 2014]

7. Consider the partition to be rigidly fixed so that it does not move. When equilibrium is achieved, the final
temperature of the gases will be
(A) 550 K (B) 525 K (C) 513K (D) 490 K

Page # 82
HEAT & THERMODYNAMICS PHYSICS PART - IV

8. Now consider the partition to be free to move without friction so that the pressure of gases in both
compartments is the same. Then total work done by the gases till the time they achieve equilibrium will
be
(A) 250 R (B) 200 R (C) 100 R (D) –100 R

Passage (9 to 11) [JEE ADVANCED 2017]


An ideal gas is undergoing a cyclic thermodynamic process in different ways as shown in the
corresponding - diagrams in column 3 of the table. Consider only the path from state 1 to state 2.
denotes the corresponding work done on the system. The equations and plots in the table have standard
notations as used in thermodynamic processes. Here  is the ratio of heat capacities at constant pressure
and constant volume. The number of moles in the gas is .
Column 1 Column 2 Column 3

1
(I) W12  ( P2V2  P1V1 ) (i) isothermal (P)
 1

(II) W12   PV2  PV1 (ii) Isochoric (Q)

(III) W1 2  0 (iii) Isobaric (R)

V 
(IV) W1 2  nRT ln  2  (iv) Adiabatic (S)
 V1 

9. Which one of the following options correctly represents a thermodynamic process that is used as
a correction in the determination of the speed of sound in an ideal gas ?
(A) (IV) (ii) (R) (B) (I) (ii) (Q) (C) (I) (iv) (Q) (D) (III) (iv) (R)

Page # 83
HEAT & THERMODYNAMICS PHYSICS PART - IV

10. Which of the following options is the only correct representation of a process in which U = Q – PV?
(A) (II) (iii) (S) (B) (II) (iii) (P) (C) (III) (iii) (P) (D) (II) (iv) (R)
11. Which one of the following options is the correct combination ?
(A) (II) (iv) (P) (B) (III) (ii) (S) (C) (II) (iv) (R) (D) (IV) (ii) (S)

MATCH THE COLUMN TYPE QUESTIONS


1. Column I gives some devices and Column II gives some processes on which the functioning of these
devices depend. Match the devices in Column I with the processes in Column II. (IIT 2007)
Column I Column II
A. Bimetallic strip p. Radiation from a hot body
B. Steam engine q. Energy conversion
C. Incandescent lamp r. Melting
D. Electric fuse s. Thermal expansion of solids
2. Column I contains a list of processes involving expansion of an ideal gas. Match this with Column II
describing the thermodynamic change during this process. (IIIT 2008)

I
II
Vacuum
Ideal gas

Column I Column II
A. An insultated container has two p) The temperature of the gas decreases
chambers separated by a valve
Chamber I contains an ideal gas
and the chamber II has vacuum.

B. An ideal monoatomic gas expands (q) The temperature of the gas increases to twice
original volume such that its or remains constant

1
its pressure P , where V is the
V2
volume of the gas
C) An ideal moniatomic gas expands to (r) The gas loses heat
expands to twice its original volume

1
such that its pressure P
V 4/3
Where V is its volume
D) An ideal monoatomic gas expands such (s) The gas gains heat
that its pressure P and volume V follows
the behaviour shown in the graph

Page # 84
HEAT & THERMODYNAMICS PHYSICS PART - IV

3. The figure shows a cylic process ABCDA.

Column I (Process) Column II


A. AB (p) W < 0
B. BC (q) Q > 0
C. CD (r) W > 0

Page # 85
HEAT & THERMODYNAMICS PHYSICS PART - IV

ANSWER KEY

TRY YOURS SELF


1. D 2. C 3. A,B,C,D 4. B,C,D 5. A,B,C 6. A
7. C 8. C 9. C 10. A 11. C 12. B
13. A 14. C 15. C 16. A 17. D 18. A
19. B 20 B 21. A 22. D 23. C 24. A
25. B 26. C 27. B 28. C 29. B 30. C
31. C 32. A 33. A 34. A 35. A 36. A
37. D 38. B 39. A 40. C 41. B 42. D
43. B 44. B 45. B 46. A 47. C 48. C
49. A 50. B 51. A 52. D 53. B,C 54.
55. C 56. D 57. C 58. A 59. C 60. A
61. D 62. C 63. A 64. CD 65. CD 66. C
67. C 68. D 69. B 70. B 71. B 72. A,B,D
73. A,B,C,D 74. B 75. D 76. A 77. A 78. D

ONLY ONE CHOICE IS CORRECT

1. (C) 2. (B) 3. (B) 4. (C) 5. (B) 6. (A)


7. (C) 8. (D) 9. (B) 10. (A) 11. (A) 12. (B)
13. (C) 14. (D) 15. (C) 16. (A) 17. (A) 18. (B)
19. (B) 20. (C) 21. (B) 22. (C) 23. (B) 24. (C)
25. (B) 26. (C) 27. (A) 28. (C) 29. (C) 30. (B)
31. (B) 32. (B) 33. (B) 34. (A) 35. ((B) 36. (B)
37. (D) 38. (B) 39. (C) 40. (A) 41. (C) 42. (C)
43. (B) 44. (B) 45. (A) 46. (A) 47. (B) 48. (B)
49. (A) 50. (C) 51. (C) 52. (C) 53. (C) 54. (C)
55. (B) 56. (A) 57. (C) 58. (C)

ONE OR MORE THAN ONE CHOICE MAY BE CORRECT


1. (A, C, D) 2. (A, B) 3. (A, B, D) 4. (A, B, C, D) 5. (A, B, D) 6. (C, D)
7. (A, C) 8. (C, D) 9. (A, C, D) 10. (B, C) 11. (A, D) 12. (C, D)
13. (B, D) 14. (A, B, C) 15. (A, B, C) 16. (A, B, C) 17. (B, D) 18. (B, C)
19. (B, C) 20. (C, D) 21. (A, B, D) 22. (A, B, D) 23. (A, B) 24. (A, C)
25. (A, C) 26. (A, D) 27. (B, C, D) 28. (A, B)

PASSAGE BASED QUESTIONS


1. (B) 2. (D) 3. (C) 4. (B) 5. (C) 6. (B)
7. (A) 8. (D) 9. (D) 10. (C) 11. (A) 12. (B)

Page # 86
HEAT & THERMODYNAMICS PHYSICS PART - IV

MATCH THE COLUMN TYPE QUESTIONS

1. (A)  (s) ; (B)  (q,r) ; (C)  (q) ; (D)  (p,s)


2. (i) - (D), (ii) – (C), (iii) – (B), (iv) – (A)
3. (i) - (a), (ii) – (c), (iii) - (a), (iv) – (d)
4. (i) - (d), (ii) – (c), (iii) - (b) , (iv) – (d)

INTEGER TYPE QUESTIONS


1. 6 2. 2 3. 3 4. 1 5. 7 6. 9
7. 6 8. 5 9. 2

EXERCISE 3
Questions asked in previous AIEEE / JEE MAINS

PART - I: HEAT TRANSFER


1. (B) 2. (A) 3. (A) 4. (D) 5. (D) 6. (C)
7. (C) 8. (C) 9. (A) 10. (A) 11. (B) 12. (A)
13. (B) 14. (D)

PART -II: KTG & THERMODYNAMICS


1. (A) 2. (C) 3. (D) 4. (C) 5. (C) 6. (A)
7. (D) 8. (C) 9. (B) 10. (A) 11. (A) 12. (C)
13. (A)(D) 14. (B) 15. (A) 16. (C) 7. (C) 18. (A)
19. (B) 20. (A) 21. (A) 22. (A) 23. (C) 24. (B)
25. (B) 26. (A) 27. (B) 28. (B) 29. (A) 30. (B)
31. (D) 32. (D) 33. (A) 34. (C) 35. (B) 36. (C)
37. (C)

Questions asked in previous IIT JEE/JEE ADVANCED

1. (b) 2. (a) 3. (d) 4. (d) 5. (b) 6. (c)


7. (c) 8. (c) 9. (a) 10. (c) 11. (c) 12. (b)
13. (a) 14. (c) 15. (d) 16. (b) 17. (c) 18. (c)
19. (b) 20. (c) 21. (d) 22. (AB) 23. (a) 24. (c)
25. (d) 26. (d) 27. (a) 28. A 29. (d) 30. (a)

ONE OR MORE THAN ONE CHOICE MAY BE CORRECT


1. (B,D) 2. (A, D) 3. (B, D) 4. (B). (D) 5. (A, B, C, D) 6. (A, B)
7. (C, D) 8. (B, D) 9. (B, D) 10. ABD 11. ABC 12. BCD

Page # 87
HEAT & THERMODYNAMICS PHYSICS PART - IV

PASSAGE BASED QUESTIONS

1. (A) 2. (D) 3. (C) 4. (D) 5. (B) 6. (B)


7. (D) 8. (D) 9. (C) 10. (B) 11. (B)

MATCH THE COLUMN TYPE QUESTIONS


1. (A-s, B-q, C-p, q, D-q, r)
2. A  q,  b   p, q  c  p,s  d   q,s
3. (A-s, B-q, r, C-q, D-p, s)

Page # 88
SIMPLE HARMONIC MOTION
2
 Theory

 Try YourSelf
CONTENTS
 Solved Examples

 EXERCISES

 Single Correct Answer

 More than One Correct Questions

 Passage Questions

 Matrix Match Type Questions.

 Integer Type Questions

 Previous Years Questions

 Answer Key
SIMPLE HARMONIC MOTION PHYSICS PART - IV

INTRODUCTION

PERIODIC MOTION:
Periodic motion of a body is that motion which repeats itself after a fixed interval of time.The fixed
interval of time is called time period of periodic motion.
Examples
(i) The revolution of earth around the sun is an example of periodic motion. Its period of revolution is one
year.
(ii) The motion of hands of a clock is a periodic motion. The period of motion of hour’s hand of a clock is
12 hours, of minute’s hand of a clock is 1 hour and of second’s hand of a clock is one minute.
(iii) Uniform circular motion is a periodic motion.

OSCILLATORY MOTION:
Oscillatory or Vibratory motion is that motion in which a body moves to and fro or back and forth
repeatedly about a fixed point (called mean position), in a definite interval of time. In such motion, the
body is confined within well defined limits (called extreme positions) on either side of mean position.
Thus a periodic and bounded motion of a body about a fixed point is called oscillatory or vibratory
motion.
(i) The motion of the bob of a simple pendulum when it is displaced once from its mean position and left
to itself, is oscillatory motion.
(ii) The motion of liquid contained in U-tube when it is compressed once in one limb and left to itself, is
oscillatory motion.
(iii) Equation for Oscillatory Motion is F = -kxn where n is odd.

SIMPLE HARMONIC MOTION:


Definition : most important of all oscillatory motion is SHM, in this motion the particle oscillates
periodically in such a way that the linear / angular acceleration of the particle is directly proportional to
its linear/angular displacement from a fixed point (mean position) and is always directed towards that
fixed point.
EQUATION OF SHM : The necessary and sufficient condition for a motion to be simple harmonic is that
the net restoring force (or torque) must be linear i.e.,
F  ma  kx (Where k is a constant)

d2 x k d2 x k k
 a 2
  x or 2
 2 x  0 where 2  or  
dt m dt m m
where x is the instantaneous displacement..
K dx
Solution of equation of SHM : We know that a   x . Multiplying both sides byy and integrating
m dt
with respect to t ,
 d  dx   dx  k x  dx
 dt  dt   dt   m  dt
    

dv k .x.dx  dx 
  v. dt   m dt  v
dt 

v 2 k x 2
 . c
2 m 2

Page # 90
SIMPLE HARMONIC MOTION PHYSICS PART - IV

2
1  dx  k x2
We get     C …(i)
2  dt  2m

dx
Where C is a constant of integration. Now when x is maximum will be zero. The maximum
dt
displacement X max of the particle from the mean position is called amplitude and is represented by A ,
k A2
then the value of comes out to be C
m 2
2
1  dx  k
Here  (A 2  x 2 )
2  dt  2m

k dx
Putting  2 we get   A2  x2 … (ii)
m dt
This equation gives the velocity of the particle in S.H.M.
dx
Again  dt … (iii)
A2  x 2

x
Integrating this equation with respect to . we get sin 1  t  
A
Where is another constant of integration which depends on initial conditions.
Thus x  Asin(t  ) … (iv)

Here  is called ANGULAR FREQUENCY and  is called INITIAL PHASE OR PHASE CONSTANT OR
EPOCH , whose value depends upon initial conditions. If the time is recorded from the instant when x is
zero and is increasing then according to equation (iv),  must be equal to zero..

Now, the periodic time or time required to complete one vibration or the time to increase the phase angle
2
by 2 is T = is called the time period of oscillation.

The frequency , the number of complete vibration per second is given by

1  1 1 k
f    … (vi)
T 2 2 2 m
If we put in place of then
x  Acos(t  ) … (vii)

Thus S.H.M. may also be represented by a cosine function but initial phase is different in two cases.
Parametersof SHM :
Displacement :It is the position of the particle from mean position at a given instant, it is given by
x  A sin t   
Amplitude :It is the magnitude of maximum value of displacement,denoted by A. It is also known as
‘displacement amplitude’
Time period : It is the smallest interval of time after which the motion repeats. It is represented by T .
Let us replace t by t  T in equation (vii).
 t T   2t 
'x' at (t  T)  Acos  2  0   Acos   2  0 
 T   T 
It is clear from here that the motions repeat after time T.

Page # 91
SIMPLE HARMONIC MOTION PHYSICS PART - IV

Time period may also be defined as the time taken by the oscillating particle to complete one oscillation
Frequency :It is the number of oscillations made in one second.it is denoted by f,it is reciprocal of time
period. When frequency is multiplied by 2 then the quantity obtained is called angular frequency  .

2
Therefore f=1/T = And  =2  f = 2  /T

Phase : Phase of a vibrating particle at any instant is the state of the vibrating particle regarding its
displacement and direction of vibration at that particular instant. The argument of the cosine in equation
x  A cos(t  0 ) gives the phase of oscillation at time t . It is denoted by
y .
t
   2  0 or   t  0
T
It is clear that phase  is a function of time . The phase of a vibrating particle can be expressed in terms of
fraction of the time period that has elapsed since the vibrating particle left its mean position in the positive
direction.
2 2t
Again,   0  t  t So, the phase change in time is .
T T
The phase change in second will be which actually means ‘no change in phase’. Thus, time period may also
be defined as the time interval during which the phase of the vibrating particle changes by 2 .
Phase constant :It is the phase of the oscillating particle at t =0,it is also called initial phase or epoch.in
equation (vii) it is represented by 0 . For example if a particle start its oscillation from mean position then

0 =0, if a particle start its oscillation from extreme position then 0 = .
2

RELATION BETWEEN SHM AND UNIFORM CIRCULAR MOTION:


When the particle executes uniform circular motion on the circle then the foot of the perpendicular from
the particle on the diameter executes simple harmonic motion about O .
Let at t=0 the particle is on X axis and at some other instant t it is at P,
in this time t radius vector has been turned by angle   t
In figure, the magnitude of the displacement of N , from the mean position, at any instant is given by
y

Inright triangle ONP


x t
…(iv) cos  
x  A cos  A
T

Y Y
P B
N’ At t=t
 A A A
y y At t=0
t
X  X X’ 0 X
O N O N
x x

Y Y

where A is the radius of the reference circle and  is the angle covered by the reference particle in time t .
If  be the uniform angular velocity of the reference particle, then
x  Acos t …(v)
If the particle is not on X axis but at A on t=0,and at other instant t it is at B, then the displacement of the
projection from centre is given by
y  Rsin(t  0 ) …(i)

Page # 92
SIMPLE HARMONIC MOTION PHYSICS PART - IV

The maximum displacement of the projection from the centre is called amplitude. If amplitude is denoted
by A then R = A.
 y  Asin(t  0 ) …(ii)
and x  Acos(t  0 ) …(iii)
Expression for displacement
If the projection N  of the reference particle is taken on the diameter YOY  , then
y  Asin  or y  Asin t .
Note:
In general, x or y could be a linear displacement, an electrical voltage etc. So, equation (ii) or (iii) can be
used in a more general sense.

Illustration : If two SHMs are represented by equations y 1  10sin[3t  (  / 4)] and


y 2  5[sin(3t )  3 cos(3t )] , find the ratio of their amplitudes.

Solution : As y 2  5sin 3t   5 3cos 3t 

So if 5  A cos  and5 3  A sin 


2
i.e., if A  52  5 3   10 and tan   5 3 / 5  3

i.e.,    / 3 the above equation becomes

y 2  Acos sin 3t   Asin  cos3t  A sin 3t  

i.e., y 2  10sin 3t    /3 

A1 10
So  ,
A2 10 i.e., A 1 : A 2  1:1 .

VELOCITY AND ACCELERATION IN SHM:


Velocity in SHM:Differentiating y  Asin t w.r.t. time t , we get
d d
(y)  (A sin t)
dt dt

or velocity, v  A cos t or v  A cos     t 

A2  y2
or v  A
A

or v   A2  y 2

At the mean position, y  0 .  v  A (maximum value)


At the extreme position, y  A

 v   A2  A2 or v = 0. Y
A2  y 2

y A
X’ 
X
O N
x

Y

Page # 93
SIMPLE HARMONIC MOTION PHYSICS PART - IV

Conclusion. A particle in SHM has maximum velocity at mean position and zero velocity at the extreme position.
5.2.Acceleration in SHM:
d d
Differentiating v  A cos t w.r.t. time t , we gett (v)  (Acos t)
dt dt

d
or acceleration  A (cos t)   A2 sin t  2 (A sin t)
dt

or acceleration  2 y
At the mean position, y  0  acceleration = zero
At the extreme position, y  A  acceleration  2 A (maximum value)

Conclusion. A particle in simple harmonic motion has zero acceleration at the mean position and maximum
acceleration at the extreme position.
An Alternative Method for Finding Velocity and Acceleration in SHM:
Let V be the velocity of the reference particle at P . Resolve velocity into two rectangular componentss
parallel to and perpendicular to . The velocity of the projection is clearly . V cos  .
v  Vcos   A cos t Y V  Vcos
Vsin P
N  A
y2 y
or v  A 1  sin2 t or v  A 1  or v   A2  y 2
A2 
X X
O

Y
V2
The centripetal acceleration of the particle at P can be resolved into two rectangular componentss
A
V2 V2 V2
 cos  perpendicular to YOY and sin  anti-parallel to cos 
o YOY . A Y V
A A
P V2
2 2 N  sin 
V V A
Acceleration of N   sin  or Acceleration   2 (A sin )  2 (Asin t) . y A
A A 
X X
O
or Acceleration  2 y .

Y
Illustration :
A particle executing SHM of period  sec and amplitude 2 cm.find the acceleration of it when it is (i) at the
maximum displacement from the mean position and (ii) at 1 cm from the mean position.
Solution :   acceleration at a distance x is given by a   2 x
2 2
 2  4 2
(i) when x=2 cm a    A  2  2  8cm/sec
T 
2 2
 2  4
(ii)when x=1cm a    x  2  1  4cm / sec2
 T  
ENERGY OF A BODY IN S.H.M.:
The total energy of a harmonic oscillator consists of two parts, potential energy (P.E.) and Kinetic Energy
(K.E.).

Page # 94
SIMPLE HARMONIC MOTION PHYSICS PART - IV

Kinetic energy:It is the energy possessed by the oscillator due to it‘s motion(velocity)
Since velocity of a simple harmonic oscillator is given by;
v  Acos(t  )

2 12 2 2 1 2 2 12
therefore kinetic energy at time t will be K  mv  mA  cos (t   )  m (A  x )
2 2 2
Potential energy:It is the energy possessed by the simple harmonic oscillator due to its displacement
from the mean position . .
Simple harmonic motion is defined by the equation
F  kx .
The work done by the force F during a displacement from x to
o x  dx is
dW  Fdx

 kxdx .
The work done in a displacement from x  0 to is
x 1
W   ( kx)dx   kx2 .
0 2
Let U ( x) be the potential energy of the system when the displacement is x . As the change in potential
energy corresponding to a force is negative of the work done by this force,
1 1
U(x)  U(0)  W  kx2 .  U(x)  U(0)  kx2
2 2
Let us choose the potential energy to be zero when the particle is at the centre of oscillation x  0 .

2 1
Then U(0)  0 and U(x)  kx .
2
This expression for potential energy is same as that for a spring and has been used so far in this chapter.

k
As  , k  m2
m

1
we can write U(x)  m2 x 2 …(i)
2
The displacement and the velocity of a particle executing a simple harmonic motion are given by executing
a simple harmonic motion are given by
x  Asin(t  ) and v  Acos(t  )
therefore potential energy at time t is
1 1
U  m2 x 2  m2 A2 sin2 (t   )
2 2
Total energy: The total mechanical energy at time t is E U  K
1
 m2 A 2[sin2 (t  )  cos2 (t  )] + U (0)
2

1
 m2 A2 .+ U(0) ….(ii)
2
We see that the total mechanical energy at time is independent of . Thus, the mechanical energy remains
constant as expected.
Note: E  U max  K max if U(0) =0 and : E  U max  K max if U(0)  0

Page # 95
SIMPLE HARMONIC MOTION PHYSICS PART - IV

K, U or E 1 K, U or E
E  m2 A 2 1
E m2 A2
2 2
1
E
PE  m2 x 2 1
2 KE  m2 A 2 cos 2 t
E 2
1 2 1
PE  m2 (A 2  x 2 ) PE  m2 A 2 sin 2 t
2 2
-A A A +A x v 1 Time
  T T
2 2 2

Average value of potential energy and kinetic energy:


1 1
U  m2 x 2  m2 A 2 sin2 (t  ) {since at time t, x = Asin(t  ) }
2 2
The average value of P.E. of complete vibration is given by
T T T
1 1 1 m2 A 2
U average   Udt   m2 A 2 sin2 ( t  )  2sin2 ( t  )dt  1 m2 A2
T0 T 02 4T 0 4
because the average value of sine or of cosine function for the complete cycle is equal to zero.
Now K.E. at x is given by
2 2
1  dx  1  d  1
K.E.  m    m  {Asin(t  )}  m2 A 2 cos2 (t  )
2  dt  2  dt  2
The average value of K.E. for complete cycle
T
1 1
K.E.average  m2 A 2 cos2 ( t  )dt
T 0 2
T
m2 A 2 2 2
  {1  cos2(t  )}dt  m A .T  1 m2 A2
4T 0 4T 4
Thus average values of K.E. and P.E. of energy of a harmonic oscillator are equal to half of the total energy.
Illustration :
A solid cylinder is attached to a horizontal massless spring so that it can roll without slipping along a
horizontal surface. If the cylinder is slightly displaced and released, find the time period of the SHM it
executes.

1 1 1
Solution : From law of conservation of M.E E  mv 2  I2  kx2
2 2 2
v  r as body rolls without slipping
TYPES OF SHM
Linear SHM : It is the that SHM in which the oscillator moves on a straight line and the linear acceleration
of the particle is directly proportional to its linear displacement from a fixed point (mean position) and is
always directed towards that fixed point.
Examples : oscillations of spring block system,oscillation of an object floating in a liquid etc.
Angular SHM : It is the that SHM in which the oscillator moves on an arc and the angular acceleration of
the particle is directly proportional to its angular displacement from a fixed point (mean position) and is
always directed towards that fixed point.
Examples : oscillations of simple pendulam,oscillation of a sphere in a spherical bowl etc.

Page # 96
SIMPLE HARMONIC MOTION PHYSICS PART - IV

METHODS TO SOLVE SYSTEMS EXECUTING SHM


Restoring Force or Torque Method.The following steps are usually followed in this method.
Step-1 : Find the stable equilibrium position which usually is also known as the mean position. Net force
or torque on the particle in this position is zero and Potential energy is minimum.
Step-2 : Displace the particle from its mean postion by a small displacement x (in case of a linear SHM) or
 (in case of an angular SHM).
Step-3 : Find net force or torque in this displaced position.
Step-4 : Show that this force or torque has a tendency to bring the particle back to its mean postion and
magnitude of force or torque is proportional or displacement, i.e.,
F  – x or F = - kx … (i)
   or    k  … (ii)
or this force or torque is also known as restroing force or restoring torque.
Step-5 : Find linear acceleration by dividing Eq.(i) by mass m or angular accenleration by dividing
Eq.(ii) by moment of inertia I. Hence,
k k
a .x  2 x,or      2
m I
Step-6 :

a  2 a 
Finally,  or Or  or
x  T x 

x 
 T  2 or 2
a 
Energy Method : Repeat step 1 and step 2 as in method 1. find the total mechnical energy (E) in the
displaced    k  position. Since, mechanical energy in SHM remains constant..

dE
0
dt
By diffenrentiating the energy equation wiht respect to time and substituting
dx d dv d
 v,  ,  a, and   we come to step 5. The remaining procedure is same.
dt dt dt dt
Note : (i) E usually consists of following terms :
(a) Gravitational P.E. (b) Elastic P.E. (c) Electrostatic P.E. (d) Rotational K.E.and
(e) Translational K.E.
(ii) For gravitational P.E., choose the reference point (h=0) at mean position.

TRY YOURSELF
1. The graph plotted between the velocity and displacement from mean position of a particle executing S.H.M.
is :
(A) circle (B) ellipse (C) parabola (D) straight line.
2. If a body of mass 36 gm moves with SHM of amplitude A = 13 cm and period T = 12 sec. At a time t = 0, the
displacement x is + 13 cm, the force acting on the body at t = 2 sec is :
(A) 36 dyne (B) 32 dyne (C) 65 dyne (D) 128 dyne
3. The displacement of a particle executing SHM is given by x  0.01 sin 100 (t  0.05) . The time period is:
(A) 0.01 sec (B) 0.02 sec (C) 0.1 sec (D) 0.2 sec.
4. A particle is vibrating in simple harmonic motion with an amplitude of 4 cm. At what displacement from the
equilibrium position is its energy half potential and half kinetic ?
(A) 1 cm (B) 2 cm (C) 2 cm (D) 2 2 cm .

Page # 97
SIMPLE HARMONIC MOTION PHYSICS PART - IV

5. Which of the following expressions does not represent Simple Harmonic Motion ?
(A) A cos t (B) A sin 2t (C) A sin t + B cos t (D) Aesint
6. For a particle executing SHM the displacement x is given by x = A cos  t. Identify the graph which representss
the variation of potential energy (PE) as a function of time t and displacement x.
I II E IV
III

t x
–A A

(A) I, III (B) II, IV (C) II, III (D) I, IV.


7. A particle executes simple harmonic motion with amplitude A and time period T. The average speed of the
particle over n complete oscillations is :
(A) 0 (B) A/nT (C) 4A/nT (D) 4A/T
8. The total energy of the body executing S.H.M. is E. Then the kinetic energy, when the displacement is half
of the amplitude, is:
(A) E/2 (B) E/4 (C) 3E/4 (D) 3/ 4 E

SYSTEM EXECUTING SHM


Simple Pendulum
A small mass (bob) suspended by a light, long and inextensible string forms simple pendulum.
Length of the simple pendulum is the distance between the point of suspension and the centre of mass
of the suspended mass.
Consider the bob when string deflects through a small angle  .
Force acting on the bob are tension T in the string and weight mg of the bob.
Torque on the bob about point O is
  mg  T  mg lsin   0

   mgl as is very small.. …(i)


M.I. of the bob about the point o is

d2 
Hence   ml2 …(ii)
dt 2

O
O
 T

l l

mg sin mg mg cos

As torque  and  are oppositely directed hence from (i) and (ii), we gett

d2 d2
ml2  mg l   (g / l)
dt 2 dt 2

d2 x g
Comparing with the equation  2 x , we get  
dt 2 l

l
Since T  2 /  T  2
g

Page # 98
SIMPLE HARMONIC MOTION PHYSICS PART - IV

Note: If a simple pendulum is made to oscillate in a non inertial frame of reference, the torque due to pseudo
force must be taken into account.
MEASUREMENT OF ACCELERATIONDUE TO GRAVITY USING A SIMPLE PENDULUM
For of acceleraton deflection, the time period of a simple pendulum of effective length  eff is given by
y

 eff  
T  2 or, g  42  eff2 
g T 

d
Where left    r   
2

  eff 
By determining the average value of  2  experimentally, g can be calculated..
T 
Clamp
Split
Cork S S S
Stand
Thread
Table

l
eff

B Hook
C A Lab Floor r
G G Bob
5cm 5cm
for the different values of  the time period (T) is measured and graph is plotted between  and T2.

 tan= slope of curve


= average value of /T2

T2

Illustration :
A simple pendulum is executing simple harmonic motion with a time period T. If the length of the pendulum
is increased by 21%. Find the percentage increase in time period.

l T
Solution : T  2 g (Percentage increment in time is  100  10% )
T

T2 l 1.21l
  2   1.1 T2  1.1T1
T1 l1 l

Spring Mass System: Let us find out the time period of a spring-mass system oscillating on a smooth
horizontal surface as shown in the figure.
At the equilibrium position the spring is relaxed. When the block is displaced through a distance x towardss
right, it experiences a net restoring force F  kx towards left..
The negative sign shows that the restoring force is always opposite to the displacement. That is , when x
is positive, F is negative, the force is directed to the left. When x is negative, F is positive, the force always
tends to restore the block to its equilibrium position .

Page # 99
SIMPLE HARMONIC MOTION PHYSICS PART - IV

d2 x
Applying Newton’s Second Law, F=-kx or Fm  kx
dt 2

d2 x  k 
  x  0
dt 2  m 

d2 x
Comparing the above equation with, a   2 x we get
dt 2

k m
2  or T  2
m k

k Smooth
m

x
k
m

Block of mass m attached with a spring of stiffness k oscillates on a smooth horizontal plane.
Note: Time period is independent of the amplitude. For a given spring constant, the period increases with the
mass of the block that means more massive block oscillates more slowly. For a given block, the period
decreases as k increases. A stiffer spring produces quicker oscillations.
Illustration :
Find the period of oscillation of a vertical spring-mass system.
Let x0 be the deformation in the spring in equilibrium. Then kx0  mg When the block is further displaced
by x , the net restoring force is given by F  [k(x  x0 )  mg]

d2 x  k  m
or    x  0 Thus, or T  2 .
dt 2  m  k
Note: Time period is independent of the amplitude. For a given spring constant, the period increases with the
mass of the block that means more massive block oscillates more slowly. For a given block, the period
decreases as k increases. A stiffer spring produces quicker oscillations.
Using second law of motion,

d2 x d2x  k 
m   kx or  x 0
dt 2 dt 2  m 

k m
Thus, 2  or T  2 .
m k

kx0 k(x+x0)
k k k

x0

x mg mg
m
A block of mass m attached to a spring of stiffness k oscillates in a vertical plane.
Combinations of Springs
(a) Series combination of spring
When two spring are joined in series, the equivalent stiffness of the combination may be obtained as
1 1 1
 
k k1 k 2
Page # 100
SIMPLE HARMONIC MOTION PHYSICS PART - IV

k1 k2 k
m m

(b) Parallel combination of spring


When two springs are joined in parallel, the equivalent stiffness of the combination is given by
k  k1  k2

k1
k
k2 m m

(c) Two body sping system


Asystem of two bodies connected by a spring so that both are free to oscillate simple harmonically along
the length of the spring constitutes a two body harmonic oscillator.
Suppose, two masses, m1 and m2 are connected by a horizontal massless spring of force constant k, so as
to be free to oscillate along the length of the spring on a frictionless horizontal surface.

 K
Then the system is given by T  2 m1 m2
k

1 2 mm
Where   m  m is the reduced mass of system
1 2

Illustration :
3 
A spring of stiffness constant k and natural length  is cut into two parts of length and respectively,
ely,
4 4
and an arrangement is made as shown in the figure. If the mass is slightly displaced, find the time period of
oscillation.
Solution: The stiffness of a spring is inversely proportional to its length. Therefore the stiffness of each part is
4 m 3m  3m
k 1  k and k 2  4k Time period,, T  2 T  2
k 1  k 2 or .

3 16k 2 k

k1=(4/3)k k2=4k

Physical Pendulum or Compound Pendulum : Any rigid body suspended from a fixed support
constitutes a physical pendulum. Consider the situation when the body is displaced through a small angle
 . Torque on the body about O is given by
y
  mglsin  …(i)
where l  distance between point of suspension and centre of mass of the body..
If I be the M.I. of the body about O.
Then   I …(ii)
From (i) and (ii), we get
d2  d2 
I   mglsin  as  and 2 are oppositely directed.
ed.
dt 2 dt

d2  mgl
 2
  Since  is very small..
dt I

d2  mgl
Comparing with the equation  2 , we get 
dt 2 I

Page # 101
SIMPLE HARMONIC MOTION PHYSICS PART - IV

I
 T  2 .
mgl
Equivalent length of Simple pendulum : Moment of inertia of compound pendulum can also be written
by theorem of parllal axis as :
O
2 2 2 2 2
I  Icm  MI  MK  MI  M(K  l )
G
K2 
2 2 1 leq
M(K  l )
 T  2l  2 l  2
Mgl g g
mg
K2
where leq   l is the equivalent length of simple
e
l

Illustration :
A disc of mass M and radius R is suspended from one point on its periphery and is lying in the vertical plane
as shown in the figure so that it can rotate freely in the vertical plane about a horizontal axis through the
point of suspension. It is slightly displaced in the same plane as shown. Find the angular frequency of
oscillations. about a horizontal

 +y
Solution:
+x

R sin

3 2g 2g
   mg  R sin   MR 2  or   comparing with   2  we get  
2 3R 3R
Torsional Pendulum
In torsional pendulum, an extended body is suspended by a light thread or a wire. The body is rotated
through an angle about the wire as the axies of rotation.

The wire remains vertical during this motion but a twist ‘  ’ is produced in the wire. The twisted wire a
restoring torque on the body, whish is proportional to the angle of twist.
  
  k  ; k  is proportionality constant and is called torsional constant of the wire.

Page # 102
SIMPLE HARMONIC MOTION PHYSICS PART - IV

If I be the moment of inertia of the body about the verical axis. The angular acceleration is
 k
    2
I I

k 2 I
  time period T  2
I  k
Illustration :
The moment of inertia of at the disc used in a trosional endulum about the suspension wire is 0.2 kg- m 2 .
It oscillates with a period of 2 s. Another disc is placed over the first one and the time period of the system
becomes 2.5 s. Find the moment of inertia of the second disc about the wire.
Solution : Let the torsional constant of the wire be k.

I 0.2
T  2  2 2 …..(i)
K k
when the second disc having moment of inertie 1 about the wire is added, the time period is,

0.2  I
2.5 = 2 ……(ii)
K
from (i)&(ii), I1  0.11kg  m2

COMPOSITION OF TWO SHMS IN THE SAME DIRECTION


Let the direction be along x-axis and the simple harmonic motions produced by two forces
 
F1 & F2 be x1  A1 sin t
x2  A 2 sin(  t  ) respectively
y
From above discussion, the resultant postion of the particle is then,
x  x1  x 2  A1 sin t  A 2 sin  t   
Put A 1  A 2 cos   Acos 
A 2 sin   Asin 

 A  (A 1  A 2cos )2  (A 2sin )2

A  A 21  A 22  2A1 A 2 cos  ….(i)

and x  A cos  sin t  A sin  cos t


x  Asin(  t  ) ...(20)
A2 sin 
and tan   A  A cos  …(21)
1 2

The amplitude of resulttant simple harmonic motion is given by

A  A 21  A 22 2 A 1 A 2 cos  …(22)

It is maximum when  =0

A max  (A 1  A 2 )2  A 1  A 2

It is minimum when cos   1i.e.,   

A min  (A 1  A 2 )2  A1  A 2
Page # 103
SIMPLE HARMONIC MOTION PHYSICS PART - IV
The amplitude of resulttant simple harmonic motion is given by

A  A 21  A 22  2A1 A 2 cos 

TRY YOURSELF
9. Figure (A) and (B) shows a mass m connected to two identical springs as shown.
The ratio of frequency of vibration in case (A) and (B) is :

K
K K
K

(a) (b)

(A) 1 : 1 (B) 1 : 2 (C) 1 : 4 (D) 3 : 1.


10. A pendulum bob carries a +ve charge +q. A positive charge +q is held at the point of support.
Then the time period of the bob is
(A) greater than 2 L / g (B) less than 2 L / g
(C) equal to 2 L / g (D) equal to 2 2L / g
11. One end of a long metallic wire of length L tied to the ceiling. The other end is tied with a massless spring of
spring constant K. A mass hangs freely from the free end of the spring. The area of cross section and the
young’s modulus of the wire are A and Y respectively. If the mass slightly pulled down and released, it will
oscillate with a time period T equal to :
(A) 2 (m / K) (B) 2 m(YA  KL) / (YAK) (C) 2 (m YA / KL) (D) 2 (mL / YA)
12. A particle of mass m is attached to three identical springs A, B and C each of force constant k as shown in
figure. If the particle of mass m is pushed slightly against the spring A and released, then the time period of
oscillation is :

B C
120º
(

O m

2m m
(A) 2 (B) 2 m (C) 2
2m
(D) 2
k 2k 3k 3k
13. Two pendulums begin to swing simultaneously. The first pendulum makes 9 full oscillations when the
other makes 7. The ratio of lengths of the two pendulum is
(A) 9/7 (B) 7/9 (C) 49/81 (D) 81/49
14. The springs in fig. A and B are similar but length in A is three times each of that in B.
The ratio of period TA /TB is :
(A) 3
(B) 1/3
(C) 3
(D) 1/ 3

15. A disc of mass M, and radius R is attached to the massless rod as shown.
The period of oscillation of the pendulum is :
L L
(L  R)
(A) T  2 (B) T  2
g g M
R

Page # 104
SIMPLE HARMONIC MOTION PHYSICS PART - IV

L  R 2 / 2L
(C) T  2 (D) None of these
g
16 One end of a spring of force constant k is fixed to a vertical wall and other to a body of mass m resting on
a smooth horizontal surface there is another wall at a distance x0 from the body. The spring is then
compressed by 2x0 and released. The time taken to strike the wall is :

 k k x0
(A) (B)
6 m m

A B C
2 m  k 2x0
(C) (D)
3 k 4 m

Page # 105
SIMPLE HARMONIC MOTION PHYSICS PART - IV

EXERCISES
ONLY ONE CHOICE IS CORRECT
1. A particle is moving on a circle with uniform speed; its motion is
(A) Periodic and simple harmonic (B) Periodic but not simple harmonic
(C) Aperiodic (D) None of the above
2. Which one of the following is SHM?
(A) Motion of a particle in a wave moving through a string fixed at both ends
(B) Earth spinning about its own axis
(C) Ball bouncing between two rigid vertical walls
(D) particle moving in a circle with uniform speed
3. The eqation of SHM with amplitude 5 cm and time period 0.5 sec is.

(A) y  5sin  4t  (B) y  5sin  t 

(C) y  0.5sin  2t / 5 (D) y  5sin  2t /5


4. A simple harmonic motion has an amplitude A and time period T. The time required by it to travel from x
= A to x = A/2 is
(A) T/6 (B) T/4 (C) T/ 3 (D) T/ 2

5. A particle is excuting SHM wiht amplitude A and has a maximum velocity v0 . (a) At what displacement will
its velocity be v0 /2 ? (b) What is its velocity at displacement A/2 ?

3 1 1 3 3
(A) A, v 0 (B) A, v (C) 2A, v (D) 2A, v0
2 2 2 0 4 0 4
6. A particle executes simple harmonic motion between x = -A and x = + A. the time taken for it to go from 0
to A/2 is and to go from A/2 to A is . Then :
(A) T1  T2 (B) T1  T2 (C) T1  T2 (D) T1  2T2
7. A particle executes simple harmonic motion. The amplitude of the vibration of particle is 2 cm. The
displacement of particle in one time period is
(A) 1 cm (B) 2 cm (C) 4 cm (D) zero
8. For a S.H.M oscillator, the states possible are
(A) y = 0 ; v = min ; a = 0 (B) y = A ; v = 0 ; a = max
(C) y = A ; v = max ; a = 0 (D) y = A ; v = 0 ; a = min
9. The displacement of a particle executing SHM is given by y  0.01sin100(t  0.05) . The time period is:
(A) 0.01 sec (B) 0.02 sec (C) 0.1 sec (D) 0.2 sec.
10. A simple pendulum is made of a bob which is a hollow sphere full of sand suspended by means of a wire.
If all the sand is drained out, the period of the pendulum will
(A) Increase (B) Decrease (C) Remain same (D) Become erratic
11. The period of a simple pendulum is doubled when:
(A) Its length is doubled (B) Its length is halved
(C) The length is made four times (D) The mass of the bob is doubled

Page # 106
SIMPLE HARMONIC MOTION PHYSICS PART - IV

12. A hollow metal sphere is filled with water through a small hole in it. It is hanging by a long thread
and is made to oscillate. Water slowly flows out of the hole at the bottom. How will the period of oscillation
be affected?
(A) The period will go on increasing till the sphere is empty
(B) The period will go on decreasing till the sphere is empty
(C) The period will go on decreasing till the sphere is empty
(D) The period will first increase then decrease till the sphere is empty
13. A pendulum suspended from the ceiling of a train has a period t when the train is at rest. When the train is
accelerating with a uniform acceleration a, the period of oscillation will:
(A) Increase (B) Decrease (C) Remain same (D) Become infinite
14. A simple pendulum hung from the ceiling of a train at rest has a peiod T. If it starts accelerating, the time
period will :
(a) Remain unchanged (b) Increases
(c) Decreases (d) nothing can be said
15. A pendulum is hung from the ceiling of a car accelerating uniformly . If it is left undisturbed as before the
acceleration of car, it will

(A) Tilt away from mean position by tan   a


g

a
(B) Tilt forward from mean postion by tan  
g

a
(C) Oscillate with T = 2
g

1 g
(D) Oscillate with T =
2 a
16. A clock S is based on oscillations of a spring and a clock P is based on pendulum motion. Both clocks
run at the same rate on earth. On a planet having the same density as earth but twice the radius:
(A) S will run faster than P
(B) P will run faster than S
(C) both run at the same rate as on the earth
(D) both run at equal rates but not the same as on the earth
17. A weight suspended from a spring oscillates up and down. The PE of the weight is zero at:
(A) Highest point (B) Lowest point (C) Middle point (D) None of these
18. A particle of mass m is hanging vertically by an ideal spring of force constant k. If the mass is made to
oscillate vertically, its total energy is:
(A) Maximum at the extreme position (B) Maximum at the mean position
(C) Miniumum at the mean position (D) Same at all the positions
19. A mass m oscillates with SHM with frequency f   / 2 and amplitude A on a spring of force constant
k. Therefore:

(A) The total energy of the spring is (1/2)kA2 (B) the frequency is 1/2  k / m
(C) The maximum velocity occurs when x = 0 (D) all the above statements are correct

Page # 107
SIMPLE HARMONIC MOTION PHYSICS PART - IV

20. Figure (A) and (B) shows a mass m connected to two identical springs as shown. The ratio of frequency of
vibration in case (A) and (B) is

K
K K
K

(a) (b)

(A) 1 : 1 (B) 1:2 (C) 1 : 4 (D) 3 : 1.


21. A uniform circular disc of mass 12 kg is held by two identical springs as shown in the figure. When the disc
is pressed down slightly and released, it executes SHM with a time period of 2 sec. The force constant of each
spring is

(A) 236 N m–1 (B) 118.3 N m–1 (C) 59.15 N m–1 (D) None of the above
22. A body executes simple harmonic motion. The potential energy (PE), kinetic energy (KE) and total energy
(TE) are measured as a function of displacement x. Which of the following statements is true?
(A) TE is zero when x = 0 (B) PE is maximum when x = 0
(C) KE is maximum when x = 0 (D) KE is maximum when x is maximum
23. A particle is vibrating in simple harmonic motion with an amplitude of 4 cm. At what displacement from
the equilibrium position is its energy half potential and half kinetic ?
(A) 1 cm (B) 2 cm (C) 2 cm (D) 2 2 cm .
24. A particle executes a SHM with frequency  . The frequency of K.E. will be :

v
(A) 2  (B)  (C) (D) 3
2
25. A ring and a disc of same mass and radius are hinged at periphery oscillate in vertical plane about mean
position if T1 & T2 are their time period respectively then
(A) T1 > T2 (B) T1 < T2 (C) T1 = T2 (D) All of the above
26. A thin circular disc of radious R and mass M is hinged at its periphery in a vertical plane.If it is slightly
displaced from equilibrium position and then released then the time period of oscillation of disc will be;

3R 2R 5R 2R
(A) 2 (B) 2 (C) 2 (D) 2
2g 3g 2g 5g

27. The function (sin2 t) representss


(A) A periodic but not simple harmonic motion

2
(B) A simple harmonic motion with a period

(C) A simple harmonic motion with a period twice that of cos t
(D) A simple harmonic motion with a period twice that of cot t

Page # 108
SIMPLE HARMONIC MOTION PHYSICS PART - IV

28. The velocity time graph of a simple harmonic oscillator is as shown in figure. Its frequency of oscillations
will be

4
2
0
.01 .02 .04 0.6 .08 t
2


(A) 25 Hz (B) 50 Hz (C) 12.25 Hz (D) 33.3 Hz.


29. The time period of a particle in simple harmonic motion is equal to the time between consecutive appear-
ances of the particle at a particular point in its motion. This point is
(A) the mean position
(B) an extreme position
(C) between the mean position and the positive extreme
(D) between the mean position and the negative extreme

30. A particle moves on the X-axis according to the equation x  x 0 sin 2 t . The motion is simple harmonic

(A) with amplitude x0 (B) with amplitude 2x0

2 
(C) with time period (D) with time period .
 
31. The graph plotted between the velocity and displacement from mean position of a particle
executing S.H.M. is
(A) circle (B) ellipse (C) parabola (D) straight line.
32. The displacement fuction of a S.H.M. is given by y= A cos (t  ) . If at t = 0, the displacement is 1 cm and
initial velocity is ð cms 1 , the amplitude and epoch are :

3 3 3 
(A) 2, (B) 2 , (C) 2 (D) 2 ,
4 4 4 4

v
33. A particle executing S.H.M. between x =-A to x = + A .The time taken to get retarded form v to is t 1
2
v
and to zero is t 2 .[v-Maximum velocity]
y]
2
(A) t1  t2 (B) t1  t 2 (C) t1  t2 (D) t1  2t2

34. A plank of negligible mass oscillates with a body of mass m as governed by v  sin t  3 cos t . The
time at which the mass just loses conact is :

2  2 2  2
(A) (B) (C) (D)
3g 3 g 3 g g6

Page # 109
SIMPLE HARMONIC MOTION PHYSICS PART - IV

35. The function (sin2 t) representss


(A) A periodic but not simple harmonic motion

2
(B) A simple harmonic motion with a period

(C) A simple harmonic motion with a period twice that of cos t
(D) A simple harmonic motion with a period twice that of cot t
36. A simple pendulum consists of a small sphere of mass m suspended using a leangth ’L’. A charge q is given to
the sphere and an electric field E is applied vertically upwards. The time period of the oscillation is :

2L L 1 L
2 2 1 g  qE
(a) qE (b) qE (c) (d) 2 qE
g g 2 L g
m m m
37. The free end of a simple pendulum is attached to the ceiling of a box . The box is taken to a height and the
pendulum is oscillated. When the bob is at its lowest point, the box is released to fall freely. As seen from the
box (during this period ), the bob will
(A) Continue its oscillations as before (B) Stop
(C) Will go in a circular path (D) Moving on a straight line
38. A simple pendulum has time period T1 on the earth’s surface , and T2 when taken to a height R above the
earth’s surface, where R is the radius of the earth. The value of T2 /T1 is :
(A) 1 (B) 3 (C) 4 (D) 2
39. A simple pendulum is oscillating without damping. When the displacement of the bob is less than maximum,

its acceleration vector a is correctly shown in :

a
(A) (B)
a

(C) a 0 (D)
a

40. A man measures time period of a pendulum (T) in stationary lift. If the lift moves upwards with acceleration
g/4, then the new time period will be

2T 5T 5 2
(A) (B) (C) (D)
5 2 2T 5T
41. For a simple pendulum the graph between L and T will be :
(A) hyperbola (B) parabola (C) a curved line (D) a straight line. .
42. A pendulum clock keeping correct time is taken to high altitudes,
(A) it will keep correct time
(B) its length should be increased to keep correct time
(C) its length should be decreased to keep correct time
(D) it cannot keep correct time even if the length is changed.

Page # 110
SIMPLE HARMONIC MOTION PHYSICS PART - IV

43. Four springs are attached to a mass m as shown. The time period will be 2  times :
K
2K 2K
. M
2K

m 2m 4m m
(a) (b) (c) (d)
K K K 4K
44. An ideal spring with spring-constant k is hung from the ceiling and a block of mass M is attached to its lower
end. The mass is released with the spring initially unstretched. Then the maximum extension in the spring
is
4Mg 2Mg Mg Mg
(A) (B) (C) (D) .
k k k 2k
45. Simple Pendulum attached to the ceiling of a stationary lift has a time period T. when the lift moves upward
with distance covered as y = (1 m/s²)t², the time period of the pendulum will be Equal to ;
(take g = 10.0 m/s²)

4 5 5 6
(A) T (B) T (C) T (D) T
5 6 4 5
46. A block of mass m moves with a speed v towards the right block in equilibrium with a spring. If the
surface is frictionless and collistions are elastic, the frequency fo collisions between the masses will be :

K
v
m m
L
2
v 1 K v 1 K  2L
 2   m
(A) (B) (C)    (D) none of these
2L  m  2L  m 
v K

47. A mass M is suspended from a spring of negligible mass. The spring is pulled a little and then released, so that
the mass executes SHM of time period T. If the mass is increased by m, the time period becomes 5T/3. Then
the ratio of m/M is
(A) 3/5 (B) 25/9 (C) 16/9 (D) 5/3
48. A uniform spring whose unstressed length is l, has a force constant K. The spring is cut into two pieces of
unstretched length l1 and l2 , wheree l2  nl1 , n being an integer. Now a mass m is made to oscillate with first
spring. The time period of its oscillation would be

mn m m m(n 1)
(A) T  2 (B) T  2 (C) T  2 (D) T  2
K(n  1) nk k(n  1) nk
49. The total energy of the body executing S.H.M. is E . Then the kinetic energy, when the displacement is halff
of the amplitude, is :

(A) E/2 (B) E/4 (C) 3E/4 (D) 2 / 4E .


50. A particle executes a SHM with frequency  The frequency of P.E. will be :

v
(A) 2v (B)  (C) (D) 3
2

Page # 111
SIMPLE HARMONIC MOTION PHYSICS PART - IV

51. A particle at the origin is under the influence of a force F = kx(where k is a positve constant). If the potential
energy U is zero at x = 0 , the variation of potential energy with the coordianate x is represented as :
U(x) U(x)

x x
(A) (B)

U(x) U(x)

x x
(C) (D)

52. Starting from the mean position a body oscillates simple harmonically with a period of 2 s. After what time
will its kinetic energy be 75% of the total energy ?
1 1 1 1
(A) s (B) s (C) s (D) s
6 4 3 12
53. A thin circular ring of radious R and mass M is hinged at its periphery in a vertical plane.If it is slightly
displaced from equilibrium position and then released then the time period of oscillation of ring will be;

R 2R R
(A) 2 (B) 2 (C) 2 (D) none of the above
g g 2g

54. A mass of 0.98 kg suspended using a spring of constant k= 300 N m 1 is hit by a bullet of 20gm moving with
a velocity 3m/s vertically. The bullet gets embedded and oscillates with the mass. The amplitude of oscillation
will be :
(A) 0.15cm (B) 0.12cm (C) 1.2cm (D) 12 m.
55. Three S.H.M’s of equal amplitude A and equal time period in the same direction combine. The difference
in phase between each pair is 60º ahead of the other. The amplitude of the resultant oscillation is :
(A) a (B) 2a (C) 0 (D) 4a
56. Two S.H.M’s of unequal amplitude with same frequency (act) on a particle at right angle, with a phase

difference of . The resultant motion is :
2
(A) A straight line at 45º to x- axis (B) An ellipse
(C) A circle (D) Dependent on ratio of amplitudes
57. Two simple pendulum of length 1 m and 16 m respectively are given small displacement.The number of
oscillations ‘N’ of the smaller pendulum for them to be in phase is :

4 3 1
(A) (B) (C) 4 (D)
3 4 16
58. The period of oscillation of simple pendulum of length L suspended from the roof of a vehicle which moves
without friction down as inclined plane of inclination è, is given by :

L L L L
(A) 2 (B) 2 (C) 2 (D) 2
g cos  g sin  g g tan
59. A particle, which is constrained to move along the x-axis, is subjected to a force in the same direction
which varies with the distance x of the particle from the origin as F(x) = -kx + ax³. Here k and a are positive
constants. For x  0 , the functional form of the potential energy U(x) of the particle is :

Page # 112
SIMPLE HARMONIC MOTION PHYSICS PART - IV

U(x) U(x)

(A) (C)
x x

U(x) U(x)

(B) (D)
x x

60. For a particle executing simple harmonic motion , the displacement x is given by x = a cos t .Identify
the graph which represents the variation of potential evergy(P.E.) as a function of time and displacement :

(A) I,III (B) II,IV (C) II,III (D) I,IV


61. A body is excuting a S.H.M. such that its potential energy is E1 at x and E 2 at y. when the displacement is x
+ y, the potential energy will be

(A) E1+E 2 (B) E21 E22 (C) E1  E2  2 E1 .E2 (D) E1E2

62. Three springs of constants K 1 , K 2 and K 3 are used to hang a mass m from the roof. The elastic potential
ial
energy stored in the array is
M2g 2  1 1 1   1 1 1 
(A)     (B) Mg    
2  K1 K 2 K3   K1 K2 K 3 

 1 1 1  Mg  1 1 1 
(C) M2g 2     (D)    
 K 1 K 2 K3  2  K1 K2 K3 


63. Driver of a car records a period of seconds a pedulum of 1m hung from the roof. The acceleration of the
3
car is
(A) 10 ms-2 (B) 15 ms-2 (C) 17.2 ms-2 (D) 34.5 ms-2 34.5 ms–2.
64. A uniform rod of lengh l is mounted so as to rotate about a horizontal axis perpendicular to the rod and at
a distance x from the centre of mass. The time period will be the least when x is

l l l l
(A) (B) (C) (D)
4 2 3 12
65. A bob of mass M is hung using a string of length l. A mass m moving wiht a velocity u pierces through the
u
bob and emerges out wiht velocity . The frequency of oscillation of the bob considering as amplitude A
3
is

Page # 113
SIMPLE HARMONIC MOTION PHYSICS PART - IV

3mu 1 2m 1  2mu 
(A) 2 (B) (C)   (D) cannot be found
2MA 2 3MA 2  3MA 
66. A block A of mass m is placed on frictionless horizontal surface. Another block B of same mass is kept on A
and connected to the wall with the help of spring of spring constant k is shown in the figure. The coefficient
of friction between A and B is  s . the blocks move together performing simple harmonic motion of ampli-
tude a . the maximum value of the frictional force between A and B is :

A
smooth

(A) ka (B) ka/2 (C) Zero (D)  s mg


67. A metallic disc oscillates about an axis through its edge in it`s own plane. The equivalent length of the disc
as a pendulum is

r 3r
(A) r (B)  (C) (D)
2 2
3
68. A particle oscillating as given by U(y) = K y with force constant K has an amplitude A.The maxumum
velocity during the oscillation is proportional :
(A) to A (B) proportional to A3

2K 3 2K 3
(C) A (D) A
m K

69. An inductor L and a capacitor C are connected in parallel after initial charging (Q) of the capacitor. The
frequencies of the variation of the energy i capacitor and inductor are v1 and v 2 respectively. Then

1 1
(A) v1  v 2  (B) v1  v 2 
2 LC  LC
1
(C) v1  2v 2  LC (D) v1  v 2  LC
2
70. A solid sphere (M) attached to a massless spring (K) can roll without splipping along a horizontal surface.
If the sysstem is released after an intial stretch, the time period will be :

3M 3M 7M M
(A) 2 (B) 2 (C) 2 (D) 2
2K 5K 5K K

Page # 114
SIMPLE HARMONIC MOTION PHYSICS PART - IV

71. A block of mass m , attached to a fixed position O on a smooth inclined wedge of mass M , oscillates with
amplitude A and linear frequency f . The wedge is located on a rough horizontal surface. If the angle of the
wedge is 60º, then the force of friction acting on the wedge is given by
(coefficient of static friction   ).

O
(A) (M m)g
A
1
(B) m2 A sin t m
2
M
 3  smooth
rough
 
(C)  (M m)g  2 m A sin t 
2
60º
 

(D) (M m)2 A sin t


72. A loop consists of two cords of lengths l and 2l, and their masses per unit length are  and 2 . It is placed in
stable equilibrium over a smooth peg as shown in the figure. When slightly displaced, it executes SHM. The
period of oscillation is Peg

l
5t 5
(A) 2 (B) 2 A B
3g g

5 5 2l
(C) 2 (D) 2 2
2g g
73 A particle at the end of a spring executes simple harmonic motion with a period t1, while the corresponding
period for another spring is t2. If the period of oscillation with the two springs in series is T, then
(A) T  t1  t2 (B) T2  t12  t 22 (C) T1 t2 1 t2 1 (D) T2  t1 2 t2 2

ONE OR MORE THAN ONE CHOICE MAY BE CORRECT


1. The motion of a ball bouncing between two rigid walls is
(A) Periodic (B) Oscillatory
(C) Linear simple harmonic (D) Angular simple harmonic
2. The motion of a torsional pendulum is:
(A) Periodic (B) Oscillatory
(C) Linear simple harmonic (D) Angular simple harmonic
3. If different types of pendulums are taken to the moon, the time period will not change for:
(A) Simple pendulum (B) Spring pendulum
(C) Physical pendulum (D) Torsional pendulum
4. The displcement equation of a spring block system is given by y  A sin t in air. It is completely immersed
in water if A1 and  ' be new amplitude and angular frequency then
(A)   ' (B) A = A1 (C)   ' (D) A1 < A
5. Which of the following functions represent SHM?

(A) sin 2t (B) sin 2 t (C) sin t  2 cos t (D) sin t  cos 2t

Page # 115
SIMPLE HARMONIC MOTION PHYSICS PART - IV

6. A simple pendulum has a time period T, with the bob provided a positive charge. Then,
(A) Placing a positive charge at athe point of suspension will not alter the time period.
(B) Placing a negative charge at the point of suspension will not alter the time period.
(C) A horizontal electric field will alter the time period.
(D) A vertical electric field will not alter the time period.
7. A wall clock uses a vertical spring - mass system to measure the time. Each time the mass reaches an
extreme position the clock advances by a second. the clock gives correct time at the equator. If the clock is
taken to the poles, its time period will be
(A) Less (B) More (C) Independent of g (D) Same
8. Two springs A and B of force constants 2 and 3 N/m are given. The ratio of work done in A to that in B is
(A) 2 : 3 for same extension (B) 3 : 2 for same extension
(C) 2 : 3 for same force (D) 3 : 2 for same force

9. A particle moves along the x-axis according to the equation x  4  3sin  2 t  Here, x is in cm and t in
seconds, Select the correct alternative (s):
(A) The motion of the particle is SHM with mean position at x = 0
(B) The motion of the particle is SHM with mean position at x = 4
(C) The motion of the particle is SHM with mean position at x = -4
(D) Amplitude of oscillation is 3 cm
10. A simple pendulum of length 1 m with a bob of mass m swings with an angular amplitude 300. Then: (g =
9.8m/s2)
(A) time period of pendulum is 2s
(B) tension in the string is greater than mg cos 150 at angular displacement 150.
(C) rate of change of speed at angular displacement 150 is g sin 150
(D) tension in the string is mg cos 150 at angular displacment 150.
11. In a spring-block system force constant of spring is k = 16 N/m, mass of the block is 1 kg. Maximum kinetic
energy of the block is 8J. Then:
(A) amplitude of oscillation is 1m
(B) at half the amplitude potential energy stored in the spring is 2 J
(C) at half the amplitude kinetic energy is 6 J
(D) angular frequency of oscillation is 16 rad/s
12. In SHM:
(A) potential energy and kinetic energy may not be equal in mean position
(B) potential energy and kinetic energy may be equal in extreme position
(C) potential energy may be zero at extreme position
(D) kinetic energy plus potential energy oscillates simple harmonically
13 Two blocks A and B each of mass m are connected by a massless spring of natural length l and spring
constant K. The blocks are initially resting on a smooth horizontal floor with the spring at its natrural length
as shown. A third identical block C also of mass m, moves on the floor with a speed v along line joining A
and B and collide with A. then
C v B A
m

Page # 116
SIMPLE HARMONIC MOTION PHYSICS PART - IV

(A) The K.E. of the A-B system, at the maximum compression of the spring is zero.
(B) The K.E. of the system at maximum compression of the spring is mv 2 / 4

m
(C) The maximum compression of the spring is v
K

m
(D) The maximum compression of the spring is v
2K
14. A load of mass M falls from a height h on a pan attached to a spring(K) and stays on it. Then :

2mgh
(A) Amplitude of vibration will be 1 
K
(B) Energy all through the oscillation will be same.

2Kh
(C) Amplitude of oscillation will be mg 1
mg

 1 2mgh 
(D) Amplitude of oscillation will be mg  1  K 1  K 
 
15. A particle of mass m is attached to three identical springs A, B and C each of force constant k as shown in
figure. If the particle of mass m is pushed slightly against the spring A and released, then the time period of
oscillation is : B C

m O m
(A) Extention in springs are same. (B) 2
2k

m
(C) Extension in A is different from B and C. (D) 2 A
3k
16. In a simple harmonic motion if potential energy at mean position is zero then;
(A) the maximum potential energy equals the maximum kinetic energy
(B) the minimum potential energy equals the minimum kinetic energy
(C) the minimum potential energy equals the maximum kinetic energy
(D) the maximum potential energy equals the minimum kinetic energy
17. Select the correct alternatives
(A) A simple harmonic motion is necessarily periodic.
(B) A simple harmonic motion is necessarily oscillatory
(C) Oscillation motion may be periodic
(D) A periodic motion is necessarily oscillatory
18. Which of the following quantities are always negative in a SHM ?
       
(A) F .a (B) v.r (C) a.r (D) F. r
19. Which of the following will change their time period if they are taken to moon ?
(A) a simple pendulum (B) a physical pendulum
(C) a torsional pendulum (D) a spring pendulum

Page # 117
SIMPLE HARMONIC MOTION PHYSICS PART - IV

20. Which of the following is/are the characteristics(s) of SHM?


(A) projection of uniform circular motion on any straight line
(B) periodic nature
(C) displacement time graph is a sine curve
(D) acceleration is zero at the mean position
21. Which of the following statements are true for the oscillations of the mass suspended with a spring?
(A) Time period varies directly as the square root of the suspended mass.
(B) A stiffer spring gives lesser time period
(C) The mass can execute oscillations in the state of weightlessness
(D) The system will have the same time period both on the moon and earth
22. for an oscillating pendulum, tension is
(a) Maximum at the mean position (b) Zero at the extreme position
(c) The least at the ex (d) Never zero.
23. A coin of mass m placed on a horizontal platform undergoing a S.H.M. about a mean position O.If the force
of friction on the coin is f, while the coin does not slip on the platform then f, is
(a) Directed towards O always.
(b) Directed towards O When the coin moves out.
(c) Directed towards O When the coin moves in way
(d) Maximum when mass and platform are at rest.
24. Function :  x  A sin 2 t  B cos 2 t  C sin t represents S.H.M.

(a) For any value of A, B and C (except C = 0) (b) If A = -B , C = 2B amplitude = B 2

(c) If A = B , C = 0 (d) If A = B, C = 2B, amplitude = B


25. A cylindrical block of density  remains completely immersed in two immiscible liquids (equally)of density
y

1 and  2 in a container. The cylinder when pressed a little in, will experience :
(A) A S.H.M. verically
(B) the displacement of the cylinder up and down is equal
(C) the displacement of the cylinder up and down is unequal
(D) S.H.M. is absent
26. A simple pendulum with length L and mass m of the bob is vibrating with an amplitude a.Then the maximum
tension in the string is not :

 a 2    a 
2

(A) mg (B) mg 1    


 (C) mg 1    (D) None of these

  L     2L 

27. A uniform spring has an unstretched length L and a force constant k. the spring is cut into two parts of
unstretched length l1 and l2 such that l1 = l2 wheree  is an integer. The corresponding force constantss
k1 and k 2 are :

k(  1)
(A) force has to be kept same to find k1 and k2 (B)  and k(1)

k( 1) k(  1)
(C)  and k(  1) (D)  and k(  1)

Page # 118
SIMPLE HARMONIC MOTION PHYSICS PART - IV

28. A particle vibrates in SHM along a straight line. Its greatest acceleration is 52 cm s 2 and when itss
distance from the equalibrium positon is 4cm, the velocity of the particle is 32 cm s 1 :
(A) The amplitude is 10cm (B) the peiod of oscillation 2 sec
(C) the amplitude is 5 cm (D) the period of oscillation 2 sec

29. Two springs of equal constant K are attched in two different ways as shown in figure. If F1, F2 are the
restoring forces in them then,

(B)

(A) M

(A) F1 = F2 in B (B) F1 = F2 in Az
(C) Ratio of time period is 2 :1 (D) Ratio of time period is 2 :1
30. The object as it falls through the distance are measured with a pendulum clock. The entire apparatus is
taken on the moon and the experiment is repeated,
(A) the measured times are same (B) the measured speeds are same
(C) The actual times in the fall are equal (D) the actual speeds are equal.

31. The particle is half way to its end point in a S.H.M. (where E is the total energy) then :

E 5E
(A) P.E. = (B) P.E. = (C) P.E= K.E (D) K.E = 3 P.E.
4 4
32. A particle having SHM is located at different positions as shown below. The phase difference
between the position marked by A and
(A) B is  /2 radian (B) C is  radian (C) E is >2  radian (D) D is 2  radian
y=0
A
B
C
D
E

PASSAGE BASED QUESTIONS


I. A simple harmonic oscillator of force constant K oscillates with amplitude a in a frictionless medium.

1. The position average of its potential energy is

1 1 1 2 1 2
(A) KA 2   (B) KA 2 (C) KA (D) KA
2 4 6 8
2. If the amplitude of oscillation is icreased by 20%, the increase in energy becomes (in %)
(A) 20 % (B) 40 % (C) 41 % (D) 44 %

Page # 119
SIMPLE HARMONIC MOTION PHYSICS PART - IV

3. On doubling the mass oscillating and the amplitude being halved, the energy E of the oscillator becomes,

E E
(A) 2 E (B) 4E (C) (D)
2 4
II. In simple harmonic motion force acting on a particle is given as F = -4x, total mechanical energy of the
particle is 10 J and amplitude of oscillations is 2 m. At time t = 0 acceleration of the particle is - 16 m/s2. Mass
of the particle of 0.5 kg.
4. Potential energy of the particle in mean posion is:
(A) 10 J (B) 8J (C) 6 J (D) 2 J
5. Displacement time equation of the particle is:
(A) x = 2sin 2t (B) x = 2sin 4t (C) x = 2cos 2t (D) None of the above
6. At x = +1m, potential energy and kinetic energy of the particle are:
(A) 2 J and 8 J (B) 8 J and 2 J (C) 6 J and 4 J (D) 4 J and 6 J
III. Two identical blocks P and Q have mass m each. They are attached to two identical springs initially
unstretched. Now the left spring (along with P) is compressed by A/2 and the right spring (along with Q) is
compressed by A. both the blocks are released simultaneously. They collide perfectly inelastically. Initially
time period of both the block was T.
A/2 A
P Q

7. The time period of oscillation of combined mass is:

T T
(A) (B) 2T (C) T (D)
2 2

8. the amplitude of combined mass is:

A A 2A 3A
(A) (B) (C) (D)
4 2 3 4
9. What is energy of oscillation of the combined mass?

1 2 1 2 1 2 1 2
(A) kA (B) kA (C) kA (D) kA
2 4 8 16
IV. For SHM to take place force acting on the body should be proportional to - x or f = -kx. If A be the amplitude
then energy of oscillation is 1/2 KA2.

10. Force acting on a block is F = (-2x + 8). Here F is in newton and x the position of block on x-axis in metres:

(A) Motion of the block is periodic but not simple harmonic

(B) Motion of the block is not periodic

(C) Motion of the block is simple harmonic about origin, x = 0

(D) Motion of the block is simple harmonic about x = 2m

Page # 120
SIMPLE HARMONIC MOTION PHYSICS PART - IV

11. If energy of oscillation is 18 J, between what points the block will osicllate ?

(A) between x = 0 and x = 4 m (B) between x = -1m and x = 5 m

(C) between x = -2m and x = 6m m (D) between x = 1m and x = 3 m

12. The amplitude of oscillation is

(A) 4 cm (B) 2 cm (C) 1 cm (D) 3 cm

MATCH THE COLUMN TYPE QUESTIONS


This section contains 3 questions. Each questions contains statements given in two columns
which have to be matched. Statements (A, B, C D) in Column - I have to be matched with statements
(p, q, r, s) in Column - II.

1. y is displacement, A is amplitude and PE at mean position is zero.


Column - I Column - II

A
(A) y  (P) KE = PE
2

A
(B) y  (Q) KE = 1/2 PE
2

2
(C) y  A (R) KE = 0
3
(D) y = A (S) KE = 3 PE
(T) PE = TE

 2 
2. In y  Asin t  Asin  t   match the following
 3 
Column - I Column - II
(a) Motion (p) is periodic but not SHM
(b) Amplitude (q) is SHM
(c) Initial phase (r) A


(d) Maximum velocity (s)
3
(t) None

3. In SHM match the following:


Column - I Column - II
(a) Acceleration - displacement graph (p) parabola
(b) Velocity-acceleration graph (q) straight line
(c) acceleration-time graph (r) circle
(d) Velocity-time graph (s) None

Page # 121
SIMPLE HARMONIC MOTION PHYSICS PART - IV

4. In case of second’s pendulum, match the following (consider shape of earth also):
Column - I Column - II
(a) At pole (p) T > 2s
(b) On a satellite (q) T < 2s
(c) At mountain (r) T = 2s
(d) At centre of earth (s) T = 0
(t) T = 
5. In a spring-block system; match the following:
Column - I Column - II
(a) If mass of the block is doubled (p) Energy of oscillation becomes 4 times
(b) If amplitude of oscillation is doubled (q) Speed of particle becomes 2 times
(c) If force contant is doubled (r) Time period becomes times
2
(d) If angular frequency is doubled (s) Potential energy becomes 2 times

INTEGER TYPE QUESTIONS


 
1. A particle of mass 0.10 kg has its velocity varying according to the relation v  4 sin  t  ms
 4 
What is the maximum kinetic energy? [Take  2  10 ]
2. The ratio of maximum to minimum tension in an oscillating pendulum is 2. If  is the angular amplitude and
 = cos-1 (x × 10-1) then find x.

3. In the given spring block system if k  25 2 Nm-1 then find time period of oscillation

K
5kg 5kg
K K
K

4. A plank with a bar on it executes a S.H.M of amplitude A  10 cm. What should be the coefficient of friction
(in x × 10-1) which prevents sliding of the bar for time period 1.0 second.
[take  2  g  10m / sec 2 ].
5. A mass m attached to a spring A has a frequency of 3 Hz and a spring B has a frequency of 4 Hz. When
both the springs are connected as shown in figure find the frequency (in Hz) of oscillation
A B
m

6. A 2 kg mass moving with a speed of 5 ms 1 collides inelastically with another mass of 3 kg rest attechd to
o
a spring of constant K  50 N/m whose other end is fixed to a rigid wall. Find the maximum compression
in the spring in x × 10-4 m. where x is nearest integer.
7. A tiny mass perfroms S.H.M . along a straight line with a time period if T  0.60 second and amplitudee
A  10.0 cm Calculate the meam velocity (in ms-1)in the time to displace by A/2..

Page # 122
SIMPLE HARMONIC MOTION PHYSICS PART - IV

8. Figure below show a massless a pully, a spring of constant K  1000 N/m and a mass 1 kg. On displacing
the mass slightly , if the freduency of its vertical oscillation is x × 101 Htz. Then find x?

9. An electric dipole formed by charges e,m  1027 kg.each separated by 0.01 mm is placed in a field of

E  10 5 N/C. Caculate the time period of oscillation. (in x × 10 -4 sec.)


10. A mass m is allowed to slide down a plane ( 1  600 ) from a verticle height of 15 cm and allowed to climb
another plane ( 2  300 ) . What is the time period of oscillation in seconds?
11. If the displacement of a moving point at any time be given by an equation of the form
y(t)  a cos t  b sin t show that the motion is simple harmonic. If a  3m, b  4m and   2; find the
maximum acceleration in multiple of 10.

12. Consider a liquid which fills a uniform U-tube, as shown in the figure upto a height
h = 10 m. Find the angular frequency of small oscillations of the liquid in the U--
tube. (g = 10 ms-2) h

13. A uniform plank of mass m = 1 kg, free to move in the horizontal direction only, is placed at the top of a solid
cylinder of mass 2m and radius R. The plank is attached to a fixed wall by means of a light spring of spring
constant k = 7 N/m2. There is no slipping between the cylinder and the plank, and between the cylinder and
the ground. Find the angular frequency of small oscillations of the system.

m k

R
2m

Page # 123
SIMPLE HARMONIC MOTION PHYSICS PART - IV

Questions asked in previous AIEEE / JEE MAINS


1. A spring when connected by mass m gives time period 'T'. If spring is cut in n equal parts and each part
connected in parallel with same mass. New time- period will be - [AIEEE-2002]

(A) nT (B) T/n (C) T/ n (D) nT


2. A child is sitting on a swing and swinging. If he stands up. The time period of swing will [AIEEE-2002]
(A) increase
(B) decrease
(C) remain same
(D) increase if the child is long and decrease if the child is short
3. In a simple harmonic oscillator, at the mean position – [AIEEE-2002]
(A) Kinetic energy is minimum, potential energy is maximum
(B) Both kinetic energy and potential energies are maximum
(C) Kinetic energy is maximum, potential energy is minimum
(D) Both kinetic & potential energies are minimum
4. A mass M is suspended from a spring of negligible mass. The spring is pulled a little and then released so that
5T
the mass executes SHM of time period T. If the mass is increased by m, the time period becomes . Then
3
m
the ratio of is – [AIEEE-2003]
M
(A) 25/9 (B) 16/9 (C) 5/3 (D) 3/5
5. The length of a simple pendulum executing simple harmonic motion is increased by 21%. The percentage
increase in the time period of the pendulum of increased length is – [AIEEE-2003]
(A) 21% (B) 42% (C) 10% (D) 11%
6. Two particles A and B of equal masses are suspended from two massless springs of spring constants k1 and
k2 , respectively. If the maximum velocities, during oscillation, are equal, the ratio of amplitudes of A and B
is [AIEEE-2003]
k2 k2 k1 k1
(A) k1 (B) k1 (C) k2 (D) k2
7. The displacement of a particle varies according to the relation x = 4 (cos t + sin t). The amplitude of the
particle is – [AIEEE-2003]
(A) 4 (B) 42 (C) 8 (D) – 4
8. A body executes simple harmonic motion. The potential energy (P.E.), the kinetic energy (K.E.) and total
energy (T.E.) are measured as a function of displacement x. Which of the following statement is true ?
[AIEEE-2003]
(A) T.E. is zero when x = 0 (B) K.E. is maximum when x is maximum
(C) P.E. is maximum when x = 0 (D) K.E. is maximum when x = 0
9. The bob of a simple pendulum executes simple harmonic motion in water with a period t, while the period
of oscillation of the bob is t0 in air. Neglecting frictional force of water and given that the density of the bob
is (4/3) × 1000 kg/m3. What relationship between t and t0 is true ? [AIEEE-2004]
(A) t = t0 (B) t = t0 / 2 (C) t = 2t0 (D) t = 4t0
10. A particle at the end of a spring executes SHM with a period t1, while the corresponding period for another
spring is t2. If the period of oscillation with the two springs in series is T, then – [AIEEE-2004]

(A) T = t1 + t2 (B) T2 = t 12 + t 22 (C) T–1 = t 11 + t 21 (D) T–2 = t 12 + t 22

Page # 124
SIMPLE HARMONIC MOTION PHYSICS PART - IV

11. The total energy of a particle, executing simple harmonic motion is – [AIEEE-2004]
(A) X (B)  X2 (C) Independent of X (D)  X1/2
where x is the displacement from the mean position.
12. A particle of mass m is attached to a spring (of spring constant k) and has a natural angular frequency 0. An
external force F(t) proportional to cos  t (  0) is applied to the oscillator. The time displacement of the
oscillator will be proportional to – [AIEEE-2004]
m 1 1 m
(A) (B) (C) (D)
(  2 )
2
0 m (  2 )
2
0 m (  2 )
2
0
(   2 )
2
0

13. In forced oscillation of a particle the amplitude is maximum for a frequency 1 of the force, while the energy
is maximum for a frequency 2 of the force ; then – [AIEEE-2004]
(A) 1 = 2
(B) 1 > 2
(C) 1 < 2 when damping is small and 1 > 2 when damping is large
(D) 1 < 2
14. The function sin2(t) represents [AIEEE-2005]
(A) a periodic, but not simple harmonic motion with a period 2/
(B) a periodic, but not simple harmonic motion with a period /
(C) a simple harmonic motion with a period 2/
(D) a simple harmonic motion with a period /
d2 x
15. If a simple harmonic motion is represented by  x = 0, its time period is [AIEEE-2005]
dt 2
2 2
(A) (B) (C) 2 (D) 2
 
16. The bob of a simple pendulum is a spherical hollow ball filled with water. A plugged hole near the bottom of
the oscillating bob gets suddenly unplugged. During observation, till water is coming out, the time period of
oscillation would [AIEEE-2005]
(A) first increases and then decrease to the original value
(B) first decrease and then increase to the original value
(C) remain unchanged
(D) increase towards a saturation value
17. The maximum velocity of a particle, executing simple haromonic motion with an amplitude
7 mm, is 4.4 m/s. The period of oscillation is – [AIEEE 2006]
(A) 0.1 s (B) 100 s (C) 0.01 s (D) 10 s
18. Starting from the origin a body oscilates simple haromonically with a period of 2 s. After what time will its
kinetic energy be 75% of the total energy – [AIEEE 2006]

1 1 1 1
(A) s (B) s (C) s (D) s
3 12 6 4

19. A coin is placed on a horizontal platform which unergoes vertical simple harmonic motion of angular
frequency . The amplitude of oscillation is gradually increased. The coin will leave contact with the plat-
form for the first time– [AIEEE 2006]
(A) for an amplitude of g2/2 (B) at the highest position of the platform
(C) at the mean position of the platform (D) for an amplitude of g/2

Page # 125
SIMPLE HARMONIC MOTION PHYSICS PART - IV

20. The displacement of an object attached to a spring and executing simple harmonic motion is given by
x = 2 × 10-2 cos t metres. The time at which the maximum speed first occurs is -
[AIEEE 2007]
(A) 0.5 s (B) 0.75 s (C) 0.125 s (D) 0.25 s
21. A point mass oscillates along the x-axis according to the law x = x 0 cos ( t – /4). If the acceleration
of the particle is written as a = A cos ( t + ), then [AIEEE 2007]
 
(A) A = x0 ,  = – (B) A = x0 2 ,  =
4 4
 3
(C) A = x0 2,  = – (D) A = x0 2,  =
4 4
22. A block of mass ‘m’ is connected to another block of mass ‘M’ by a spring (mass less) of spring constant
‘k’. The blocks are kept on a smooth horizontal plane. Initially the blocks are at rest and the spring
is unstretched. Then a constant force ‘F’ starts acting on the block of mass ‘M’ to pull it. Find the force
on the block of mass ‘m’. [AIEEE 2007]

mF ( M  m)F mF MF
(A) (B) (C) (D)
M m ( m  M) ( m  M)
23. Two springs, of force constant k1 and k2, are connected to a mass m as shown. The frequency of
oscillation of the mass if f. If both k1 and k2 are made four times their original values, the frequency
of oscillation becomes [AIEEE 2007]
k1 m k2

(A) f/2 (B) f/4 (C) 4f (D) 2f


24. A particle of mass m executes simple harmonic motion with amplitude ‘a’ and frequency ‘v’. The
average kinetic energy during its motion from the position of equilibrium to the end is
[AIEEE 2007]

1
(A) 2m a2 v2 (B) m a2 v 2 (C) 2m a2 v2 (D) 22m a2 v2
4
25. If x, v and a denote the displacement, the velocity and the acceleration of a particle executing simple
harmonic motion of time period T, then, which of the following does not change with time ?
[AIEEE-2009]
(A) aT/x (B) aT + 2v (C) aT/v (D) a2T2 + 42v2
26. Two particles are executing simple harmonic motion of the same amplitude A and frequency w along
the x-axis. Their mean position is separated by distance X 0 (X0 > A). If the maximum separation
between them is (X0 + A), the phase difference between their motion is: [AIEEE-2011]
   
(A) (B) (C) (D)
3 4 6 2
27. A mass M, attached to a horizontal spring, executes S.H.M. with amplitude A1. When the mass M passes
through its mean position then a smaller mass m is placed over it and both of them move together
 A1 
with amplitude A 2. The ratio of   is: [AIEEE-2011]
A
 2
1/ 2 1/ 2
Mm  M  Mm M
(A) (B)   (C)   (D)
M Mm  M  Mm

Page # 126
SIMPLE HARMONIC MOTION PHYSICS PART - IV

28. If a simple pendulum has significant amplitude (up to a factor of 1/e of original) only in the period
between t = Os to t = ts, then t may be called the average life of the pendulum. When the spherical
bob of the pendulum suffers a retardation (due to viscous drag) proportional to its velocity, with ‘b’
as the constant of proportionality, the average life time of the pendulum is (assuming damping is
small) in seconds: [AIEEE 2012]

0.693 1 2
(A) (B) b (C) (D)
b b b
29. The amplitude of a damped oscillator decrease to 0.9 times its original magnitude is 5s. In another 10s
it will decrease to  times its original magnitude, where  equals. s. [JEE Mains-2013]
(A) 0.81 (B) 0.729 (C) 0.6 (D) 0.7
30. A particle moves with simple harmonic motion in a straight line. In first  s, after starting from rest
it travels a distance a, and in next  s it travels 2a, in same direction, then : [JEE MAINS 2014]
(A) amplitude of motion is 4a (B) time period of oscillations is 6 
(C) amplitude of motion is 3a (D) time period of oscillations is 8 
31. For a simple pendulum, a graph is plotted between its kinetic energy (KE) and potential energy (PE) against
its displacment d. WHich one of the following represents these correctly ? [JEE MAINS 2015]
(Graphs are schematic and not drawn to scale)
E E
KE PE

(a) (b)
PE KE
d d
E
KE

E
PE
d
(c) (d)
KE

PE
32. A particle is executing simple harmonic motion with a time period T. AT time t = 0, it is at its position of
equilibrium. The kinetic energy-time graph of the particle will look like [JEE MAIN 2017]

(a) (b)

(c) (d)

Page # 127
SIMPLE HARMONIC MOTION PHYSICS PART - IV

Questions asked in previous IIT JEE/JEE ADVANCED


ONLY ONE CHOICE IS CORRECT
1. A particle executes simple harmonic motion with a frequency, f. The frequency with which its kinetic
energy oscillates is : (1987; 2M)

f
(a) (b) f (c) 2f (d) 4f
2
2. Two bodies M and N of equal masses are suspended from two separate massless springs of spring constant
k1 and k2 respectively. If the two bodies oscillate vertically such that their maximum velocities are equal,
the ratio of the one amplitude of vibration of M to that of N is : (1988; 1M)

k1 k1 k2 k2
(a) K2 (b) K2
(c) k1 (d) k1

3. A uniform cylinder of length L and mass M having cross-sectional area A is suspended, with its length
vertical, from a fixed point by a massless spring, such that it is half-submerged in a liquid of density x  s
at equilibrium position. When the cylinder is given a small downward push and released it starts oscillating
vertically with a small amplitude. If the force constant of the spring is k, the frequency of oscillation of the
cylinder is : (1990; 2M)
1/2 1/2 1/2 1/2
1  k  Ag  1  k  Ag  1  k  gL2  1  k  Ag 
(a)   (b)   (c)   (d)  
2  M  2  M  2  M  2  Ag 

4. A highly rigid cubical block A of small mass M and side L is fixed rigidly on to another cubical block B of the
same dimensions and of low modulus of rigidity h such that the lower face of A completely covers the upper
face of B. The lower face of B is rigidly held on a horizontal surface. A small force F is applied perpendicular
to one of the sides faces of A. After the force is withdrawn, block A executes small oscillations, the time
period of which is given by : (1995 S)

M ML M
(a) 2 ML (b) 2 (c) 2p (d) 2p
L  L
5. One end of a long metallic wire of length L is tied to the celling. The other end is tied to a masless spring of
spring constant K. A mass m hangs freely from the free end of the spring. The area of cross-section and the
Young’s modulus of the wire are A and Y respectively. If the mass is slightly pulled down and released, it will
oscillate with a time period T equal to : (1993; 2M)
1/2 1/2 1/2
m m(YA  KL  mYA   mL 
(a) 2  (b) 2 (c) 2P   (d) 2P  
k YAK  KL   YA 
6. A particle of mass m is executing oscillation about the origin on the x-axis. Its potential energy is U(x) = k
|x|3, where k is a positive constant. If the amplitude of oscillation is a, then its time period T is :
(1998; 2M)
1
(a) proportional to (b) independent of a
a

(c) proportional to a (d) proportional to a3/2


7. A spring of force constant k is cut into two pieces such that one piece is double the length of the other. Then
the long piece will have a force constant of : (1999; 2M)

2 3
(a) k (b) k (c) 3k (d) 6 k
3 2

Page # 128
SIMPLE HARMONIC MOTION PHYSICS PART - IV

8. A particle free to move along the x-axis has potential energy given by U(x) = k[1 – exp (–x2)] for –   x
+  , where k is a positive constant of appropriate dimensions. Then : (1999; 2M)
(a) at points away from the origin, the particle is in unstable equilibrium
(b) for any finite non-zero value of x, there is a force directed away from the origin

k
(c) if its total mechanical energy is , it has its minimum kinetic energy at the origin
2
(d) for small displacements from x = 0, the motion is simple harmonic
9. The period of oscillation of simple pendulum of length L suspended from the roof of the vehicle which
moves without friction, down an inclined plane of inclination a, is given by : (2000; 2M)

L L L L
(a) 2 (b) 2 (c) 2 (d) 2
g cos  g sin  g g tan 
10. A particle executes simple harmonic motion between x = –A and x = + A. The time taken for it to go from O
A A
to is T1 and to go from to A is T2, then : (2001; 2M)
2 2
(a) T1 < T2 (b) T1 > T2 (c) T1 = T2 (d) T1 = 2T2
11. For a particle executing SHM the displacement x given by x = A cos wt. Identify the graph which represents
the variation of potential energy (PE) as a function of time t and displacement x: (1999; 2M)
(a) I, III (b) II, IV (c) II, III (d) I, IV
12. A simple pendulum has time period T1. The point of suspension is now moved upward according to the
relation y = Kt2, (K = 1 m/s2) where y is the vertical displacement. The time period now becomes T2. The
T12
ratio of is : (g = 10 m/s2) (2005; 2M)
T22

6 5 4
(a) (b) (c) 1 (d)
5 6 5
ONE OR MORE THAN ONE CHOICE MAY BE CORRECT
13. A linear harmonic oscillator of force constant 2 × 106 N/m and amplitude 0.01 m has a total mechanical
energy of 160 J. Its : (1989; 2M)
(a) maximum potential energy is 100 J (b) maximum kinetic energy is 100 J
(c) maximum potential energy is 160 J (d) maximum potential energy is zero
14. Three simple harmonic motions in the same direction having the same amplitude and same period are
superimposed. If each differ in phase from the next by 45°, then : (1999; 3M)
(a) the resultant amplitude is (1 + 2 )a
(b) the phase of the resultant motion relative to the first is 90°
(c) the energy associated with the resulting motion is (3 + 2 2 ) times the energy associated with anyy
single motion
(d) the resulting motion is not simple harmonic

Page # 129
SIMPLE HARMONIC MOTION PHYSICS PART - IV

SUBJECTIVE TYPE QUESTION


15. Two masses m1 and m2 are suspended together by a massless spring of spring constant k (Fig). When the
masses are in equilibrium, m1 is removed without disturbing the system. Find the angular frequency and
amplitude of oscillation of m2. (1981; 3M)
16. An ideal gas is enclosed in a vertical cylindrical container and supports a freely moving piston of mass M.
The piston and the cylinder have equal cross-sectional area A. Atmospheric pressure is P0 and when the
piston is in equilibrium, the volume of the gas is V0. The piston is now displaced slightly from its equilibrium
position. Assuming that the system is completely isolated from its surroundings, show that the piston
executes simple harmonic motion and find the frequency of oscillation. (1981; 6M)
17. A thin fixed ring of radius 1 m has a positive charge 1 × 10–5 C uniformly distributed over it. A particle of
mass 0.9 g and having a negative charge of 1 × 10–6 C is placed on the axis at a distance of 1 cm from the
centre of the ring. Show that the motion of the negatively charged particle is approximately simple harmonic.
Calculate the time period of oscillations. (1982; 5M)
18. Two light springs of force constant k1 and k2 and a block of mass m are in one line AB on a smooth
horizontal table such that one end of each spring is fixed on rigid supports and the other end is free as
shown in the figure. The distance CD between the free ends of the spring is 60 cm. If the block moves along
AB with a velocity 120 cm/s in between the springs, calculate the period of oscillation of the block. (k1 = 1.8
N/m, k2 = 3.2 N/m, m = 200 g) (1985; 6M)
19. A point particle of mass M attached to one end of a massless rigid non-conducting rod of length L. Another
point particle of the same mass is attached to the other end of the rod. The two particles carry charges +q
and –q respectively. This arrangement is held in a region of a uniform electric field E such that the rod
makes a small angle  (say of about 5 degree) with the field direction, fig. Find an expression for the
minimum time needed for the minimum time needed for the rod to become parallel to the field after it is set
free. (1989; 8M)
20. Two non-viscous, incompressible and immiscible liquids of densities  nd 1.5  are poured into the two wo
limbs of a circular tube of radius R and small cross-section kept fixed in a vertical plane as shown in fig.
Each liquid occupies one-fourth the circumference of the tube. (1991; 4 + 4M)
(i) Find the angle q that the radius to the interface makes with the vertical in equilibrium position.
(ii) If the whole liquid column is given a small displacement from its equilibrium position, shown that the
resulting oscillations are simple harmonic. Find the time period of these oscillations.
21. Two identical balls A and B, each of mass 0.1 kg, are attached to two identical massless springs. The spring-
mass system is constrained to move inside a rigid smooth pipe bent in the form of a circle as shown in
figure. The pipe is fixed in a horizontal plane. The centers of the balls can move in a circle of radius 0.06 m.
Each spring has a natural length of 0.06p metre and spring constant 0.1 N/m. Initially, both the balls are

displaced by an angle  = radian with respect to the diameter PQ
Q of the circle (as shown in fig.) and
6
released from rest. (1993; 6M)
(i) Calculate the frequency of oscillation of ball B.
(ii) Find the speed of ball A when A and B are at the two ends of the diameter PQ.
(iii) What is the total energy of the system?
22. A thin rod of length L and uniform cross-section is pivoted at its lowest point P inside a stationary
homogeneous and non-viscous liquid. The rod is free to rotate in a vertical plane about a horizontal axis
passing through P. The density d1 of the material of the rod is smaller than the density d2 of the liquid. The
rod is displaced by small angle  from its equilibrium position and the released. Show that the motion of the
rod is simple harmonic and determine its angular frequency in terms of the given parameters.
(1996; 5M)
Page # 130
SIMPLE HARMONIC MOTION PHYSICS PART - IV

23. A solid sphere of radius R is floating in a liquid of density r with half of its volume submerged. If the sphere
is slightly pushed and released, it starts performing simple harmonic motion. Find the frequency of these
oscillations. (2004; 4M)
24. A mass m is undergoing SHM in the vertical direction about the mean position y0 with amplitude A and
angular frequency  . At a distance y from the mean position, the mass detaches from the spring. Assume
that the spring contracts and does not obstruct the motion of m. Find the distance y. (measured from the
mean position) such that the height h attained by the block is maximum. (  2A > g) (2005)
25. The x-t graph of a particle undergoing simple harmonic motion is shown below. The acceleration of the
particle at t = 4/3 s is [IIT JEE - 2009]

3 2  2 2 3 2
(A)  cm/s 2 (B) cm/s 2 (C) cm/s 2 (D)   cm/s 2
32 32 32 32
26. A uniform rod of length L and mass M is pivoted at the centre. Its two ends are attached to two springs of
equal spring constants k. The springs are fixed to rigid supports as shown in the figure, and rod is free to
oscillate in the horizontal plane. The rod is gently pushed through a small angle  in one direction and
released.The frequency of oscillation is [IIT JEE-2009]

1 2k 1 k 1 6k 1 24k
(A) (B) (C) (D)
2 M 2 M 2 M 2 M
27. The mass M shown in the figure oscillates in simple harmonic motion with amplitude A. The amplitude of
the point P is [IIT JEE-2009]

k1 A k 2A k 1A k 2A
(A) (B) (C) (D)
k2 k1 k1  k 2 k1  k 2

Page # 131
SIMPLE HARMONIC MOTION PHYSICS PART - IV

Comprehension :
When a particle of mass m moves on the x-axis in a potential of the form V(x) = kx2, it performs simple
m
harmonic motion. The corresponding time period is proportional to , as can be seen easily using
k
dimensional analysis. However, the motion of a particle can be periodic even when its potential energy
increases on both sides of x = 0 in a way different from kx2 and its total energy is such that the particle does
not escape to infinity. Consider a particle of mass m moving on the x-axis. Its potential energy is V(x) = x4
( > 0) for |x| near the origin and becomes a constant equal to V0 for |x|  X0 (see figure) [JEE 2010]

28. If the total energy of the particle is E, it will perform periodic motion only if :

(A) E<0 (B) E>0 (C) V0 > E > 0 (D) E > V0


29. For periodic motion of small amplitude A, the time period T of this particle is proportional to :

m 1 m  1 
(A) A (B) (C) A (D)
 A  m A m
30. The acceleration of this particle for |x| > X0 is :
V0
(A) proportional to V0 (B) proportional to mX
0

V0
(C) proportional to mX0 (D) zero

31. A ball of mass m = 5 kg is attached to the end of a string having length L = 0.5 m. The ball is rotated on a
horizontal circular path about vertical axis. The maximum tension that the string can bear is 324 N. The
maximum possible value of angular velocity of ball (in radian/s) is : [JEE 2011]

(A) 9 (B) 18 (C) 27 (D) 36


32. A metal rod of length ‘L’ and mass ‘m’ is pivoted at one end. A thin disk of mass ‘M’ and radius ‘R’ (<L) is
attached at its center to the free end of the rod. Consider two ways the disc is attached: (case A). The disc is
not free to rotate about its center and (case B) the disc is free to rotate about its center. The rod-disc system
performs SHM in vertical plane after being released from the same displaced position. Which of the following
statement(s) is (are) true? [JEE 2011]

Page # 132
SIMPLE HARMONIC MOTION PHYSICS PART - IV

(A) Restoring torque in case A = Restoring torque in case B


(B) Restoring torque in case A < Restoring torque in case B
(C) Angular frequency for case A > Angular frequency for case B.
(D) Angular frequency for case A < Angular frequency for case B.

Paragraph for Question Nos. 33 to 35

Phase space diagrams are useful tools in analyzing all kinds of dynamical problems. They are especially
useful in studying the changes in motion as initial position and momentum are changed. Here we consider
some simple dynamical systems in one-dimension. For such systems, phase space is a plane in which
position is plotted along horizontal axis and momentum is plotted along vertical axis. The phase space
diagram is x(t) vs. p(t) curve in this plane. The arrow on the curve indicates the time flow. For example, the
phase space diagram for a particle moving with constant velocity is a straight line as shown in the figure.
We use the sign convention in which position or momentum. upwards (or to right) is positive and
downwards (or to left) is negative. [JEE 2011]
33. The phase space diagram for a ball thrown vertically up from ground is :

Momentum Momentum

Position
(A) Position (B)

Momentum Momentum

(C) Position (D)


Position

34. The phase space diagram for simple harmonic motion is a circle centered at the origin. In the figure, the two
circles represent the same oscillator but for different init ial conditions, and E1 and E2 are the total mechanical
energies respectively. Then

(A) E1  2 E 2 (B) E1  2 E 2 (C) E1  4 E 2 (D) E1  16 E 2

Page # 133
SIMPLE HARMONIC MOTION PHYSICS PART - IV

35. Consider the spring-mass system, with the mass submerged in water, as shown in the figure. The phase
space diagram for one cycle of this system is :

Momentum

(A) Position (B)

(C) (D)

36. A point mass is subjected to two simultaneous sinusoidal displacements in x-direction, x1 (t) = A sin t and

 2 
x2 (t) = A sin  t   . Adding a third sinusoidal displacement x3 (t) = B sin (t + ) brings the mass to a
 3 
complete rest. The values of B and  are [JEE 2011, 3/160, –1]

3 4 5 
(A) 2 A, (B) A, (C) 3 A, (D) A,
4 3 6 3
37. A small block is connected to one end of a massless spring of un-stretched length 4.9 m. The other end of the
spring (see the figure) is fixed. The system lies on a horizontal frictionless surface. The block is stretched by
0.2 m and released from rest at t = 0. It then executes simple harmonic motion with angular frequency
   / 3 rad/s. simultaneously at t = 0, a small pebble is projected with speed v from point P at an angle of
450 as shown in the figure. Point P is at a horizontal distance of 10 m from O. If the pebble hits the block at
t = 1 s, the value of v is (take g = 10 m/s2) [IIT JEE 2012]

Page # 134
SIMPLE HARMONIC MOTION PHYSICS PART - IV

(A) 50m / s (B) 51m / s (C) 52m / s (D) 53m / s


38. A particle of mass m is attached to one end of a mass-less spring of force constant k, lying on a frictionless
horizontal plane. The other end of the spring is fixed. The particle starts moving horizontally from its
equilibrium position at time t = 0 with an initial velocity u0. When the speed of the particle is 0.5 u0. It
collides elastically with a rigid wall. After this collision. [JEE Advanced-2013]
(A) the speed of the particle when it returns to its equilibrium position is u0.
(B) the time at which the particle passes through the equilibrium position for the first time

m
is t  
k

4 m
(C) the time at which the maximum compression of the spring occurs is t 
3 k
(D) the time at which the particle passes through the equilibrium position for the second time is

5 m
t
3 k
39. Two independent harmonic oscillators of equal mass are oscillating about the origin with angular frequen-
cies 1 and 2 and have total energies E1 and E2, respectively. The variations of their momenta p with
a a
positions xare shown in the figures. If  n 2 and  n , then the correct equation(s) is (are)
b R
[JEE MAINS 2015]

2
(a) E11  E 22 (B)  n2
1

E1 E 2
(C) 12  n 2 (D) 
1 2

Page # 135
SIMPLE HARMONIC MOTION PHYSICS PART - IV

ANSWER KEY
TRY YOURSELF
1. B 2. C 3. B 4. D 5. D 6. A
7. D 8. C 9. B 10. C 11. B 12. C
13.C 14. C 15. C 16. C

ONLY ONE CHOICE IS CORRECT

1. (B) 2. (A) 3. (A) 4. (A) 5. (A) 6. (A)


7. (D) 8. (B) 9. (B) 10. (C) 11. (C) 12. (D)
13. (B) 14. (C) 15. (A) 16 (B) 17. (C) 18. (D)
19. (D) 20. (B) 21. (C) 22. (C) 23. (D) 24. (A)
25. (A) 26. (A) 27. (A) 28. (A) 29. (B) 30. (D)
31. (B) 32. (A) 33. (A) 34. (B) 35. (A) 36. (B)
37. (C) 38. (A) 39. (C) 40. (A) 41. (D) 42. (C)
43. (C) 44. (B) 45. (D) 46. (C) 47. (C) 48. (C)
49. (C) 50. (B) 51. (D) 52. (A) 53. (B) 54. (B)
55. (B) 56. (B) 57. (A) 58. (A) 59. (A) 60. (A)
61. (C) 62. (A) 63. (D) 64. (D) 65. (C) 66. (B)
67. (D) 68. (C) 69. (B) 70. (C) 71. (B) 72. (C)
73. (B)

ONE OR MORE THAN ONE CHOICE MAY BE CORRECT


1. (A, B) 2. (A, B, D) 3. (B, D) 4. (A, D) 5. (A, C) 6. (A, B, C)
7. (C, D) 8. (A, D) 9. (B,D) 10. (B, C) 11. (A,B,C) 12. (A,B,C)
13. (B, D) 14. (B, C) 15. (B, C) 16. (A,B) 17. (A,B,C) 18. (C,D)
19. (A,B) 20. (B,C,D) 21. (A,B,C,D) 22. (A,B,C,D) 23. (A,C,D) 24. (A,B,D)
25. (A,B) 26. (B) 27. (A,B,C) 28. (A,D) 29. (B,C) 30. (A,B,C)
31. (B) 32. (A,D)

PASSAGE BASED QUESTIONS


1. (C) 2. (D) 3. (D) 4. (D) 5. (D) 6. (D)
7. (C) 8. (A) 9. (D) 10. (D) 11. (B) 12. (A)

MATCH THE COLUMN TYPE QUESTIONS


1. (A-p; B-s; C-q; D-r, t) 2. (A-q; B-r; C-s; D-t)
3. (A-q; B-s; C-s; D-s) 4. (A-q; B-t; C-p; D-t)
5. (A-r; B-p; C-s; D-p,q)

Page # 136
SIMPLE HARMONIC MOTION PHYSICS PART - IV

INTEGER TYPE QUESTIONS


1. 8 2. 7 3. 1 4. 2 5. 5 6. 6
7. 1 8. 1 9. 3 10. 6 11. 2 12. 1
13. 2

Questions asked in previous AIEEE / JEE MAINS


1. B 2. B 3. C 4. B 5. C 6. B
7. B 8. D 9. C 10. B 11. C 12. B
13. A 14. B 15. B 16. A 17. C 18. C
19. D 20. A 21. C 22. C 23. D 24. A
25. A 26. A 27. C 28. A 29. B 30. B
31. B 32. B

Questions asked in previous IIT JEE/JEE ADVANCED

ONLY ONE CHOICE IS CORRECT


1. C 2. B 3. B 4. D 5. B 6. A
7. B 8. D 9. A 10. A 11. A 12. A
ONE OR MORE THAN ONE CHOICE MAY BE CORRECT
13. a, c 14. a, c
SUBJECTIVE TYPE QUESTION

k mg 1 V0 M
15. w = , A 1 16. 2  P A2  MgA 17. 0.628 s
m2 k 0 
 ML 1
18. 2.82 s 19. 20. (a) q = tan–1 5
2 2qE

R 1
(b) 2 21. (i) Hz (ii) 0.0628 m/s (iii) 3.9 × 10–4 J
6.11 

3g  d 2 – d1  1 3g g
22. w = 23. 24.
2d1 L 2 2 R 2
25. D 26. C 27. D 28. C 29. B 30. D
31. D 32. A, D 33. B 34. C 35. B 36. B
37. A 38. A, D 39. B,D

Page # 137
WAVES ON STRING AND SOUND
WAVES
3
 Theory

 Try YourSelf
CONTENTS
 Solved Examples

 EXERCISES

 Single Correct Answer

 More than One Correct Questions

 Passage Questions

 Matrix Match Type Questions.

 Integer Type Questions

 Previous Years Questions

 Answer Key
AVES ON STRING AND SOUND WAVES PHYSICS PART - IV

INTRODUCTION - DEFINITION :
A wave is a disturbance which propagates energy (and momentum) from one place to the other without
the transport of matter. It is well spread over a region of space without clear cut boundaries. It cannot be
said to be localized here or there.

TYPES OF WAVE
(a) Electromagnetic wave
The waves which do not require medium for their propagation are called non–mechanical e.g., light,
heat (infrared) and radio waves are non–mechanical as they can propagate through vacuum. In fact all
electromagnetic waves (EMW) such as   rays , X–rays or microwaves are non–mechanical..
(b) Mechanical wave :
The waves which require medium for their propagation are called mechanical waves. In the
propagation of mechanical waves elasticity and density of the medium play an important role. This is
why mechanical waves sometimes are also referred to as elastic waves.

CLASSIFICATION OF WAVES
Mechanical waves are further divided into two types :
(a) Transverse wave
If the particles of the medium vibrate at right angle to the direction of wave motion or energy
propagation the wave is called transverse wave. These are propagated as crests and troughs. Waves on
strings are always transverse.

Vibration
C C C
Wave Motion
T T T

(A) (B)

(b) Longitudinal wave


If the particles of a medium vibrate in the direction of wave motion, the wave is called longitudinal.
These are propagated as compressions and rarefactions and also known as pressure or compressional
waves. Waves on springs or sound waves in air are example of longitudinal waves.

R R R R
Vibration

Wave Motion
C C C C
(A) (B)

EQUATION OF WAVE MOTION :


Suppose, man holding a stretched string starts snapping his hand at t = 0 and finishes his job at t  t . The
vertical displacement y of the left end of the string is a function of time. It is zero for t < 0, has non-zero
value of 0  t  t and is again zero for t  t . Let us represent this function by f(t). Take the left end of
the string a the origin and take the X-axis along the string towards right. The function f(t) represents the
displacement y of the particle at x = 0 as a function of time y (x = 0, t) = f(t)

Page # 139
AVES ON STRING AND SOUND WAVES PHYSICS PART - IV

The disturbance travels on the string towards right with a constant speed v. Thus, the displacement,
produced at a distance x from the left end at time t was originated at the left end at the time t–x/v. But the
displacement of the left end at time t – x/v is f (t – x/v). Hence,
y (x, t) = y (x = 0, t – x/v)
= f(t – x/v).
The displacement of the particle at x at time t i.e., y (x, t) is generally abbreviated as y and the wave
equation is written as
y = f (t – x/v). …(i)
Equation (i) represents a wave traveling in the positive x-direction with a constant speed v. Such a wave
is called a traveling wave or a progressive wave. The function f is arbitrary and depends on how the
source moves. The time t and the position x must appear in the wave equation in the combination (t – x/
v) only.
For example,

(t  x/ v) (t  x/ v)
y  Asin , y  Ae
T T
etc. are valid wave equations. They represent waves traveling in positive x-direction with constant
(x 2  v 2 t 2 )
speed. The equation y  A sin does not represent a wave traveling in x-direction with a
L2
constant speed.
If a wave travels in negative x-direction with speed v, its general equation may be written as
y = f (t + x/v) …(ii)
The wave traveling in positive x-direction (equation (i)) can also be written as
 vt  x 
y f 
 v 
or, y = g (x – vt), …(iii)
where g is some other function having the following meaning. If we put t = 0 in equation (iii), we get the
displacement of various particles at t = 0 i.e.,
y (x, t = 0) = g (x)
Thus, g(x) represents the shape of the string at t = 0. If the displacement of the different particles at t = 0
is represented by the function g(x), the displacement of the particle at x at time t will be y = g(x – vt).
Similarly, if the wave is traveling along the negative x-direction and the displacement of different particles
at t = 0 is g(x), the displacement of the particle at x at time t will be
y = g(x + vt) …(iv)
Thus, the function f in equation (i) and (ii) represents the displacement of the point x = 0 as time passes
and g in (iii) and (iv) represents the displacement at t = 0 of different particles.
The travelling wave moving with constant speed v towards positive x direction must satisfy the following
wave function condition

2 y 2
2  y
 v …(v)
t 2 x 2

Page # 140
AVES ON STRING AND SOUND WAVES PHYSICS PART - IV

Illustration:

10
A traveling wave pulse is given by y  . In which direction and with what velocity is the pulse
5  (x  2t)2
propagating? What is the amplitude of pulse ?
Solution: A wave pulse is a disturbance localized only in a small part of space at a given instant [as shown in
figure] and its shape does not change during propagation. Though a pulse can be represented by exponential
or trigonometrical functions also, it is usually expressed by the form
y
a t=0
y a
b  (x  t)2 A= b v
Comparing the above with the given pulse we find that
x
10
f ( x   t )  5  ( x  2 t )2 .
i.e., the pulse is travelling along negative x-axis with velocity 2 m/s.
Further amplitude is the maximum value of wave function which will be when
10
(x  2t)2  0 So, A  y max  2 .
5
Illustration:
1
The amplitude of a wave disturbance propagating in the positive x–direction is given by y 
1  x2
1
at t = 0 and y  at t = 2s, where x and y are in meter. Assuming that the shape of the wavee
2  x2  2x
disturbance does not change during the propagation, the speed of the wave is
(A) 0.5 m/s (B) 1 m/s (C) 1.5 m/s (D) 2 m/s.

1
Solution : At t = 0, y  , or x  1  y  x1
1  x2 y
1 1
1y
t = 2s, y  2  x2  2x 
2
At 2 or,  x  1  
1   x  1 y

1y
or, x 1  x2
y

x x2  x 1 1
 Speed of the wave v     0.5m/s
t t 2  t 1 2  0

 Alternative solution: The given pulse is represented by


a
y 2
, where v is the velocity of the wave.
b   x  vt 

1
At t = 0, it is given that y  2
, and
1   x  v  0
1 1 1
At t = 2s, y   
2  x  2x 1   x  1  1   x  vt 2
2 2

 vt = 1, or v  2  1  t  2s 

 v = 0.5 m/s

Page # 141
AVES ON STRING AND SOUND WAVES PHYSICS PART - IV

EQUATION OF A SIMPLE HARMONIC PLANE WAVE :


In case of harmonic wave the displacement of successive particles of the medium is given by a sine wave
or cosine function of position.
The displacement y for different values of x at t = 0 is given by
y  Asinkx …(vi)
where A and k are constants.
Suppose this disturbance is propagating along positive x-direction then
y  A sin k(x  vt) …(vii)
Since the waveform represented by equation (vi) is based on sine function, it would repeat itself at
regular distances. The first repetition would take place when

2
kx  2 or x
k
This distance after which the repetition takes place is called the wavelength and denoted by  . Hence

2 2
 or k
k 
This constant k is called propagation constant or wave number. Now equation (vii) turns into
2 2
y  Asin (x  vt) …(viii) y x y=Asin x
 

2
At t 0 y  A sin
x …(ix) O x
 B D
/2 
Phase difference and path difference : C

2v
The argument of harmonic function ( t  kx) is called phase of the wave and it is constant if the

shape of the wave remains unchanged.
Further if we consider two points at positions x1 and x2 on a wave at a given instant, then

2v 2v
1  t  kx1 and 2  t  kx2
 

so, 2  1  k(x 2  x 1 )

 2 
i.e.,   2 x as k   
  

From this it is clear that if x  ,   2, i.e., a path difference  corresponds to a phase change of
(2) rad..
Relation Between Wavelength and Velocity of Propagation:
Time taken for one complete cycle of wave to pass any point is the time period (T).
This is also the time taken by the disturbance in propagating a distance  .

v  f where f = frequency (Hz)
T

Page # 142
AVES ON STRING AND SOUND WAVES PHYSICS PART - IV

2
  2f = circular frequency (rad/s)
T
Different Forms of Simple Harmonic Wave Equation:
  t x  
y  A sin(t  kx  )  A sin 2     
  T  2  
  x 
 A sin 2f  t     
  v 
where  = phase angle.
Illustration:
Consider the wave y = (5mm) sin [(1 cm) x  (60 s1)t]. Find
(a) The amplitude (b) The wave number,
(c) The wavelength (d) The frequency,
(e) The time period and (f) The wave velocity.
Solution:
Comparing the given equation with wave equation, we find.
(a) Amplitude A = 5 mm (b) Wave number k = 1 cm1
2  60 30
(c) Wavelength    2 cm (d) Frequency    Hz  Hz
k 2 2 
1 
(e) Time period T   s
 30
(f) Wave velocity     60 cm / s .
Illustration:
The displacement of a particle of a string carrying a traveling wave is given by y = (3.0 cm) sin 6.28
(0.5x  50 t),where x is in centimeter and t in second. Find (a) the amplitude, (b) the wavelength (c), the
frequency and (d) the speed of the wave.
Solution: Comparing with the standard wave equation
y  A sin(kx  t)

x t 
 A sin2   
 T
we see that,
amplitude = A = 3.0 cm,
1
wavelength   cm  2.0cm,
0.50
1
and the frequency  v   50 z
T
The speed of the wave is   
= (50 s1) (2.0 cm) = 100 cm/s.

Page # 143
AVES ON STRING AND SOUND WAVES PHYSICS PART - IV

PARTICLE VELOCITY AND WAVE VELOCITY RELATION :


In plane progressive harmonic wave particles of the medium oscillate simple harmonically about their
mean position. Therefore, all the formulae what we have read in SHM apply to the particles here also.
Suppose the wave function is,
y(x,t)  A sin(kx  t) …(i)
Let us differentiate this function partially with respect to t and x.

y(x, t)
  A cos(kx  t) …(ii)
t

y(x, t)
 Ak cos(kx  t) …(iii)
x
Now, these can be written as,

y(x, t)    y(x, t)
  
t  k  x

y(x, t)
Here,  particle velocity vP
t

 y(x, t)
 wave velocity v and 
k x
Thus, vp = – v (slope)
i.e., particle velocity at a given position and time is equal to negative of the product of wave velocity with
slope of the wave at that point at that instant.
The acceleration of the particle is the second partial derivative of y(x, t) with respect to t,

 2 y(x, t)
 ap  2
 2 A sin(kx  t)  2 y(x, t)
t

i.e., the acceleration of the particle equals – 2 times its displacement, which is the result we obtained for
2
SHM. Thus, ap   (displacement).

WAVE FRONT :
A wave front is a surface over which all the points vibrate in same phase.
Spherical wave front :
When the source of light is a point source, the wave front is a sphere
with centre at the source, (figure). S

Cylindrical wave front :


When the source of light is linear, e.g. a slits as shown in figure, all the
points equidistant from the source lie on a cylinder. Therefore, S
the wave front is cylindrical.

Page # 144
AVES ON STRING AND SOUND WAVES PHYSICS PART - IV

Plane Wave front :


When the point source or linear source of light is at very large distance,
a small portion of spherical or cylindrical wave front appears to be plane.
Such a wave front is called a plane wave front. Figure.

INTENSITY OF WAVE :
Intensity due to a point source :
If a point source emits wave uniformly in all directions, the energy at a distance r from the source is
distributed uniformly on a spherical surface of radius r and area S  4r2 . If P is the power emitted by
y
P
the source, the power per unit area at a distance r from the source is . The average power per unit
4r2
area that is incident perpendicular to propagation is called the intensity.
Therefore :
P 1
I or I
4r2 r2
Now, as amplitude A  I , a spherical harmonic emanating form a point source can therefore, be written
as
A
y(r, t)  sin(kr  t)
r
Intensity due to cylindrical source :
As for a cylindrical wave power will be related to intensity by the relation
P  I(area)  I2r

So, I  (1/ r) and I  A 2 , A  (1/ r ) . Further as equation of plane progressive waves is


y  A sin( t  kx) , the equation of a harmonic expanding cylindrical wave will be

y  (B / r )sin(t  kr)

Where B is a constant of proportionality having dimensions [L3/2 ] .

SPEED OF TRANSVERSE WAVE IN A STRETCHED STRING :


Consider a transverse pulse produced in a taut string of linear mass density  . Consider a small segment
of the pulse of length  , forming an arc of a circle of radius R. A force equal in magnitude to the tension
T pulls this segment tangentially at each end.

l

T  R
O

Let us set an observer at the centre of the pulse which moves along with the pulse towards right. For the
observer any small length dl of the string as shown will appear to move backward with a velocity v.

Page # 145
AVES ON STRING AND SOUND WAVES PHYSICS PART - IV

Now the small mass of the string is in a circular path of radius R moving with speed v. Therefore, the
required centripetal force is provided by the only force acting, (neglecting gravity) is the component of
tension along the radius.
The net restoring force on the element is

F  2Tsin( )  T(2)  T
R
The mass of the segment is m  
The acceleration of this element toward the centre of the circle is

v2
a , where v is the velocity of the pulse.
R

v2
Using second law of motion, F  m
R

  v2  T
T  ()   or v  .
R R 

Illustration:
A uniform rope of length 12 m and mass 6 kg hangs vertically from a rigid support. A block of mass 2 kg
is attached to the free end of the rope. A transverse pulse of wavelength 0.06 m is produced at the lower
end of the rope. What is the wavelength of the pulse when it reaches the top of the rope?
Solution: As the rope is heavy, its tension will be different at different points. The tension at the free end will
be (2 kg) g and that at the upper end it will be (8 kg) g.
We have,   v

F 6 kg
or,  v

2 kg
F
or, v  . . . .(i)

The frequency of the wave pulse will be the same everywhere on the rope as it depends only on the
frequency of the source. The mass per unit length is also the same throughout the rope as it is uniform.

F
Thus, by (i) is constant..

(2 kg)g (8 kg)g
Hence,  ,
0.06m 1

where 1 is the wavelength at the top of the rope. This gives 1  0.12 m.
Illustration:
A uniform rope of mass 0.1 kg and length 2.45 m hangs from the ceiling.
(a) Find the speed of transverse wave in the rope at a point 0.5 m distance from the lower end.
(b) Calculate the time taken by a transverse wave to travel the full length of the rope.
Solution : (a) The speed of the transverse wave is given by
T
v , where T  tension and   linear mass density
y

Page # 146
AVES ON STRING AND SOUND WAVES PHYSICS PART - IV

At a distance x from the free end of the rope, the tension


T  xg

xg
 v  xg …(i)

Here x  0.15 m, g  9.8 m/s2

 v  0.5 m  .  9.8 m/s2 


 2.21 m/s
(b) The speed of the wave at a distance x from the free end is
dx
v  xg {from (i)}
dt
1 dx
or, dt 
g x t 
1 dx 
Integrating over the whole length of the rope, t   dt   2
0 g 0 x g
2.45
2  1 s.
9.8
Illustration:
The speed of a transverse wave, going on a wire having a length 50 cm and mass 5.0 g, is 80 m/s. The area
of cross-section of the wire is 1.0 mm2 and its Young’s modulus is 16 × 1011 N/m2. Find the extension of
the wire over its natural length.
Solution : The linear mass density is
5  103 kg kg
 2
 1.0  102
50  10 m m
The wave speed is   F /  .

Thus, the tension is F   2

F/ A
Y
L / L
The extension is, therefore,

FL   64N 0.50m 
L   0.02mm .
AY 1.0  10 m   16  10 N / m 
6 2 11 2

ENERGY TRANSFER IN TRANSVERSE PROGRESSIVE WAVE IN A STRING :


Kinetic energy per unit volume
We can calculate the kinetic energy of unit volume of the string from the wave function. Mass of unit
volume is the density  . Its displacement from equilibrium is the wave function y  A sin(kx  t) . Itss

dy
speed is where x is considered to be fixed. The kinetic energy of unit volume is K is then
dt

2
1 1  dy 
K  ( m)v 2y    
2 2  dt 

Page # 147
AVES ON STRING AND SOUND WAVES PHYSICS PART - IV

Using y  A sin(kx  t) , we obtain


dy
 A cos(kx  t)
dt
So the kinetic energy per unit volume is

1
K  2 A2 cos2(kx  t)
2
Potential energy per unit volume

dy
The potential energy of a segment is the work done in stretching the string and depends on the slope .
dx
The potential energy per unit volume of the string is related to the slope and tension T by (for small
slopes)
2
1  dy 
U  v 2  
2  dx 


where v  wave speed 
k

dy
Using  kA cos(kx  t) , we obtain for the potential energy
dx

1
U  2 A2cos2(kx  t)
2
Which is the same as the kinetic energy. The total energy per unit volume is

E  K  U  2 A2 cos2(kx  t)


1
We see that E varies with time, since the average value of cos2 (kx  t) at any point is , the average
age
2
energy per unit volume (called the energy density u) is

1
u  2 A2 .
2
Power (P)
Power is the instantaneous rate at which energy is transferred along the string (if we consider a transverse
wave on a string).
In unit time, the wave will travel a distance v. If s be the area of cross–section of the string, then volume
of this length would be sv and energy transmitted per unit time (called power) would be,
P = (energy density) (volume)

1
 P  2 A2sv .
2
Intensity (I)
Flow of energy per unit area of cross–section of the string in unit time is known as the intensity of the
wave. Thus,

Page # 148
AVES ON STRING AND SOUND WAVES PHYSICS PART - IV

power P
I 
area of cross section s
1
or I  2 A2v .
2
Illustration:
A stretched string is forced to transmit transverse waves by means of an oscillator coupled to one end.
The string has a diameter of 4 mm. The amplitude of the oscillation is 10–4 m and the frequency is 10 Hz.
Tension in the string is 100 N and mass density of wire is 4.2 × 103 kg/m3. Find
(a) the equation of the waves along the string
(b) the energy per unit volume of the wave
(c) the average energy flow per unit time across any section of the string and
(d) power required to drive the oscillator.
Solution : (a) Speed of transverse wave on the string is,
T
v (as   S )
S
Substituting the values, we have
100
v

(4.2  103 )   (4.0  103 )2
4
= 43.53 m/s
rad rad
  2f  20  62.83
s s

k  1.44m 1
v
 Equation of the waves along the string,
y (x, t) = A sin (kx – t)
  rad  
 (104 m)sin (1.44m 1 )x   62.83 t .
  s  
(b) Energy per unit volume of the string,

1 2 2
u = energy density   A
2
Substituting the values, we have

1
u   (4.2 103 )(62.83)2 (104 )2
 2

 8.29  10 2 J / m3 .
(c) Average energy flow per unit time,
P = power
1 
  2 A2  (sv)  (u)(sv)
 2 
Substituting the values, we have

Page # 149
AVES ON STRING AND SOUND WAVES PHYSICS PART - IV


P  (8.29  102 ) (4.0  103 )2 (43.53)
4
 4.53 105 J / s
(d) Power required to drive the oscillator is obviously 4.53 × 10–5 watt.

SPEED OF LONGITUDINAL WAVE IN FLUID :


Sound wave in air is a longitudinal wave. As a sound wave passes through air, potential energy is associated
with periodic compressions and expansions of small volume elements of the air. The property that
determines the extent to which an element of the medium changes its volume as the pressure applied to
it is increased or decreased is the bulk modulus B.
p
B
V / V

V
where is the fractional change in volume produced by a change in pressure p .
V
Suppose air of density  is filled inside a tube of cross-sectional area A under a pressure p. Initially the air
is at rest.
At t  0 , the piston at left end of the tube (as shown in the figure) is Air
pA , p
set in motion toward the right with a speed u. After a time interval
t , all portions of the air at the left of section 1 are moving with vt
ut (I)
speed u whereas all portions at the right of the section are still at rest.
The boundary between the moving and the stationary portions travels (p+p)A pA
to the right with v, the speed of the elastic wave (or sound wave). In
the time interval t , the piston has moved u t and the elastic ic
disturbance has traveled a distance v t .

The mass of air that has attained a velocity u in a time t is (v t)A . Therefore, the momentum imparted
ed
is [v( t)A]u and the net impulse =  pA  t

Thus, impulse = change in momentum


(pA)t  [v( t)A]u

or p  vu …(xii)
p
Since B
V / V

 V 
 p  B  
 V 

where V  Av t

and V  Au t

V Aut u
  
V Avt v
u
Thus, p  B …(xiii)
v
B
From (xii) and (xiii) v  .

Page # 150
AVES ON STRING AND SOUND WAVES PHYSICS PART - IV

Velocity of Sound in an Ideal Gas:


Air can be assumed to be adiabatic to study motion of sound in air . In the case of an ideal gas, the relation
between pressure p and volume V during an adiabatic process is given by

pv   constant..

where  is the ratio of the heat capacity at constant pressure to that at constant volume.
After differentiating, we get
dp 
V  pV  1  0
dV
dp
Since B  V  p
dV
p
 v

P
(Laplace correction in contrast to Newton’s formula v  )

p RT
Using the gas equation  where M is the molar mass.
 M
RT
Thus, v (T = Temperature in Kelvin)
M
From practical standpoint it is easier to measure pressure variation in a sound wave than the
displacements, so it is worthwhile to develop a relation between the two. Let p be the instantaneous
pressure fluctuation at any point, that is, the amount by which the pressure differs from normal
atmosphere pressure. If the displacements of two neighboring points x and x + x are the same, the gas
between these points is neither compressed nor rarefied, there is no volume change, and consequently p
= 0. Only when y varies from one point to a neighboring point there is a change of volume and therefore
of pressure.
 y 
The fractional volume change V / V in an element near point x turns out to be   , which is the ratee
 x 
of change of y with x as we go from one point to the neighboring point. To see why this is so, we note that
V / V is proportional to change in length of an element which has length x when no wave disturbance
is present, divided by x . The change in length is the value of y at the point x + x , minus the value at the
point x. If x is very small, this is approximately multiplied by the derivative of y with respect to x, thus

y
y(x  x,t)  y(x,t)  x
x
V y(x  x, t)  y(x,t) y
  …(xi)
V x x
Now from the definition of the bulk modulus B,
V
p  B , and we find
V
y
p  B
x
Now y  A sin( t  kx)

 p  BAk cos(t  kx)

Page # 151
AVES ON STRING AND SOUND WAVES PHYSICS PART - IV

Maximum amount by which the pressure differs from atmospheric, that is, the maximum value of p, is
called the pressure amplitude, denoted P.
 P  BkA
Displacement wave and Pressure wave :
A longitudinal wave in a fluid (liquid or gas) can be described either in terms of the longitudinal
displacement suffered by the particles of the medium or in terms of the excess pressure generated due to
the compression or rarefaction. Let us see how the two representations are related to each other.

s s+s

x x+x

Consider a wave going in the x-direction in a fluid. Suppose that at a time t , the particle at the undisturbed
position x suffers a displacement s in the x-direction. The wave can then be described by the equation

s  s0 sin (t  x / v) …(xv)

Consider the element of the material which is contained within x and x  x (figure)in the undisturbed
state. Considering a cross-sectional area A , the volume of the element in the undisturbed state is Ax
and its mass is  Ax . As the wave passes, the ends at x and x  x are displaced by amountss s and
s  s according to equation (xv) above. The increase in volume of this element at time t is

s
V  A s = A .x
x

 As0 ( / v)cos(t  x / v)x ,

where s has been obtained by differentiating equation (xv) with respect to x. The element is, therefore,
under a volume strain.
V  As0cos (t  x / v)x
V

vAx
V  Ax
s 
 0 cos (t  x / v) .
v
The corresponding stress i.e., the excess pressure developed in the element at x at time t is,

 V 
p  B ,
 V 
where B is the bulk modulus of the material. Thus,
s0 
pB cos (t  x / v) ….(xvi)
v
Comparing with (xv), we see that the pressure amplitude p0 and the displacement amplitude s0 aree
related as
B
p0  s0  Bks0 ,
v
where k is the wave number. Also, we see from (xv) and (xvi) that the pressure wave differs in phase by
y
 / 2 from the displacement wave. The pressure maxima occur where the displacement is zero and
displacement maxima occur where the pressure is at its normal level.

Page # 152
AVES ON STRING AND SOUND WAVES PHYSICS PART - IV

The fact that, displacement is zero where the pressure-change is maximum and vice versa, puts the two
descriptions on different footings. The human ear or an electronic detector responds to the change in
pressure and not to the displacement in a straight forward way. Suppose two audio speakers are driven
by the same amplifier and are placed facing each other. A detector is placed midway between them.

The displacement of the air particles near the detector will be zero as the two sources drive these particles
in opposite directions. However, both send compression waves and rarefaction waves together. As a
result, pressure increases at the detector simultaneously due to both the sources. Accordingly, the pressure
amplitude will be doubled, although the displacement remains zero here. A detector detects maximum
intensity in such a condition. Thus, the description in terms of pressure wave is more appropriate than
the description in terms of the displacement wave as far as sound properties are concerned.
FACTORS AFFECTING SPEED OF SOUND IN AIR :
We have stated that for an ideal gas, the pressure, volume and temperature of a given mass satisfy

PV
 constant..
T
(a) As the density of a given mass is inversely proportional to its volume, the above equation may also
be written as

P
 cT ,

where c is a constant. The speed of sound is

P
v  cT …(xvii)

Thus, if pressure is changed but the temperature is kept constant, the density varies proportionally and
P /  remains constant. The speed of sound is not affected by the change in pressure provided the
temperature is kept constant.
(b) If the temperature of air is changed then the speed of sound is also changed.
From equation (xvii),

v T .

At STP, the temperature is 0º C or 273 K. If the speed of sound at 0º C is v 0 , its value at the room
temperature T (in Kelvin) will satisfy
v T 273  t
 
v0 273 273
where t is the temperature in ºC. This may be approximated as
1/2
v  t  t
 1   1
v 0  273  546
 t 
or, v  v 0  1  .
 546 

Page # 153
AVES ON STRING AND SOUND WAVES PHYSICS PART - IV

(c) The density of water vapour is less than dry air at the same pressure. Thus, the density of moist air is
less than that of dry air. As a result, the speed of sound increases with increasing humidity.

Illustration:
A wave pulse starts propagating in +x–direction along a non–uniform wire of length 10 m with mass per
unit length given m  m0  x and under a tension of 100 N. Find the time taken by the pulse to travell
from the lighter end (x = 0) to the heavier end.

(m0  102 kg / m and   9  103 kg / m2 ) :

Solution : Velocity of transverse wave in a string,

dx T T
v  
dt m m0  x

 m0  x dx  T dt

Integrating within proper limits,


L t
 m0   x dx  T  dt
0 0

L
 2(m0  x)3/2 
   Tt
 3 0
2
or, [(m0  L)3/2  m03/2 ]  T t
3
2
 t [(m0  L)3/2  m3/20 ]
3 T
2
 3
[(102  9  103  10)3/2  (102 )3/2 ]
3  9  10  10
2
 2
[(102  9  102 )3/2  103 ]  2.335 s .
27  10
Illustration:
(a) Find the speed of sound in mixture of 1 mol of helium and 2 mol of oxygen at 27ºC.
(b) If the temperature is raised by 1 K from 300 K, find the percentage change in the speed of sound in
the gaseous mixture. (R = 8.31 J/mol K).
Solution : (a) In terms of temperature, the speed of sound is given by

RT
v …(i)
M
Now here as 2 mole of helium is mixed with 2 mole of oxygen.
n1M1  n2M2 1  4  2  32 68
Mmix     103 kg / mol …(ii)
n1  n 2 12 3
Further as helium is monoatomic [CV = (3/2) R] while oxygen is diatomic (CV = (5/2) R), so for mixture
n1CV1  n 2C V2 1  (3/ 2)R  2(5/ 2)R 13
(CV )mix    R
n1  n 2 12 6
13 19
(CV )mix  (CV )mix  R  RR  R [as CP  CV  R]
6 6
Page # 154
AVES ON STRING AND SOUND WAVES PHYSICS PART - IV

CP 19 6 19
 (  )mix   R  …(iii)
CV 6 13R 13
So, substituting the values of M and  from equation (ii) and (iii) in (i) with T = 300 K and R = 8.31 J/mol
K, we get,
19 9.31  300
v   400.9 m / s
13 (68 /3  103 )

(b) As v  ( RT / M) , for a given gas,

v 1 T 1 1
    100  0.167%.
v 2 T 2 300
SPEED OF LONGITUDINAL WAVE IN DIFFERENT MEDIUM :
Sound waves can travel in solids just like they can travel in fluids. The speed of longitudinal sound waves
in a solid rod can be shown to be

v  Y / ,

where Y is the Young’s modulus of the solid and  its density. For extended solids, the speed is a moree
complicated function of bulk modulus and shear modulus. Table gives the speed of sound in some common
materials.

Medium Speed m/s Medium Speed m/s


Air (dry 0ºC) 332 Copper 3810
Hydrogen 1330 Aluminium 5000
Water 1486 Steel 5200

ENERGY TRANSFER IN LONGITUDINAL PROGRESSIVE WAVE :


As a wave travels in a medium, energy is transported from one part of the space to another part. The
intensity of a sound wave is defined as the average energy crossing a unit cross-sectional area
perpendicular to the direction of propagation of the wave in unit time. It may also be stated as the
average power transmitted across a unit cross-sectional area perpendicular to the direction of
propagation.
The loudness of sound that we feel is mainly related to the intensity of sound. It also depends on the
frequency to some extent.
Consider again a sound wave traveling along the x-direction. Let the equations for the displacement of
the particles and the excess pressure developed by the wave be given by

s  s0 sin (t  x / v)
and p  p0 cos (t  x / v) …(xviii)

Bs0
where p0  .
v
Consider a cross-section of area A perpendicular to the x-direction. The medium to the left to it exerts a
force pA on the medium to the right along the X-axis. The points of application of this force movee

Page # 155
AVES ON STRING AND SOUND WAVES PHYSICS PART - IV

s
longitudinally, that is along the force, with a speed . Thus, the power W , transmitted by the wave
ve
t
from left to right across the cross-section considered, is

s
Power = W  (pA) .
t

By (xviii), W  Ap0 cos (t  x / v)s0 cos (t  x / v)

A2s02B
 cos2 (t  x / v) .
v

The average of cos2 (t  x / v) over a complete cycle or over a long time is 1/2. The intensity I , which
is equal to the average power transmitted across unit cross-sectional area
Thus

1 2s20B 22B 2 2
Intensity = I   s0  .
2 v v
p20 v
I
2B
As B  v 2 , the intensity can also be written as

v 2 p20
I p0 
2v 2 2v

We see that the intensity is proportional to the square of the pressure amplitude p0 .

LOUDNESS, PITCH AND QUALITY OF SOUND :


Loudness :
Human ear is sensitive for extremely large range of intensity. So a logarithmic rather than an arithmetic
scale is convenient. Accordingly, intensity level  of a sound wave is defined by the equation

 I
  10log   decibell
 I0 

where I0  10 12 W / m2 is the reference or threshold intensity level to which any intensity I is compared..

Pitch :
It is the sensation received by the ear due to frequency and is the characteristic which distinguishes a
shrill (or sharp) sound from a grave (or flat) sound. As pitch depends on frequency, higher the frequency
higher will be pitch and shriller will be the sound.

Quality :
y
It is the sensation received by the ear due to waveform. Two a t
T
sounds of same intensity and frequency as shown in figure will
y
produce different sensation on the ear if their waveforms are a t
different. Now as waveform depends on overtones present, T
quality of sound depends on number of overtones, i.e., harmonics y
a
present and their relative intensities. t
T

Page # 156
AVES ON STRING AND SOUND WAVES PHYSICS PART - IV

Illustration:
The pressure amplitude in a sound wave from a radio receiver is 2.0  102 N/m2 and the intensity at a
point is 5.0  107 W/m2. If by turning the “volume” knob the pressure amplitude is increased to 2.5 102
N/m2 evaluate the intensity.
Solution: The intensity is proportional to the square of the pressure amplitude.
2
I  p0 
Thus,   
I  p0 
2 2
 p   2.5  7
or I   0  I     5.0  10 W/m2
p
 0  2.0 

 7.8  107 W/m2.


Illustration:
If the intensity is increased by a factor of 20, by how many decibels is the sound level increased?

Solution: Let the initial intensity be I and the sound level be 1 . When the intensity is increased to 20 I, the
level increase to 2 .

Then 1 = 10 log (I/I0) And 2 = 10 log (20 I/I0).

Thus, 2  1 = 10 log (20 I/I) = 10 log 20 = 13 dB.


SUPERPOSITION OF WAVES :
Two or more waves can traverse the same space independently of one another. Thus the displacement of
any particle in the medium at any given time is simply the vector sum of displacements that the individual
waves would give it. This process of the vector addition of the displacement of a particle is called
superposition.

CONCEPT OF COHERENT SOURCE :


The source of light, which emit continuous light waves of the same wavelength, same frequency and in
same phase or having a constant phase difference are called coherent sources.
INTERFERENCE :
Suppose two identical sources send sinusoidal waves of same angular frequency  propagates in
positive x-direction. Also, the wave velocity and hence, the wave number k is same for the two waves.
One source may be started a little later than the other or the two sources may be situated at different
points. The two waves arriving at a point then differ in phase. Let the amplitudes of the two waves be A1
and A 2 and the two waves differ in phase by an angle  . Their equations may be written as
y 1  A 1 sin(kx  t )
and y 2  A 2 sin(kx  t   ) .
According to the principle of superposition, the resultant wave is represented by
y  y 1  y 2  A1 sin(kx  t)  A 2 sin(kx  t  )
 A 1 sin(kx  t)  A 2 sin(kx  t)cos   A 2 cos(kx  t)sin 
 sin(kx  t)(A1  A 2 cos )  cos(kx  t )(A 2 sin ) .
We can evaluate it using the method described to combine two simple harmonic motions.

Page # 157
AVES ON STRING AND SOUND WAVES PHYSICS PART - IV

If we write

A 1  A 2 cos   A cos  …(x)

and A 2 sin   A sin  …(xi)


we get y  A[sin(kx  t)cos   cos(kx  t)sin ]
 A sin(kx  t   ) .
Thus, the resultant is indeed a sine wave to amplitude A with a phase difference  with the first wave. By (x)
and (xi),

A2  A2 cos2   A2 sin2 
A
2 2
A2
 (A1  A2 cos )  (A2 sin )
 
 A21  A22  2A1 A2 cos  A1

or, A  A 21  A 22  2A1 A 2 cos  ….(A)


Asin  A2 sin 
Also, tan   
A cos  A1  A2 cos  . …(B)

These relations may be remembered by using a geometrical model. We draw a vector of length A1 to
o
represent y 1  A 1 sin(kx  t ) and another vector of length A 2 at an anglee  with the first one to represent
y 2  A 2 sin(kx  t   ) . The resultant of the two vectors then represents the resultant wavee
y  A sin(kx  t  )
Constructive and Destructive Interference:
We see from equation (A) that the resultant amplitude A is maximum when cos   1 , or   2n and
is minimum when cos   1 , or   (2n  1) , where n is an integer. In the first case, the amplitude is
A1  A 2 and in the second case, it is | A1  A 2 | . The two cases are called constructive and destructive
ve
interferences respectively. The conditions may be written as,
constructive interference :   2n

destructive interference :   (2n  1) .


Illustration:

Two coherent sound sources are at distances x1  0.2 m and x = 0.27m from a point. Calculate the
intensity of the resultant wave at the at point if the frequency of each wave is f = 800 Hz and velocity of
wave in the medium is v  224 m / s . The intensity of each wave is I0  60W / m2 .

Solution: Path difference, p  x 2  x1  0.27  0.2  0.07 m

2  2f  2(800)(0.07) 
 p  p 
  v  224 2

I  I1  I2  2 I1I2 cos 

Page # 158
AVES ON STRING AND SOUND WAVES PHYSICS PART - IV

or I  I0  I0  2I0 cos(  / 2)

 2I0  2(60)  120 W/m2

STANDING WAVES :
A standing wave is formed when two identical waves traveling in the opposite directions along
the same line, interfere.
On the path of the stationary wave, there are points where the amplitude is zero, they are known as
NODES. On the other hand there are points where the amplitude is maximum, they are known as
ANTINODES.

The distance between two consecutive nodes or two consecutive antinodes is .
2

The distance between a node and the next antinode is .
4
Consider two waves of the same frequency, speed and amplitude, which are traveling in opposite directions
along a string. Two such waves may be represented by the equations

y1  a sin(kx  t) and

y 2  a sin(kx  t)
Hence the resultant may be written as

y  y 1  y 2  a sin(kx  t )  a sin(kx  t )

y  2asin kx cos t
This is the equation of a standing wave.
Illustration:

The displacement y of a particle executing periodic motion is given by y  4cos2  t  sin 1000t  . This
expression may be considered to be a result of the superposition of waves :
(A) two (B) three (C) four (D) five.

Solution : Given : y  4cos2  t  sin 1000t 

 21  cos2t  sin 1000t 

= 2sin1000t  2sin1000t  cos2t

 2sin1000t  sin1002t  sin 998t


Thus the periodic motion consists of three components.

STANDING WAVES IN STRING :

N ANANA
N A A N N N

L L
(b) (c)

Page # 159
AVES ON STRING AND SOUND WAVES PHYSICS PART - IV

A string of length L is stretched between two points. When the string is set into vibrations, a transverse
progressive wave begins to travel along the string. It is reflected at the fixed end. The incident and the
reflected waves interfere to produce a stationary transverse wave in which the ends are always nodes.
(a) In the simplest form, the string vibrates in one loop in which the ends are the nodes and the centre is
the antinode. This mode of vibration is known as the fundamental mode and the frequency of vibration
is known as the fundamental frequency or first harmonic.

1
L  1  2L
2

If f1 is the fundamental frequency of vibration, then the velocity of transverse waves is given as,
v  1f1 or f1  v / 2L

 v  2Lf1 …(xix)
(b) The same string under the same conditions may also vibrate in two loops, such that the centre is also
the node.

2
 L2  2  L
2

If f2 is the frequency of vibrations, then the velocity of transverse waves is given as,

v  2 f2  v  Lf2 or f2  v / L = 2f1 …(xx)

The frequency f2 is known as second harmonic or first overtone.


(c) The same string under the same conditions may also vibrate in three segments.
3 2
 L 3  3  L
2 3
If f3 is the frequency in this mode of vibration, then,

2
v  3f3  v  Lf3 or f3  3v /2L = 3f1 …(xxi)
3
The frequency f3 is known as the third harmonic or second overtone. Thus a stretched string in addition
to the fundamental node, also vibrates with frequencies which are integral multiples of the fundamental
frequencies. These frequencies are known as harmonics.
The velocity of transverse wave in a stretched string is given as
T
v where T = tension in the string.

 = linear density or mass per unit length of string.
If the string fixed at two end, vibrates in its fundamental mode, then
1 T
v  2Lf  f 
2L 
 = volume of unit length × density
y
D2
 r2  1      where D = diameter of the wire,  = density.
y.
4

Page # 160
AVES ON STRING AND SOUND WAVES PHYSICS PART - IV

Sonometer Experiment :
A C1 C C2 B
A typical design of a sonometer is shown in figure. One has a D2
D1
wooden box, also called the sound box, on which two bridges D
A and B are fixed at the ends. A metal wire C is welded with
the bridges and is kept tight.

The waves can be produced on the wire by vibrating a tuning fork (by holding its stem and gently hitting
a prong on a rubber pad) and pressing its stem on the platform on the sound box of the sonometer. The
simple harmonic disturbance is transmitted to the wire through the bridges. The frequency of vibration
is same as that of the tuning fork. If this frequency happens to be equal to one of the natural frequencies
of the wire, standing waves with large amplitudes are set up on it. The tuning fork is then said to be in
“resonance” or in “unison” with the wire.
Laws of Transverse Vibrations of A String:
1 T
The fundamental frequency of vibration of a stretched string fixed at both ends is given by   .
2 
From this equation, one can immediately write the following statements known as “Laws of transverse
vibrations of a string”
(a) Law of length – The fundamental frequency of vibration of a string (fixed at both ends) is inversely
proportional to the length of the string provided its tension and its mass per unit length remain the same.
  1/ L if T and  are constants.
s.
(b) Law of tension – The fundamental frequency of a string is proportional to the square root of its
tension provided its length and the mass per unit length remain the same.

  T if L and  are constants.


s.
(c) Law of mass – The fundamental frequency of a string is inversely proportional to the square root of
the linear mass density, i.e., mass per unit length provided the length and the tension remain the same.

1
 if L and T are constants.
s.

Illustration:
A sonometer wire 100 cm in length has a fundamental frequency of 330 Hz. Find
(a) the velocity of propagation of transverse waves along the wire and
(b) the wavelength of the resulting sound in air if velocity of sound in air is 330 m/s.

Solution: (a) In case of transverse vibration of string for fundamental mode :

L  ( /2), i.e.,   2L  2  1  2 m

i.e., the wavelength of transverse waves propagating on string is 2 m. Now as the frequency of wire is
given to be 330 Hz, so from   f velocity of transverse waves along the wire will be

 wire  330  2  660 m/s

i.e., for transverse mechanical waves propagating along the wire,

f = 330Hz,   2m and   660 m/s

Page # 161
AVES ON STRING AND SOUND WAVES PHYSICS PART - IV

(b) Here vibrating wire will act as source and produce sound, i.e., longitudinal waves in air. Now as
frequency does not change with change in medium so f  330 Hz and as velocity in air is given to be =
330 m/s so from   f

 air  (air / f )  (330/330)  1 m

i.e., for sound (longitudinal mechanical waves) in air produced by vibrations of wire (body),

f  330 Hz,   1m and   330 m/s.

STATIONARY WAVES IN AIR COLUMN :


Open pipe: A A A
3/4
If both ends of a pipe are open and a system of air is directed 2/4
against an edge, standing longitudinal waves can be set up in N 2/2
L
the tube. The open ends are a displacement antinodes and 3/2
pressure nodes 1/2
(a) For fundamental mode of vibrations,
(a) (b) (c)

L  1  1  2L
2
v
v  1f1  v  2Lf1 or f1  …(xxii)
2L
(b) For the second harmonic or first overtone,

L   2 or 2  L

v 2v
v   2 f2  v  Lf 2 or f2    2f …(xxiii)
L 2L
(c) For the third harmonic or second overtone,
3 2
L  3  3  L
2 3
2 3
v  3f3  v  Lf3 or f2  …(xxiv)
3 2L
From (xxii), (xxiii) and (xxiv) we get, f1 : f2 :f3 :..............  1:2:3:..............

i.e. for a cylindrical tube, open at both ends, the harmonics excitable in the tube are all integral multiples
of its fundamental.
2L
 In the general case,   where n  1,2.......
n
v nv
Frequency   where n  1,2......
 2L
A
A 3 N A
Closed pipe: 2 4
If one end of a pipe is closed the reflected wave is 2
4 N 3 A
180º out of phase with the incoming wave. Thus L
4 2 N
the displacement of the small volume elements at 1
3
A
the closed end must always be zero. Hence the 2
2
A
N N
closed end must be a displacement node and N
pressure antinode (a) (b) (c)
Page # 162
AVES ON STRING AND SOUND WAVES PHYSICS PART - IV

(a) This represents the fundamental mode of vibration,


1
L  1  4L
4
if f1 is the fundamental frequency, then the velocity of sound waves is given as,
v
v  1 f1  v  4Lf1 or f1  …(xxv)
4L
(b) This is the third harmonic or first overtone.
2 4
L  3  2  L
4 3
4 v
v  2f2  v  Lf2 or f2  2  3f , …(xxvi)
3 4L
(c) This is the fifth harmonic or seconds overtone.
3 4
L  5 3  L
4 5
4 5V
v  3f3  v  Lf3 or f3   5f …(xxvii)
5 4L

From (xxv), (xxvi) and (xxvii) we get, f1 : f2 : f3 : ......  1: 3 : 5 : ........

4l
In general,   where n = 0, 1, 2 ……
(2n  1)
Velocity of sound = v

(2n  1)v
Frequency  where n = 0, 1, 2 ……
4L

Resonance Column Method :


Figure shows schematically the diagram of a simple apparatus used in
laboratories to measure the speed of sound in air. A long cylindrical glass
tube (say about 1m) is fixed on a vertical wooden frame. It is also called a
resonance tube. A rubber tube connects the lower end of this glass tube to a
vessel which can slide vertically on the same wooden frame. A meter scale is
fitted parallel to and closed to the glass tube.
The vessel contains water which also goes in the resonance tube through the rubber tube. The level of
the water in the resonance tube is same as that in the vessel. Thus, by sliding the vessel up and down, one
can change the water level in the resonance tube. A tuning fork (frequency > 256 Hz if the tube is 1 m
long) is vibrated by hitting it on a rubber pad and is held near the open end of the tube in such a way that
the prongs vibrate parallel to the length of the tube. Longitudinal waves are then sent in the tube.
The water level in the tube is initially kept high. The tuning fork is vibrated and kept closed to the open
end, and the loudness of sound coming from the tube is estimated. The vessel is brought down a little to
decrease the water level in the resonance tube. The tuning fork is again vibrated, kept close to the open
end and the loudness of the sound coming from the water level corresponding to the maximum loudness
is located. Fine adjustments of water level are made to locate accurately the level corresponding to the
maximum loudness. The length of the air column vibrates in resonance with the tuning fork. The minimum
length of the air column for which the resonance takes place corresponds to the fundamental mode of

Page # 163
AVES ON STRING AND SOUND WAVES PHYSICS PART - IV

vibration. A pressure antinode is formed at the water surface (which is the closed end of the air column)
and a pressure node is formed near the open end. In fact, the node is formed slightly above the open end
(end correction) because of the air-pressure from outside.
Thus, for the first resonance the length l1 of the air column in the resonance tube is given by


1  d  …(1)
4
where d is the end correction.
N - Pressure node,
A - Pressure antinode.
The length of the air column is increased to a little less than three times of l. The water level is adjusted
so that the loudness of the sound coming from the tube becomes maximum again. The length of the air
column is noted on the scale. In this second resonance the air column vibrates in the first overtone.
There is one node and one antinode in between the ends of the column. The length l2 of the column is
given by

I2  d  3 / 4 …… (2)
By (1) and (2), we get

  2   1    / 2 or,   2   2  1 
The frequency of the wave is same as the frequency of the tuning fork. Thus, the speed of sound in the
air of the laboratory is

  v  2   2  1  v ….. (3)

Illustration:
An air column is constructed by fitting a movable piston in a long cylindrical tube. Longitudinal waves
are sent in the tube by a tuning fork of frequency 416 Hz. How far from the open end should the piston
be so that the air column in the tube may be vibrate in its first overtone ? Speed of sound in air is 333 ms.
Solution : The piston provides the closed end of the column and an anti-node of pressure is formed there. At
the open end, a pressure node is formed. In the first overtone there is one more node and an anitnode in
the column as shown in the figure. The length of the tube should then be 3  / 4 . H

 333m / s A
The wavelength is     0.800m .
v 416s 1
l
Thus, the length of the tube is
N
3 3 0.800m
  60.0cm .
4 4 A
Illustration:
The third overtone of a closed pipe is observed to be in unision with the second overtone of an open
pipe. The ratio of the lengths of the pipes is
(A) 3 / 2 (B) 5 / 3 (C) 7/4 (D) 7/6.

Page # 164
AVES ON STRING AND SOUND WAVES PHYSICS PART - IV

v
Solution : Frequency of the fundamental note in closed pipe is 4  n1 . Only the odd harmonics,
1

3n1 (first overtone)


5n1 (second overtone)
7n1 (third overtone)
_______________________
are present
v
Frequency of the fundamental note in open pipe is 2  n2 . All the harmonics
2

2n2 (second harmonic or first overtone)

3n2 (third harmonic or second overtone)

4n2 ( fourth harmonic or third overtone)


___________________________________
are present
Given that : 7n1  3n2

v v 1 7
or 7 3  
4 1 2 2 2 6

ENERGY IN STANDING WAVE :


For a standing wave amplitude at a distance x from
x = 0 is given by A = a sin kx
Total mechanical energy at x of length dx is

1
dE  (dm)A22
2
1
 (dx)(asin kx)2(2f )2
2
or dE  2 2f 2a 2 sin 2 kxdx …(1)

T
2 
v 
Here, f  2   2 and k 
 (4l ) l
Substituting these values in Equation (1) and integrating it from x = 0 to x = l, we get total energy between
conservative nodes
2a2T
E =
4l

REFLECTION AND TRANSMISSION OF WAVE :


When sound waves are incident on a boundary between two media, a part of energy of incident waves
returns back into the initial medium (reflection) while the remaining is partly absorbed and partly
transmitted into the second medium (refraction). In case of reflection and refraction of sound :

Page # 165
AVES ON STRING AND SOUND WAVES PHYSICS PART - IV

Incident
Wave
i
r t
Reflected
wave Transmitted
wave

(1) The frequency of the wave remains unchanged, i.e.,

i  r  t  

(2) The incident ray, reflected ray, normal and refracted ray are always in the same plane.
(3) In case of reflection,
Angle of incidence i = Angle of reflection r
(4) In case of refraction,

sini v i

sin t v t [vi = velocity in the first medium, vt = velocity in the second medium]

(5) In case of reflection from a denser medium or rigid support or fixed end there is inversion
of reflected displacement wave, i.e., if the incident waves is given by

y  Ai sin(t  kx)
the reflected wave will be
y   A r sin( t  kx)  A r sin( t  kx  )
i.e., the phase of displacement wave changes by  in case reflection from a denser medium, fixed
end or rigid support, while in case of transmission no change . The transmitted wave is never inverted.
This is shown in figure.

Rarer Denser Denser Rarer


Incident wave
Incident wave
yi = Ai sin (t - k 1 x) y = A sin (t – k x)
yi = A i sin (t + k1x)
r r 2 y=Asin(t–kx)
t t  2

Transmitted Transmitted
yr = –A rsin (t + k 1x) wave yr = A r sin (t + k1x) wave
Reflected wave Reflected wave

(A) (B)

Illustration:
A wave y i  a i sin (x / C1  t ) reached the boundary of media 1 and 2, at which it is partly reflected into
o
medium 1 and partly transmitted into medium 2. Call these waves y r  a r sin (x / C1  t) and
y t  a t sin (x / C 2  t) , respectively. Assuming that at the boundary the displacement arising from the

transmitted wave, show that ai  ar  a t . Further, assuming that the slope of the displacement wave at
the boundary of medium 1 is equal to the slope of the displacement wave at the boundary of medium 2,
show that

Page # 166
AVES ON STRING AND SOUND WAVES PHYSICS PART - IV

ar C1  C2

ai C1  C2

Solution: The wave in medium 1 is given by

 x   x 
y 1  y i  y r  a i sin    t   a r sin    t 
 C1   C1 

 x 
and that in medium 2 is y 2  y t  a t sin   C  t 
 2 
At the boundary x  0, y1  y 2

 a i sin(t)  ar sin t  a t sin(t)

or ai  at  ar ( sin t  0)

 dy 1    x    x 
 dx   a i C cos   C  t   a r C cos   C  t 
  2  1  1  1 
dy 1   x 
 a t cos   t 
dx C2  C2 
dy 1 dy 2
At the boundary x  0 
dx dx
ai  a a
 cos t  r cos t  t cos t
C1 C1 C2
ai  ar a t a i  a r
or   ( ai  ar  a t )
C1 C2 C2
ar C1  C2
or 
ai C1  C2 .
BEATS :
When two interacting waves have slightly different frequencies the resultant disturbance at any point
due to the superposition periodically fluctuates causing waxing and waning in the resultant intensity.
The waxing and waning in the resultant intensity of two superposed waves of slightly different frequency
are known as beats.
Let the displacement produced at a point by one wave be

y 1  A sin(2f1 t  1 )
And the displacement produced at the point produced by the other wave of equal amplitude as

y 2  A sin(2f2t  2 )
By the principle of superposition, the resultant displacement is

y  y 1  y 2  A sin(2f1t  1 )  Asin(2f 2 t  2 )

  f  f         f  f      
y  2A sin 2  1 2  t   1 2   cos 2  1 2  t   1 2  
  2   2    2   2 

  f  f      
Y  R sin 2  1 2  t   1 2  
  2   2 
Page # 167
AVES ON STRING AND SOUND WAVES PHYSICS PART - IV

  f 1  f2   1  2  
where, R = 2 A cos 2  2  t   2  
    

The time for one beat is the time between consecutive maximum or minima.
First maxima would occur when
 f f 
cos2  1 2  t  1
 2 
 f 1  f2 
Then 2  t 0
 2 
 t0
For second maxima would occur when
 f f 
cos2  1 2  t  1
 2 
 f 1  f2 
Then 2  t  
 2 
1
or, t  f  f
1 2

 1  1
The time for one beat =  f  f  0   f  f
 1 2  1 2
1
Similarly it may also be shown that time between two consecutive minima is f  f .
1 2
Hence, frequency of beat i.e. number of beats in one second or
Beat frequency = f1  f2 .
Illustration:
A tuning fork of frequency 256 Hz and an open organ pipe of slightly lower frequency are at 17ºC. When
sounded together, they produce 4 beats per second. On altering the temperature of the air in the pipe, it
is observed that the number of beats per second first diminishes to zero and then increases again to 4. By
how much and in what direction has the temperature of the air in the pipe been altered?

c17
Solution: n where l = length of the pipe
2l

c17 c
 256   4 or 17  252
2l 2l
Since beats decrease first and then increase to 4, the frequency of the pipe increases. This can
happen only if the temperature increases.
Let t be the final temperature, in Celsius,

ct c
Now  256  4 or t  260
2l 2l
c t 260
Dividing c  252
17

273  t 260
or  ( c  T ) or t  308.7  273  35.7 ºC
273  17 252
Page # 168
AVES ON STRING AND SOUND WAVES PHYSICS PART - IV

 rise in temperature = 35.7 – 17 = 18.7ºC.


Illustration:
Two tuning fork A and B sounded together give 6 beats per second. With an air resonance tube closed
at one end, the two forks give resonance when the two air columns are 24 cm and 25 cm respectively.
Calculate the frequencies of forks.

Solution: Let the frequency of the first fork be f1 and that of second be f2 .
We then have,
v v
f1  and f2 
4  24 4  25
We also see that f1  f2

 f1  f2  6 …(i)
f1 24
and  …(ii)
f2 25

Solving (i) and (ii), we get, f1  150 Hz and f2  144 Hz.


DOPPLER’S EFFECT IN SOUND :
The apparent shift in frequency of the wave motion when the source of sound or light moves with
respect to the observer, is called Doppler Effect.
Calculation of Apparent Frequency:

Suppose v is the velocity of sound in air, v s is the velocity of the source of sound(s), v 0 is the velocity of
the observe (O), and f is the frequency of the source.

(i) Source moves towards stationary observer. If the source were stationary the f waves sent out in
one second towards the observer O would occupy a distance v, and the wavelength would be v/f. If S
moves with a velocity v s towards (O), the f waves sent out occupy a distance (v  v s ) because S has
moved a distance v s towards O in 1s. So the apparent wavelength would be

v  vs
' 
f
Velocity of sound relative to O
Thus, apparent frequency f '  Wavelenght of wave reaching O

v  v 
f' f 
'  v  vs 

(ii) Source moves away from stationary observer. Now, apparent wavelength

v  vs
' 
f

Page # 169
AVES ON STRING AND SOUND WAVES PHYSICS PART - IV

 Apparent frequency f '  v /  ' n

 v 
or f'f 
 v  vs 

(iii) Observer moves towards stationary source.

Velocity of sound relative to O


f'
Wavelenght of wave reaching O

here, velocity of sound relative to O = v + v 0

and wavelength of waves reaching O = v/f


v  v0  v  v0 
 f' f 
v/f  v 
v  v0  v  v0 
(iv) Observer moves away from the stationary source f '  f 
v/f  v 

v  v0  v  v0 
f' f 
(v) Source and observer both moves toward each other v  vs  v  vs 
f
 v  v0 
(vi) Both moves away from each other f '  f  v  v 
 s   v  v0 
(vii) Source moves towards observer but observer moves away from source f '  f  
 v  vs 
 v  v0 
(viii) Source moves away from observer but observer moves towards source f '  f  .
 v  vs 

Illustration:
A sound detector is placed on a railway platform. A train, approaching the platform at a speed of 36 km/
h, sounds its whistle. The detector 12.0 kHz as the most dominant frequency in the whistle.If the train
stops at the platform and sounds the whistle, what would be the most dominant frequency detected ? The
speed of sound in air is 340 m/s.
Solution: Here the observer (detector) is at rest with respect to the medium (air). Suppose the dominant
frequency as emitted by the train is n0. When this same train was approaching the observer, the fre-
quency detected was,

   us  u 
v  0 or, 0      1  s  
  us   

The speed of the source is


36  103 m
us  36km / h   10m / s
3600s
 10 
Thus, 0   1    12.0kHz = 11.6 kHz.
 340 

Page # 170
AVES ON STRING AND SOUND WAVES PHYSICS PART - IV

TRY YOURSELF
2
1. A transverse wave is represented by the equation y = y0 sin (vt – x). For what value of  is the

maximum particle velocity equal to two times the wave velocity -
y 0 y 0
(A)  = 2y0 (B) = (C) = (D) = y0
3 2
2. A plane wave is represented by x = 1.2 sin (314 t + 12.56 y) Where x and y are distance measured along in
x and y direction in meters and t is time in seconds. This wave has-
(A) A wavelength of 0.25 m and travels in + ve x direction
(B) A wavelength of 0.25 m and travels in + ve y direction
(C) A wavelength of 0.5 m and travels in – ve y direction
(D) A wavelength of 0.5 m and travels in – ve x direction

1
3. The amplitude of a wave disturbance propagating along positive X-axis is given by, y = at t = 0 and
(1  x2 )

1
y= at t = 4sec where x and y in meters. The shape of the wave disturbance does not change
1  (x  2) 2
with time. The velocity of the wave is modified -
(A) 0.5 m/s (B) 1 m/s (C) 2 m/s (D) 4 m/s
4. The property of a medium necessary for wave propagation is -
(A) Its inertia (B) Its elasticity (C) Its low resistance (D) All of above

 x
5. The equation of a wave is represented by y = 10-4 sin  100t   m, then the velocity of wave will be :-
10 

(A) 100 m/s. (B) 4 m/s. (C) 1000 m/s. (D) 0.00 m/s.
6. The time taken by a particle in reaching from trough to crest in a transverse wave is -
(A) T/4 (B) T/2 (C) 3T/4 (D) T

t x
7. Equation of progressive wave is given by y = a sin   2  4  , where t is in seconds an x is in meters. Then
 
the distance through which the wave moves in 8 seconds is (in meter) -
(A) 4 (B) 2 (C) 16 (D) 8
8. If the speed of the wave shown in the figure is 330 m/s in the given medium, then the equation of the
wave propagating in the positive x-direction will be - (all quantities are in MKS units)
0.05m
y
(A) y = 0.05 sin 2 (4000 t – 12.5 x)
(B) y = 0.05 sin 2 (4000 t – 122.5 x)
(C) y = 0.05 sin 2 (3300 t – 10 x)
0.25m x
(D) y = 0.05 sin 2 (3300 × – 10 t)

9. The temperature in ºC at which the velocity of sound in air is half its value at 0ºC is -
(A) + 204.75 (B) 0.5 (C) –204.75 (D) – 273
10. A uniform rope of length 10m and mass 15 kg hangs vertically from a rigid support. A block of mass 5 kg
is attached to the free end of the rope. A transverse pulse of wavelength 0.08 m is produced at the lower
end of the rope. The wavelength of the pulse when it reaches the top of the rope will be -

Page # 171
AVES ON STRING AND SOUND WAVES PHYSICS PART - IV

(A) 0.08 m
15kg
(B) 0.04 m
(C) 0.16 m
5kg
(D) 0m

11. The density of the material of a wire used in sonometer is 7.5 × 10–5kg/m3. If the stress on the wire is 3.0
× 108N/m2, the speed of transverse wave in the wire will be -
(A) 100 m/s (B) 200 m/s (C) 300 m/s (D) 400 m/s
12. The length of a copper wire is 5m and its radius is 1mm. A force of 31.4 N is applied at each end of the
wire. The Young's modules of elasticity for copper is 11 × 1010 N/m2 . The density of copper is 8900 kg/
m3. The velocity of transverse waves is the wire is -
(A) 33.5 m/s (B) 82 m/s (C) 123 m/s (D) 164 m/s
13. If vm is the velocity of sound in moist air and vd is the velocity of sound in dry air, then -
(A) vd > vm (B) vd = vm (C) vd  vm (D) vm > vd
14. Disturbances of two waves are shown as a function of time in the following figure. The ratio of their
intensities will be -

II
(A) 1:4
I
(B) 4:1 a f
(C) 1:2 y
(D) 1:1
15. The length, mass and tension of a string are 1000 cm, 0.01 kg and 10 N respectively. The speed of transverse
waves in the string will be -
(A) 102m/s (B) 104m/s (C) 105m/s (D) none of these
16. In a transverse progressive wave of amplitude A, the maximum particle velocity is four times its wave
velocity. The wavelength of the wave is -
(1) A/4 (2) A/2 (3) A (4) 2A
17. The displacement equations for two waves undergoing super position are respectively y1 = 4 sin t and
y2 = 3 sin (t + /2) the amplitude of the resultant wave is -
(A) 5 (B) 7 (C) 1 (D) 0
18. Two waves of same frequency and of intensity 0 and 90 produces interference. If at a certain point the
resultant intensity is 70 then the minimum phase difference between the two sound waves will be -
(A) 90º (B) 100º (C) 120º (D) 110º
19. The equation y = 0.15 sin 5x cos 300 t, describes a stationary wave.The wavelength of the station any
wave is -
(A) zero (B) 1.256 metres (C) 2.512 metres (D) 0.628 metres
20. A stretched string of length L, fixed at both ends can sustain stationary waves of wavelength  given by -
(A)  = n2/2L (B)  = L2/2n (C)  = 2L/n (D)  = 2L n
21. A cylindrical tube open at both ends has a fundamental frequency f in air. The tube is dipped vertically in
water so that half of it is in water. The fundamental frequency of air column is now-
(A) f/2 (B) f (C) 2f (D) 3f/4
22. The first overtone of a closed organ pipe P1 and the third overtone of an open P2 are in unision with a
tuning fork. The ratio of the length of P1 and P2 will be -
(A) 3:8 (B) 8:3 (C) 3:1 (D) 1 :3
Page # 172
AVES ON STRING AND SOUND WAVES PHYSICS PART - IV

23 When a body of mass 25Kg is suspended from a sonometer wire then it barites with frequency 200 Hz. If
the volume of the body is 0.009 m3 and it is immersed in then the frequency of vibration of the wire will
be -
(A) 220 Hz (B) 160 Hz (C) 240 Hz (D) 160 Hz
24. Four wires of same length and same material, whose diameters are in the ratio 4 : 3 : 2 : 1, are clamped in
such a way that each wire produces note of frequency double that of the preceding wire. If the tension in
the first wire is 2 Kg-wt, then tension in the second wire will be -
(A) 4.5 (B) 8 (C) 9 (D) 16
25. Fundamental frequency of an open pipe of length 0.5 m is equal of the frequency of the first overtone one
of a closed pipe of length c. The value of c in meter is -
(A) 2 (B) 1 (C) 1.5 (D) 0.75
26. The frequency of a sitar wire is 440 vibrations per second. If the sitar player reduces the length of the
wire by 1/5 the, then the change in the frequency of stater wire will be -
(A) 2200 vibrations/sec (B) 1760 vibrations/sec
(C) 440 vibrations /sec (D) 110 vibrations/sec
27. An open organ pipe has fundamental frequency of 300 Hz. The first overtone of this organ pipe is the
same as the first overtone of a closed organ pipe. The length of the closed organ pipe is
(A) 10 cm (B) 41 cm (C) 82 cm (D) 164 cm
28. A source X of unknown frequency produces 8 beats per second with a source of 250 Hz and 12 beats per
second with a source of 270 Hz. The frequency of the source X is (Hz)-
(A) 282 (B) 262 (C) 242 (D) 258
29. A tuning fork A produced 4 beats/second with another tuning fork B of frequency 320 CPS. On filling the
fork A, 4 beats/second are again heard. Then, the frequency of fork A before filling (CPS)-
(A) 324 (B) 320 (C) 328 (D) 316
30. A tuning fork A frequency 200 Hz is sounded with fork B, the number of beats per second is 5. By putting
some was on A, the number of beats increases to 8. The frequency of fork B is -
(A) 200 Hz (B) 195Hz (C) 192 H (D) 205 Hz
31 The frequency of a fork A is 3% more than the frequency of a standard fork whereas the frequency of
fork B is 3% less. The forks A and B produce 6 beats per second. The frequency of standard fork will be
(A) 100 Hz (B) 106 Hz (C) 103 Hz (D) 112 Hz
32. A sources of sound gives 5 beats per second when sounded with another sources of frequency 100
second–1. The second harmonic of the source, together with a source of frequency 205 second–1 gives 5
beats per second. What is the frequency of the source ?
(A) 95 second–1 (B) 100 second–1 (C) 105 second–1 (D) 205 second–1
33. 56 tuning forks are so arranged series that each fork gives 4 beats per sec with the pervious one. The
frequency of the last fork is 3 times that of the first. The frequency of the first fork is
(A) 110 (B) 56 (C) 60 (D) 52
34. nary observer then the change in apparent frequency of sound observed by the observer, when Vs < < V,
will be -

2nVs 2nV 2nV0


(A) n = (B) n = V (C) n = 2nVsV (D) n =
V s V
35 A siren emitting sound of 1000Hz is moving away from us towards a high hill with velocity 10 m/s. The
frequency of sound reflected from the hill will be (V = 330 m/s)
(A) 100 Hz (B) 1000 Hz (C) 1031.25 Hz (D) 103 Hz

Page # 173
AVES ON STRING AND SOUND WAVES PHYSICS PART - IV

36 Two tuning forks, each of frequency 340 Hz, are moving with equal velocity with respect to an observer
at rest. One fork is approaching and another is receding away from the observer. The observer is listening
beats of frequency 3Hz. If the velocity of sound is 340 m/s then the speed of fork will be -
(A) 0.4 m/s (B) 2.8 m/s (C) 4.2 m/s (D) 1.5 m/s
37. A source of sound of frequency 90 Hz is moving towards an observer with a velocity 1/10th the velocity
of sound. The frequency heard by the observer will be -
(A) 200 Hz (B) 100 Hz (C) 300 Hz (D) 50 Hz
38. If a sound source of frequency n approaches an observer with velocity V/4 and the observer approaches
the source with velocity V/5 then the apparent frequency heard will be -
(A) (5/8) n (B) (8/5) n (C) (7/5) n (D) (5/7) n
39. When a source approches a stationary observer with velocity v then apparent change in frequency is
n1. When observer approches from stationary source with same velocity v then the apparent change in
frequency is n2, then -
(A) n1 > n2 (B) n1 < n2 (C) n1 = n2 (D) none of the above
40. A sound source, emitting sound of constant frequency, moves with a constant speed and crosses a stationary
observer. The frequency (n) of sound heard by the observer is plotted against time (t). Which of the
following graphs represents the correct variation ?

n n n n

(A) (B) (3) (D)

t t t t

41 An observer moves towards a stationary source of sound with a speed 1/5th of the speed of sound. The
wavelength and frequency of the sources emitted are  and f respectively. The apparent frequency and
wavelength recorded by the observer are respectively-
(A) 1.2 f, 1.2 (B) 1.2f,  (C) f, 1.2  (D) 0.8f, 0.8 

Page # 174
AVES ON STRING AND SOUND WAVES PHYSICS PART - IV

SOLVED EXAMPLES
1. When a train is approaching the observer, the frequency of the whistle is 100 cps. When it has passed the
observer, it is 50 cps. Calculate the frequency when the observer moves with the train.
Sol. When the source (the train) moves towards the observer, the apparent frequency is

v
f  f …(i)
v  vs

while the source is moving away, the apparent frequency

v
f   f …(ii)
v  vs

 v 
100 2  v  1
f  v  vs s v
 
 f   v  v or 50 1  v  or vs
3 …(iii)
s  v  1
s

When the observer moves with the train, there is no relative motion between the source and the observe and
hence he will listen the true frequency f. Substituting (iii) in (i), we get
3 200
100  f or f  66.6 Hz.
2 3
2. A 5 watt source sends out waves in air at frequency 1000 s–1. Deduce the intensity at a 100 meter distance,
assuming spherical distribution. If v = 350 ms–1 and  = 1.3 kg/m3, deduce the displacement amplitude.
Sol. We know that intensity is given by,

Power 5Watt
= = 4(100)2 m 2
Area

I
Also,  = 2v2 n2a2  a2 =
2n 2 v

Hence, displacement amplitude

1 I
a= 2v
n

Given that, n = 1000 s–1,  = 4 × 10–5Wm–2  = 1.3 kg m–3 and v = 350 ms–1

7 4  105
a= = 6.67 × 10–8m
22  1000 2  1.3  350
3. S1 and S2 are two coherent currents sources of radiations separated by distance 100.25 , where  is the
wavelength of radiation. S1 leads S2 in phase by /2. A and B are two points on the line joining S1 and S2 as
shown in figure.
A S1 S2 B

The ratio of amplitudes of source S1 and S2 are in ratio 1 : 2. Find the ratio of intensity at A to that of B is–
Sol. For interference at A : S2 is behind of S1 by a distance of 100.25 .
Further S2 lags S1 by /2. Hence the waves from S1 and S2 interference at A with a phase difference

Page # 175
AVES ON STRING AND SOUND WAVES PHYSICS PART - IV

2 
 (100.25 )   201 is equivalent to phase difference of  .
 2
Hence the net amplitude at A is 2a – a = a
For interference at B : S2 is ahead of S1 by a distance of 100.25 .


Further S2 lags S1 by . Hence the waves from S1 and S2 interference at B with a phase difference
2

2 
 (100.25 )   200 is equivalent to no phase difference of .
 2 2
 IA   a 1
Hence the net amplitude at A is 2a + a = 3a . Hence  I    3a   9
B
4. Two travelling waves of equal amplitudes and equal frequencies move in opposite directions along a string.
They interfere to produce a standing wave having the equation y  A cos kx sin t in which A  1.0 mm,
-1 and
k  1.57 cm   78.5s 1 .
(a)Find the velocity of the component traveling waves.
(b) Find the node closest to the origin in the region x > 0.
(c) Find the antinode closest to the origin in the region x > 0.
(d) Find the amplitude of the particle at x  2.33 cm.
Sol. (a)The standing wave is formed by the superposition of the waves
A A
y1  sin(t  kx) and y 2  sin(t  kx)
2 2
The wave velocity (magnitude) of either of the waves is

 78.5s 1
v   50 cm/s.
k 1.57 cm 1

(b) For a node, cos kx  0


The smallest positive value of x satisfying this relation is given by
  3.14
kx  or x   1 cm.
2 2k 2  1.57 cm -1

(c) For an antinode, | cos kx | = 1


The smallest positive x satisfying this relation is given by
y
kx =  or x = /k = 2 cm.
(d) The amplitude of vibration of the particle at x is given by | A cos kx |. For the given point,
7 
kx  (1.57 cm -1 )(2.33 cm)    
6 6

3
Thus, the amplitude will be (1.0mm)| cos(   / 6)| mm  0.86 mm
2
5. A metre-long tube open at one end, with a movable piston at the other end, shows resonance with a fixed
frequency source (a tuning fork of frequency 340 Hz) when the tube length is 25.5 cm or 79.3 cm. Estimate
the speed of sound in air at the temperature of the experiment. The edge effects may be neglected.
Sol. The tube will act as closed organ pipe. The speed of sound wave is given by
v  2  ( 2  1 )

Page # 176
AVES ON STRING AND SOUND WAVES PHYSICS PART - IV

then v = 340 Hz, 1 = 25.5 cm,  2 = 79. 3 em


v = 2 × 340 (79. 3 – 25. 5) = 680(53.8) – 36584 ms–1 = 365.84 ms–1
6. Radio waves coming at   to vertical are received by a
radar after reflection from a nearby water surface and di-
rectly. What can be height of antenna from water surface
so that it records a maximum intensity (a maxima). h
(wavelength = )

Sol.
Total path difference = AB + BC + /2 =  for maxima C
h sec cos 2 + h sec = /2
h sec (2cos2 ) = /2 h

h
2 cos  hsec A
(hsec ) cos2
7.
B
A train, standing at the outer signal of a railway station blows a whistle of frequency 400 Hz in still air. (i)
What is the frequency of the whistle for a platform observer when the train (a) approaches the platform with
a speed of 10 m s–1,
(b) recedes from the platform with a speed of 10 m s–1? (ii) What is the speed of sound in each case ? The
speed of sound in still air can be taken as 340 m s–1.
Sol. Here air is still and listener is standing on the platform so vm = 0, v0 = 0

 v  vm  v0   v 
from formula     ;    
 v  vm  vs   v  vs 

 = 400 Hz, vs = 10 ms–1, v = 340 ms–1.

 340 
    340  10  400  412.12 Hz

(b) When the train recedes from the platform the frequency of the whistle

 v   340 
   = 400  388.57 Hz

 v  vs   340  10 

(ii) The speed of sound in each case will be same.


8. Find the fundamental frequency and the first four overtones of a 15 cm pipe
(a)if the pipe is closed at one end, (b) if the pipe is open at both ends (c) How many overtones may be
heard by a person of normal hearing in each of the above cases? Velocity of sound in air = 330 ms–1.
c 330
Sol. (a) n 0  where n0 = frequency of the fundamental  n 0   550 Hz
4 4  0.15
The first four overtones are 3n0, 5n0, 7n0 and 9n0.
 So, the required frequencies are 550, 1650, 2750, 3850, 4950 Hz.

c 330
(b) n0    1100 Hz.
2 2  0.15
The first four overtones are 2n0, 3n0, 4n0 and 5n0. So, the required frequencies are 1100, 2200, 3300, 4400,
5500 Hz.

Page # 177
AVES ON STRING AND SOUND WAVES PHYSICS PART - IV

(c) For pipe closed at one end , the frequency of the nth overtone is (2n + 1) n0 .
 (2n + 1) n0= 20000 or (2n + 1) 550 = 20000 or n = 17.68.
The acceptable value is 17.
For the pipe open at both ends , the frequency of the nth overtone is (n +1) n0.
 (n +1) n0= 20000 or (n +1)1100 = 20000 or n = 17.18
The acceptable value is 17.
9. The ratio in the densities of oxygen and nitrogen is 16 : 14. At what temperature the speed of sound will be
the same which is in nitrogen at 15ºC.
Sol. If M be the molecular weight of the gas and T be the absolute temperature, then speed of sound in a gas.

 RT   P RT 
v =     
M  d M

 R(273  t) 
Where R is universal gas constant. Velocity of sound in oxygen at tºC =  
 M0 

 R(273  15) 
Velocity of sound in nitrogen at 15ºC =  MN

 

 R(273  t)   R(273  15) 


According to the given problem, 
M0
 = 
MN

   

M0 273  t
 M =
n 288

16 273  t  M0 16 
=  M  14 
14 288 N

Solving we get, t = 56.1ºC


10. A point source is emitting sound in all directions. Find the ratio of distance of two points from the point
source where the difference in loudness levels is 3 dB. (log10 2 = 0.3)

 I  K / r2 
Sol. dB  10 log    10log    10 [log (K1 )  2log r]
 I0   I0 

dB1 = 10 (log K' – 2 log r1), dB2 = 10 (log K' – 2 log r2)

r 
3 = dB1 – dB2 = 20 log  2 
r  1

Pm  0.0001 P0

2
 r2 
log2= log   since log10 2 = 0.3
 r1 

r  1
 1 
 r2  2

11. A 3m long organ pipe open at both ends is driven to third harmonic standing wave. If the amplitude of
pressure oscillation is 0.1% of the mean atmospheric pressure (P0 = 105 N/m2). Find the amplitude of (i)
particle oscillation and (ii) density oscillation.

Page # 178
AVES ON STRING AND SOUND WAVES PHYSICS PART - IV

Speed of sound v = 330 m/s, density of air 0 =1.0 kg/m3.



Sol. 3  3.   2m
2

Pm = 100 N/m2, V = 330 m/s,  0 = 1 kg/m3 { Pm  0.0001 P0 }

2 Pm 2 100 1
(i) Pm  Bs0 k  0 v 2 s0  s0  2
= s0  m
 0 v 2  1 330  330  2 1089

dp dP d dv
(ii) B   dv / v  d /   m  v  0    v since m is constant.

.dp  P .Pm 100


d   (d) max  (dp) max  m = 2
 kg / m 3
B B B v 1089

12. In a car race sound signals emitted by the two cars are detected by the detector on the straight track at the end
point of the race. Frequency observed are 330 Hz and 360 Hz and the original frequency is 300 Hz of both
cars. Race ends with the separation of 100m between the cars. Assume both cars move with constant
velocity and velocity of sound is 330 m/s. Find the time taken by winning car.
Sol. Let the velocities of car 1 and car 2 be V1 m/s and V2 m/s
Apparent frequencies of sound emitted by car 1 and car 2 as detected at end point are

V V
f1  f 0 , f2  f0
V  V1 V  V2 V 2t
V1 t
330 1 2
330
 330  300 ; 360  300 330  V
330  V1 2 A B
100m
 V1 = 30 m/s and V2 = 100 m/s
The distance between both the cars just when the
2nd car reach and point B (as shown in figure is)
100m  V2 t  V1t  t  4 sec
13. The fundamental frequency of a sonometer wire increases by 6 Hz if its tension is increased by 44% keeping
the length constant. Find the change in the fundamental frequency of the sonometer wire when the length of
the wire is increased by 20% keeping the original tension in the wire.

1 T
Sol. Fundamental frequency of sonometer wire is f
2L 

when length L is constant, f  T  f k T

1 f 1 T
or log f  log k  log T Differentiations, 
2 f 2 T

T 6  2 300
Given that f  6 Hz, = 0.44  f  Hz
T 0.44 11

k
Next, when T is constant and length is changed, f   kL1
L
log f = log k – log L

Page # 179
AVES ON STRING AND SOUND WAVES PHYSICS PART - IV

f L  L  20 300
Differentiating,  f    f    = – 5.5 Hz
f L
  L  100 11

 The fundamental frequency of sonometer wire will decrease by 5.5 Hz


14. A wave pulse starts propagating in the +x direction along a non-uniform wire of length 10 m with mass per
unit length given by    0  x and under a tension of 100 N. Find the time taken by a pulse to travel from the
lighter end (x = 0) to the heavier end. ( 0  102 kg/m and  =g  103 kg/m 2 )
Sol. The speed of the wave pulse

T T dx
v  
  0  x dt

t 
 0  x 2 1  3/ 2 3/2
 t   dt   dx    0      0  
0 0
T 3 T  

substituting the values, we get

2 1  2 3/2 3/ 2  2  100 
t 
3 9  10  100 
3 
10  9  1010  10   
 102   27 
10 3/ 2  10 3   0.227s
15. Two metallic strings A and B of different materials are connected in series forming a joint. The strings have
equal cross-sectional area. The length of A is  A  0.3m and that of B is  B  0.75m . One end of the combined
string is tied with a support rigidly and the other end is loaded with a block of mass m passing over a
frictionless pulley. Transverse waves are set up in the combined string using an external sources of variable
frequency. Calculate
(a) Lowest of frequency for which standing waves are observed such that the joint is a node,
(b) The total number of antinodes at this frequency.
The densities of A and B are 6.3  103 kg/m3 and 2.8  103 kg/m3 respectively.
y.
Sol. (a)Let p and q be the number of loops formed in A and B

p T q T
 n A  2 AA and n B  2 A
A B B

n A p  B B q  A A 0.03 6.3 5
  . .  1 n A  n B    .  
n B q  A A p  B B 0.75 2.8 3

p 3 6 9
 the ratio of the number of loops  , , ,....
q 5 10 5

p 3
for the lowest frequency, 
q 5

this means that A will have 3 loops and B will have 5 loops. Hence the required minimum frequency

3 T 5 T 3 mg m
f min   or f min  
2A AA 2B AB 2  0.3 A  6.3  103 25.7A

(b) the total number of antinodes at this minimum frequency is 3 + 5 = 8

Page # 180
AVES ON STRING AND SOUND WAVES PHYSICS PART - IV

16. When 0.98 m long metallic wire is stressed, an extension of 0.02 m is produced. An organ pipe 0.5 m long and
open at both ends, when sounded with this stressed metallic wire, produces 8 beats in its fundamental mode.
By decreasing the stress in the wire, the frequency of beats is found to decrease. Find the Young’s modulus of
the wire. The density of metallic wire is 104 kg/m3 and speed of sound in air is 292 m/s.
Sol. Frequency of the transverse vibration of the stretched string in

1 T
f1  , …(i)
2  L  L  

T L YAL 1 YL
Here, Y  , or T  , and   A  f1  …(ii)
A L L 2  L  L  L

when the stress in the wire is decreased, f1 will decrease, consequently the beat frequency will decrease if
f1  f pipe

1 YL v
 f1  f pipe  8 or  8
2  L  L L 2

Substituting the value, we get

1 Y  0.02 292
 8
2  0.98  0.02 0.98  10 4 2  0.5

Simplifying, we get, Y  1.764  1011 N/m 2

Page # 181
AVES ON STRING AND SOUND WAVES PHYSICS PART - IV

EXERCISES
ONLY ONE CHOICE IS CORRECT
1. Which of the following is a mechanical wave ?
(A) Radio waves (B) X–rays (C) Light waves (D) Sound waves.
2. A cork floating in a calm pond executes simple harmonic motion of frequency  when a wave generated
by a boat passes by it. The frequency of the wave is
(A)  (B)  /2 (C) 2  (D) 2 .
3. A wave pulse, travelling on a two–piece string, gets partially reflected and partially transmitted at the
junction. The reflected wave is inverted in shape as compared to the incident one. If the incident wave
has wavelength  and the transmitted wave  ' ,
(A)  '   (B)  '  
(C)  '   (D) nothing can be said about the relation of  and  ' .
4. A transverse wave travels along the Z–axis. The particles of the medium must move
(A) along the Z–axis (B) along the X–axis (C) along the Y–axis (D) in the X–Y plane.
5. Longitudinal waves cannot
(A) have a unique wavelength (B) transmit energy
(C) have a unique wave velocity (D) be polarized.
6. The fundamental frequency of a string is proportional to
(A) inverse of its length (B) the diameter
(C) the tension (D) the density.
7. The displacement Vs time graph for two waves A and B
y A
which travel along the same string are shown in the figure. 3

Their intensity ratio IA / IB is


0
t
1 2 3 4 5 6 7 8 9 10 11 12
9
(A) (B) 1 –2
4 B

81 3
(C) (D)
16 2
8. Mark out the correct options :
(A) the energy of any small part of a string remains constant in a travelling wave
(B) the energy of any small part of a string remains constant in a standing wave
(C) the energies of all the small parts of equal length are equal in travelling wave
(D) the energies of all the small parts of equal length are equal in standing wave.
9. Two wave pulses travel in opposite directions on a string and approach each other. The shape of one
pulse is inverted with respect to the other.
(A) the pulses will collide with each other and vanish after collision
(B) the pulses will reflect from each other i.e., the pulse going towards right will finally move towards
left and vice versa
(C) the pulses will pass through each other but their shapes will be modified
(D) the pulses will pass through each other without any change in their shapes.

Page # 182
AVES ON STRING AND SOUND WAVES PHYSICS PART - IV

10. When two waves with same frequency and constant phase difference interfere,
(A) there is a gain of energy
(B) there is a loss of energy
(C) the energy is redistributed and the distribution changes with time
(D) the energy is redistributed and the distribution remains constant in time.
11. Consider the following statements about sound passing through a gas.
(a) The pressure of the gas at a point oscillates in time.
(b) The position of a small layer of the gas oscillates in time.
(A) both a and b are correct (B) b is correct but a is wrong
(C) b is correct but a is wrong (D) both a and b are wrong.
12. When we clap our hands, the sound produced is best described by
(A) p  p0 sin(kx  t) (B) p  p0 sin kx cos t

(C) p  p0 cos kx sin t (D) p  p0n sin(k n x  n t) .


Here p denotes the change in pressure from the equilibrium value.
13. The bulk modulus and the density of water are greater than those of air. With this much of information,
we can say that velocity of sound in air
(A) is larger than its value in water (B) is smaller than its value in water
(C) is equal to its value in water (D) cannot be compared with its value in water.
14. A tuning fork sends sound waves in air. If the temperature of the air increases, which of the following
parameters will change ?
(A) displacement amplitude (B) frequency
(C) wavelength (D) time period.
15. When sound wave is refracted from air to water, which of the following will remain unchanged?
(A) wave number (B) wavelength (C) wave velocity (D) frequency.
16. The speed of sound in a medium depends on
(A) the elastic property but not on the inertia property
(B) the inertia property but not on the elastic property
(C) the elastic property as well as the inertia property
(D) neither the elastic property nor the inertia property.
17. When you speak to your friend, which of the following parameters have a unique value in the sound
produced ?
(A) frequency (B) wavelength (C) amplitude (D) wave velocity.
18. An open organ pipe of length L vibrates in its fundamental mode. The pressure variation is maximum
(A) at the two ends (B) at the middle of the pipe
(C) at distances L/4 inside the ends (D) at distances L/8 inside the ends.
19. An organ pipe, open at both ends, contains
(A) longitudinal stationary waves (B) longitudinal travelling waves
(C) transverse stationary waves (D) transverse traveling waves.
20. The phenomenon of beats can take pace
(A) for longitudinal waves only
(B) for transverse waves only
(C) for both longitudinal and transverse waves
(D) for sound waves only.

Page # 183
AVES ON STRING AND SOUND WAVES PHYSICS PART - IV

21. The engine of a train sounds a whistle at frequency  . The frequency heard by a passenger is

1
(A)   (B)  (C)  (D)   .

22. The change in frequency due to Doppler effect does not depend on
(A) the speed of the source
(B) the speed of the observer
(C) the frequency of the source
(D) separation between the source and the observer.
B
23. A small source of sound moves on a circle as shown in figure and
C O
an observer is sitting at O. Let 1,  2 , v 3 be the frequencies
heard when the source is at A, B and C respectively. A
(A) 1  2  3 (B) 1   2   3 (C)  2  3  1 (D) 1  3   2 .
24. A source of sound moves towards an observer.
(A) the frequency of the source is increased
(B) the velocity of sound in the medium is increased
(C) the wavelength of sound in the medium towards the observer is decreased
(D) the amplitude of vibration of the particles is increased.
25. Two cars having velocities of 100 and 150 kPh are approaching each other. The frequency of a note
emitted by the first as heard by its driver of the other is 1000 Hz. Speed of sound is 330 m/s. The two
frequency of sound is
(A) 500 Hz (B) 750.6 Hz (C) 844.7 Hz (D) 900 Hz.
26. The displacement of particles in a string stretched in the x-direction is represented by y. Wave equation
is described by :
(A) cos kx sin wt (B) k2x2 – w2t2 (C) cos2(kx + wt) (D) cos (k2x2 – w2t2).
27. A wave pulse on a string has the dimension shown in figure. The wave speed is v = 1 cm/s. If point O is
a free end. The shape of wave at time t = 3 s is :
v=1cm/s
1cm
O
1cm 1cm 2cm

O 1cm
(A) (B)
1cm

1cm O

1cm 2cm
(C) (D) .
O 1cm
28. In the above problem, shape of the wave at time t = 3 s if O is a fixed end will be :

O 1cm
(A) (B)
1cm

Page # 184
AVES ON STRING AND SOUND WAVES PHYSICS PART - IV

1cm O

1cm 2cm
(C) (D) .
O 1cm

29. Equations of a stationary and a travelling waves are as follows y 1  a sin kx cos t and y 2  a sin( t  kx)
 3
The phase difference between two points x1  and x2  are 1 and 2 respectively for the two
wo
3k 2k
1
waves. The ratio  is :
2

5 3 6
(A) 1 (B) (C) (D)
6 4 7
30. In a sine wave position, of different particles at time t = 0 is shown in the
y
figure. The equation for this wave if it is travelling along positive x–axis can be
(A) y  A sin( t  kx) (B) y  A sin(kx  t) x

(C) y  Acos(t  kx) (D) y  Acos(kx  t) .


31. A 100 Hz sinusoidal wave is travelling in the positive x–direction along a string with a linear mass density
of 3.5 × 10–3 kg m–1 and a tension of 35 N. At time t = 0, the point x = 0 has zero displacement and the slope
of the string is  / 20 . Then select the wrong alternative :
(A) velocity of wave is 100 m/s (B) angular velocity is (200 ) rad/s
(C) amplitude of wave is 0.025 m (D) none of the above.
32. The time taken by sound waves to travel the distance  if the air temperature between them varies
linearly from T1 to T2 (velocity of sound in air is given by v   T

2 2  
(A)   T2  T1  (B)   T2  T1  (C)   T2  T1  (D)   T2  T1 .

 x
33. A transverse wave is described by the equation y  y 0 sin2  ft –  . The maximum particle velocity is
 
equal to four times the wave velocity if :

y 0 y 0
(A)   (B)  (C)   y 0 (D)   2y 0 .
4 2
34. A standing wave is maintained in a homogeneous string of cross–sectional area “s” and density  . It is
formed by the superposition of two waves travelling in opposite directions given by the equation
y 1  a sin( t  kx) and y 2  2a sin( t  kx)
the total mechanical energy confined between the sections corresponding to the adjacent antinodes is :
3s2a 2 s2a2 5s2a 2 2s2a 2
(A) (B) (C) (D) .
2k 2k 2k 2k
35. Two waves are represented by
y 1  5sin2(75t  0.25x)

y 2  10sin2(150t  0.50x)
I1
The intensity ratio I of the two waves is
2

Page # 185
AVES ON STRING AND SOUND WAVES PHYSICS PART - IV

(A) 1 : 2 (B) 1 : 4 (C) 1 : 8 (D) 1 : 16.


36. If two waves of the same frequency and same amplitude on superposition produce a resultant disturbances
of same amplitude the waves differe in phase by

2 
(A)  (B) (C) (D) 0.
3 3
37. Two pipes have each of length 2 m. One is closed at one end and the other is open at both ends. The speed
of sound in air is 340 m/s. The frequency at which both can resonate is ?
(A) 340 Hz (B) 510 Hz (C) 42.5 Hz (D) none of these.
38. A cylindrical tube, open at both ends, has a fundamental frequency ‘f’ in air. The tube is dipped vertically
in water so that half of it is in water. The fundamental frequency of the air column is now :
f 3f
(A) (B) (C) f (D) 2f.
2 4
39. A tube, closed at one end and containing air, produces, when excited, the fundamental note of frequency
512 Hz. If the tube is opened at both ends the fundamental frequency that can be excited is (in Hz):
(A) 1024 (B) 512 (C) 256 (D) 128
40. A movable bridge divides a sonometer wire into two parts which differ in length by 1 cm and produce 4
beats per second when sounded together. If the whole length of the wire is 100 cm, the frequencies of the
parts are
(A) 100 Hz, 104 Hz (B) 198 Hz, 202 Hz (C) 206 Hz, 210 Hz (D) 250 Hz, 254 Hz.
41. The equation for the vibration of a string fixed at both ends vibrating in its third harmonic is given by
y  2cm sin[(0.6cm1 )x] cos[(500s1 )t] The length of the string is :
(A) 24.6 cm (B) 12.5 cm (C) 20.6 cm (D) 15.7 cm.

1
42. A string is under tension so that its length is increased by
times its original length. The ratio of fundamental
n
frequency of longitudinal vibrations and transverse vibrations will be :
(A) 1 : n (B) n2 : 1 (C) n :1 (D) n : 1.

43. The vibration of a stretched string fixed at both ends are described by y  4 sin 2x cos t . The minimum
length of the wire will be
(A) 1 m (B) 0.5 m (C) 5 m (D) 2 m.
44. A certain organ pipe three successive resonace frequencies are at 425, 595 abd 765 Hz. If the speed of
sound in air is 340 m/s the length of the pipe is
(A) 2m (B) 1m (C) 1.5m (D) 2.3m.
45. A wave represented by the equation y = a cos (kx – wt) is superposed with another wave to form a
stationary wave such that point x = 0 is a node. The equation for the other wave is
(A) a sin (kx + wt) (B) – a cos (kx – wt) (C) –a cos (kx + wt) (D) –a sin (kx – wt).
46. A closed organ pipe and an open organ pipe of same length produce 2 beats when they are set into
vibrations simultaneously in their fundamental mode. The length of open organ pipe is now halved and of
closed organ pipe is doubled, the number of beats produced will be :
(A) 8 (B) 7 (C) 4 (D) 2.
47. An open pipe of length  is sounded together with another open organ pipe of length   x in their
fundamental tones. Speed of sound in air is v. The beat frequency heard will be (x   ) :

vx v2 vx vx 2
(A) (B) (C) (D) .
4 2 2x 22 2

Page # 186
AVES ON STRING AND SOUND WAVES PHYSICS PART - IV

48. A source of sound of frequency 256 Hz is moving rapidly towards a wall with a velocity of 5 m/s. If
sound travels at a speed of 330 m/s, then number of beats per second heard by an observer between the
wall and the source is
(A) 7.7 Hz (B) 9 H (C) 4 Hz (D) none of these.
49. 50 tuning fork are so arragned in series that each fork gives 5 beats/s with the previous one. If the last
fork gives the octave of first the frequency of later is
(A) 245 Hz (B) 250 Hz (C) 240 Hz (D) 260 Hz.
50. The frequency changes by 10% as the source approaches a stationary observer with constant speed vs.
What would be the percentage change in frequency as the source recedes the observer with the same
speed ? Given, that vs << v (v = speed of sound in air)
(A) 14.3% (B) 20% (C) 16.7% (D) 10%.
51. A detector is released from rest over a source of sound of frequency
f0 = 103 Hz. The frequency observed by the detector at f(Hz)
time t is plotted in the graph. 2000
2
The speed of sound in air is : (g = 10 m/s ) 1000
(A) 330 m/s (B) 350 m/s t(s)
30
(C) 300 m/s (D) 310 m/s.
52. A car, sounding a horn of frequency 1000 Hz, is moving directly towards a huge wall at a speed of
15 m/s. If speed of sound is 340 m/s, then the freqeuncy of the echo heard by the driver is
(A) 1046 Hz (B) 954 Hz (C) 1092 Hz (D) 908 Hz.
53. A tuning fork produces a wave os wavelength 110 cm in air at 0ºC. The wavelength at 25ºC would be
(A) 110 cm (B) 115 cm (C) 120 cm (D) 130 cm.
1
54. The equation of a travelling wave at t = 0 is y  . After 0.2 second the equation takes the form
(1  x 2 )
1
y 2 . The velocity of the wave is
x  2(x  1)
(A) 5 m/s towards + X-axis (B) 1 m/s towards + X-axis
(C) 5 m/s towards – X-axis (D) 1 m/s towards – X-axis.
55. Two waves of wavelengths 99 cm and 100 cm travelling with a speed of 396 ms-1 are made to interfere.
The number of beats produced per second by them is
(A) 1 (B) 2 (C) 4 (D) 8.
56. A wave is represented by the equation
1 1 
z  2Acos k(x  y) sin  k(x  y)  t 
2  2 
It is propagating along a direction.
(A) parallel to X-axis (B) parallel to Y-axis
(C) making an angle of 45º with X-axis (D) making an angle of 135º with X-axis.
57. The fundamental frequency of a sonometer wire of length  is f0. A bridge is now introduced at a distance
of  from the centre of the wire (   ) . The number of beats heard if both sides of the bridges are sett
into vibration in their fundamental modes are :

8f0  f0  2f0  4f0 


(A) (B) (C) (D) .
   

Page # 187
AVES ON STRING AND SOUND WAVES PHYSICS PART - IV

58. A source X of sound of unknown frequency produces 8 beats per second with a source of 250 Hz and 12
beats per second with a source of 270 Hz. The frequency of source X is
(A) 258 Hz (B) 242 Hz (C) 262 Hz (D) 282 Hz.
59. A point source is emitting sound in all directions. The ratio of distance of two points from the point
source where the difference in loudness levels is 3 dB is (log10 2 = 0.3)
1 1 1 2
(A) (B) (C) (D)
2 2 4 3
60. An organ pipe P1 closed at one end vibrating in its first harmonic and another pipe P2 open at ends
vibrating in its third harmonic are in resonance with a given tuning fork. The ratio of the length of P1 and
P2 is :

8 3 1 1
(A) (B) (C) (D)
3 8 6 3
61. Two sources S1 and S2, each emitting waves of wavelength  are kept symmetrically on either side of
centre O of a circle ABCD such that S1 O = S2 O = 2  . If a detector is moved along the circumferece of the
circle, it will record how many maxima in one revolution :
(A) 8 (B) 12 (C) 16 (D) 24.
62. Two coherent sources of different intensities send waves which interfere. The ratio of the maximum intensity
to the minimum intensity is 25. The intensities are in the ratio
(A) 25 : 1 (B) 5 : 1 (C) 9 : 4 (D) 625 : 1.
63. A standing wave is produced on a string clamped at one end and free at the other. The length of the string
(A) must be an integral multiple of  / 4 (B) must be an integaral multiple of  / 2
(C) must be an integral multiple of  (D) may be an integral multiple of  / 2 .
64. A transverse sinusoidal wave of amplitude a, wavelength  and frequency f is travelling on a stretched
string. The maximum speed of any point on the string is v/10, where v is the speed of propagation of the
wave. If a = 10-3 m and v = 10 ms-1, then  is given by
(A) 2  102 m (B) 103 m (C) 2  10 2 m (D) 2  10 3 m .

ONE OR MORE THAN ONE CHOICE MAY BE CORRECT


1. A wave equation which gives the displacement along the y-direction is given by:
y = 10–4 sin (60 t + 2x) where x and y are in metres and t is time in seconds. This represents a wave:
(A) traveling with a velocity of 30 m/s in the negative x-direction
(B) of wavelength m
(C) of frequency 30 /  hertzz
(D) of amplitude 10–4 m traveling along the negative x-direction
2. In a plane progressive harmonic wave :
(A) phase difference between displacement and acceleration of particle is zero
(B) phase difference between displacement and acceleration of particle is 

(C) phase difference between displacement and velocity of particle is
2

(D) phase difference between velocity and acceleration of particle is .
2
3. A wave is represented by the equation;
y  A sin(10x  15t   / 3)

Page # 188
AVES ON STRING AND SOUND WAVES PHYSICS PART - IV

where x is in meters and t is in seconds. The expression represents:


(A) a wave traveling in the positive x-direction with a velocity 1.5 m/s.
(B) a wave traveling in the negative x-direction with a velocity 1.5 m/s
(C) a wave traveling in the negative x-direction with a wavelength 0.2 m
(D) a wave traveling in the positive x-direction with a wavelength 0.2 m

4. The figure represents a longitudinal wave travelling in positive x–direction. Then


(A) part ABC represents compression
A E
(B) part ABC represents rarefaction B x
(C) part CDE represents compression D

(D) part CDE represents rarefaction. C

5. The figure shows four progressive waves A, B, C and D. It can be concluded from the figure that with
respect to wave A
y
B A C

t

t=/2 t= t=3/2

(A) the wave C is ahead by a phase angle of  / 2 and the wave B lags behind by a phase angle  / 2
(B) the wave C lags behind by a phase angle of  / 2 and the wave B is ahead by a phase angle of  / 2
(C) the wave C is ahead by a phase angle of  and the wave B lags behind by a phase angle 
(D) the wave C lags behind by a phase angle of  and the wave B is ahead by a phase angle of  .
6. An electrically maintained tuning fork vibrates with constant frequency and constant amplitude. If the
temperature of the surrounding air increases but pressure remains constant, the sound produced will
have
(a) larger wavelength (b) larger frequency
(c) larger velocity (d) larger time period.
7. A listener is at rest with respect to the source of sound. A wind starts blowing along the line joining the
source and the observer. Which of the following quantities do not change?
(a) Frequency (b) Velocity of sound
(c) Wavelength (d) Time period.
8. A mechanical wave propagates in a medium along the X-axis. The particles of the medium
(A) must move on the X-axis (B) must move on the Y-axis
(C) may move on the X-axis (D) may move on the Y-axis.
9. A wave propagating in a solid
(A) must be longitudinal (B) may be longitudinal
(C) must be transverse (D) may be transverse.
10. A wave is represented by the equation
y = (0.001 mm) sin[(50 s–1)t (2.0 m–1)x]
(A) the wave velocity = 100 m/s (B) the wavelength = 2.0 m.
(C) the frequency = 25 /  Hz. (D) the amplitude = 0.001 mm.

Page # 189
AVES ON STRING AND SOUND WAVES PHYSICS PART - IV

11. Two particles A and B have a phase difference of  when a sine wave passes through the region.
(A) A oscillates at half the frequency of B
(B) A and B move in opposite directions.
(C) A and B must be separated by half of the wavelength.
(D) The displacements at A and B have equal magnitudes.
12. An air column in a pipe, which is closed at one end, will be in resonance with a vibrating tuning fork of
frequency 264 Hz, if the length of the column in cm is : (speed of sound = 330 m/s)
(A) 31.25 (B) 62.50 (C) 93.75 (D) 12.5
13. Velocity of sound in air is 320 m/s. a pipe closed at one end has a length of 1 m. Neglecting end corrections,
the air column in the pipe can resonate for sound of frequency:
(A) 80 Hz (B) 240 Hz (C) 320 Hz (D) 400 Hz
14. The equation of a stationary wave in a string is y = (4 mm) sin [3.14 m–1) x] cos t .
Select the correct alternative(s) :
(A) the amplitude of component waves is 2 mm
(B) the amplitude of component waves is 4 mm
(C) the smallest possible length of string is 0.5 m
(D) the smallest possible length of string is 1.0 m.
15. The fundamental frequency of a vibrating organ pipe is 200 Hz.
(a) The first overtone is 400 Hz (b) The first overtone may be 400 Hz
(c) The first overtone may be 600 Hz (d) 600 Hz is an overtone.
16. In a stationary wave.
(A) all the particles of the medium vibrate in phase
(B) all the antinodes vibrate in phase
(C) the alternate antinodes vibrate in phase
(D) all the particles between consecutive nodes vibrate in phase.
17. In a resonance tube experiment, a closed organ pipe of length 120 cm resonates when tuned with a
tuning fork of frequency 340 Hz. If water is poured in the pipe then (given vair = 340 m/s)
(A) minimum length of water column to have the resonance is 45 cm
(B) the distance between two successive nodes is 50 cm
(C) the maximum length of water column to create the resonance is 95 cm
(D) none of these.
18. A source of sound moves along a circle of radius 2m with constant angular velocity 40 rad/s. Frequency
of the source is 300 Hz. A detector is kept at some distance from the circle in the same plane of the circle
(as shown in figure). Which of the following is not the possible value of frequency registered by the
detector? (Speed of sound = 320 m/s)

(A) 250 Hz (B) 360 Hz (C) 410 Hz (D) 220 Hz.

Page # 190
AVES ON STRING AND SOUND WAVES PHYSICS PART - IV

19. A source of sound and an observer both move relative to the ground. The frequency of the sound heard
by the observer is less than the frequency of source when
(A) the source recedes from the observer
(B) both the source and the observer move away from each other
(C) velocity of wind is parallel to the velocity of the observer
(D) velocity of wind is anti-parallel to the velocity of the source.
20. When beats are produced by two progressive waves of nearly the same frequency, which one of the
following is incorrect ?
(A) the particles vibrate simple harmonically, with the frequency equal to the difference in the
component frequencies.
(B) the amplitude of vibration at any point changes simple harmonically with a frequency equal to the
difference in the frequencies of the two waves
(C) the frequency of beats depends upon the position, where the observer is
(D) the frequency of beats changes as the time progresses.
21. A standing wave of time period T is set up in a string clamped between two rigid supports. At t =0
antinode is at its maximum displacement 2A.
(A) The energy of a node is equal to energy of an antinode for the first time at t = T/8.
(B) The energy of a node is equal after every T/2 second.

T
(C) The displacement of the particle of antinode at t  is 2A .
8
(D) The displacement of the particle of node is zero.

6
22. A traveling wave pulse is given by y  2  x  3t 2 where symbols have their usual meanings, x, y are in
 
metre and t is in second. Then
(A) the pulse is traveling along +ve x-axis with velocity 3 m/s
(B) the pulse is traveling along –ve x-axis with velocity 3 m/s
(C) the amplitude of the wave pulse is 3m
(D) the pulse is a symmetric pulse.
23. The velocity of sound in air increases with increase in
(A) pressure (B) temperature (C) moisture (D) none of the above.
24. Under similar conditions of temperature and pressure, in which the following gases the velocity of
sound will be largest.
(A) H2 (B) N2 (C) He (D) CO2.
25. For a certain transverse standing wave on a long string, an antinode is formed at x = 0 and next to it, a node
is formed at x = 0.10 m. The displacement y(t) of the string particle at x = 0 is shown in figure.

4
y(cm)

t(s)
0.05 0.1 0.15 0.2

–4

Page # 191
AVES ON STRING AND SOUND WAVES PHYSICS PART - IV

(A) transverse displacement of the particle at x = 0.05 and t = 0.05 s is 2 2 cm

(B) transverse displacement of the particle at x = 0.04 and t = 0.025 s is 2 2 cos36º cm


(C) speed of the travelling waves that interfere to produce this standing wave is 2 m/s

1
(D) the transverse velocity of the string particle at x  m and t = 0.1 s is 20  cm/s.
15
26. Sinusoidal waves 5.00 cm in amplitude are to be transmitted along a string having a linear mass density
equal to 4.00 × 10–2 kg/m. If the source can deliver a maximum power of 90 W and the string is under a
tension of 100 N, then the frequency at which the source cannot operate is (take 2 = 10)
(A) 45.3 Hz (B) 50 Hz (C) 30 Hz (D) 62.3 Hz
27. In a stationary wave
(A) particles at all antinodes vibrate in the same phase.
(B) the energy does not transfer across antinodes
(C) all particles between consecutive nodes vibrate in the same phase.
(D) all particles between consecutive antinodes vibrate in the same phase.

 
28. A wave disturbance in a medium is described by y(x, t) = 0.02 cos  50t   cos 10x  where x and y aree
 2
in meter and t is in second. Then
(A) First node occurs at x = 0.15m (B) First antinode occurs at x = 0.3 m
(C) the speed of interfering waves is 5.0 m/s (D) the wavelength is 0.2 m.
29. A wire of density 9 × 103 kg/m3 is stretched between two clamps 1 m apart and is stretched to an extension
of 4.9 × 10-4 metre. Young’s modulus of material is 9 × 1010 N/m2. Then
(A) the lowest frequency of standing wave is 35 Hz
(B) the frequency of 1st overtone is 70 Hz
(C) the frequency of 1st overtone is 105 Hz
(D) the stress in the wire is 4.41 × 107 N/m2.
30. A wire, under tension between two fixed points A and B, executes transverse vibrations so that the
midpoint O of AB is a node. Then
A B
O

(A) all points of wire between A and B are in the same phase
(B) all points between A and O are in the same phase
(C) a point between A and O and a point between O and B may have a phase difference of  / 2
(D) a point between A and O and a point between O and B may have a phase difference of  .
31. Which of the following equations does not represent a progressive wave.
(A) y = A sin k (x + vt) (B) y = f (x – vt) (C) y = A log (x – vt) (D) y = f (x3 – v3 t3).
32. An organ pipe closed at one end resonates with frequency 180 Hz and 300 Hz. It will also resonate with
tuning fork of frequency
(A) 360 Hz (B) 420 Hz (C) 480 Hz (D) 540 Hz.

Page # 192
AVES ON STRING AND SOUND WAVES PHYSICS PART - IV

33. A triangular pulse is moving with speed 2 cm/s along a rope (kept along x-axis) whose one end is free at
x = 0 as shown in the figure. Choose the correct option regarding this pulse.

1 cm

O B X=0
A
2 cm 1 cm 1 cm

2 cm 2 cm
(A) 0.5 cm (B)
2cm x=0 At t = 1 s 2cm x=0 At t = 1 s

(C) particle speed (between A to O) at t = 0 s is 1 cm/s


(D) particle speed (between A to O)at t = 0 s is 2 m/s.

PASSAGE BASED QUESTIONS

I. Speed of a transverse wave depends on mass and tension. Two strings of equal lengths are joined at B.
Mass of string BC is four times mass of string AB. If a wave pulse is generated in string A(B) which travels
towards boundary at B with speed v. Equation of incident pulse is given as
y i  A i sin(t  kx)
Based on above information, answer the following questions.

Tension = constant

v 1 2
B
C
A
x=0 x=L x=2L

1. Amplitude of wave reflected back after incident on boundary at point B

Ai 2A i Ai 2A i
(A)  (B)  (C) (D)
3 3 3 3
2. Speed of transmitted wave on string BC is
v
(A) v (B) (C) 2v (D) None of these.
2
3. Equations of reflected and transmitted waves respectively are :

Ai Ai
yr   sin(t  kx) yr  sin(t  kx)
3 3
(A) 2A (B) 2A
y t   i sin(t  2kx) y t  i sin( t  2kx)
3 3

Page # 193
AVES ON STRING AND SOUND WAVES PHYSICS PART - IV

Ai Ai
yr   sin( t  kx) yr   sin( t  kx)
3 3
(C) 2A (D) 2A k .
y t  i sin( t  2kx) y t  i sin( t  x)
3 3 2

II. A narrow tube is bent in the form of a circle of radius R, as shown in the figure. Two small holes S and D
are made in the tube at the positions right angle to each other. A source placed at S generates a wave of
intensity I0 which is equally divided into two parts: one part travels along the longer path, while the other
travels along the shorter path. Both the part waves meet at the point D where a detector is placed.

R
S

D
4. If a maxima is formed at a detector then, the magnitude of wavelength  of the wave produced is given
by
R R
(A) R (B) (C) (D) all of these
2 4
5. If a minima is formed at the detector then, the magnitude of wavelength  of the wave produced is given
by

3 4
(A) 2R (B) R (C) R (D) None of these
2 5

6. The maximum intensity produced at D is given by


(A) 4I0 (B) 2I0 (C) I0 (D) 3I0

7. The maximum value of  to produce a maximum at D is given by


y

R 3
(A) R (B) 2R (C) (D) R
2 2

III. A point sound source is situated in a medium of bulk modulus 1.6 × 105 N/m2. An observer standing at a
distance 10 m from the source, writes down the equation for the wave as y = A sin (15 x – 6000 t ); y
and x are in metre and t is in second. The maximum pressure amplitude received to the observer’s ear is
24  Pa, then
8. The density of the medium is
1
(A) 1 kg/m3 (B) 2 kg/m3 (C) 3 kg/m3 (D) kg/m3
2
9. If another sound wave of the same form and having same phase is emitted from some other point the
intensity of sound at a point which is equidistant from both the sources is
(A) 0.72 2 (B) 1.44 2 (C) 2.16 2 (D) 2.88 2

Page # 194
AVES ON STRING AND SOUND WAVES PHYSICS PART - IV

MATCH THE COLUMN TYPE QUESTIONS

1. A source of sound in moving along a circular orbit of radius 3 m with angular velocity of
10 rad s–1. A sound detector located for away is executing linear SHM with amplitude 6 m on line BCD as
5
shown. The frequency of detector for oscillation is   per second. The source is at A when detector at
 
B at t = 0. Source emits a continuous sound wave of frequency 340 Hz. (velocity of sound = 330 ms–1).
Match the column A with B.

M
A B C D

Column A Column B
(A) The frequency of sound recorded by (p) 255 H.
detector at t = 3T/4
(B) The frequency of sound recorded by (q) 1 : 1
detector at t = T/4
(C) The ratio of the time period of source (r) 442 Hz
and detector (circular motion and SHM)
(D) Maximum velocity of detector/ (s) 2:1
maximum velocity of source
(T is time period of oscillation).

2. Match the following :


Column A Column B

v
(A) Stiff string has N segments (p) All harmonic of exist
2

v
(B) Open pipe of smaller diameter (q) Fundamental frequency,  
2

2
(C) Closed pipe (r) Wave length 
N
(D) Antinode at ends (s) Odd harmonics are only available
(  = length of string / pipe, v = velocity of wave)

3. Match the following :


Column A Column B
(A) Phase difference between any two (p) Stationary waves
particles can have any value between
O to 2
(B) Energy is transferred from one place (q) Travelling waves
to other place

Page # 195
AVES ON STRING AND SOUND WAVES PHYSICS PART - IV

(C) Phase difference between any two (r) Sound waves


particles is either zero or 
(D) Amplitude of vibration of all particles (s) Standing waves in an open organ pipe.
are equal

INTEGER TYPE QUESTIONS

1. Three sound sources of equal amplitude and frequencies. v  1 , v and v  1 are sounded together. How w
many beats are observed?
2. Transewers wave is propagating in a string. Tension in the string is increased to twice the initial tension.
Simultaceonly, area of cross-section of the string is increased so that there is no change in speed of the
wave. Intial cross-section area is A0 . Final crossection area is KA0 . Value of K is

3. A transeverse wave is tranveling in a string at any moment a small element ' dx ' is at inclination 30o with
the direction of propagation of the wave. After some time interval its inclination changes to 60o with

direction of propagation. Potential energy of this small element is initially U 0 and finally it is K . U 0 ,
value of K is
4. A closed organ pipe is vibrating in fundamental frequency. There are two

points A and B in the organ pipe as shown, at a distance AB  L n .

Ratio of maximum pressure variation at point A to point B is 2 / 3


find value of n

5. A cylindrical tube, open at both ends, has a fundamental frequency  . The tube is dipped vertically in water
so that of its length is inside the water. Ratio of new fundamental frequency to old fundamental frequency is
6. The loudness level of sound at a point is increased by 30dB. Ratio of final pressure to initial pressure is
2k value of k is .
7. A source emitting sound of frequency 180Hz is placed in front of a wall at a distance of 2m from it. A
detector is also placed in front of the wall at some distance from it. Find the minimum distance (in meter)
between the source and the detector for which the detector detects a maximum of sound speed of sound
in air = 360 m/sec
8. A train approaching a railway crossing at a speed of 120 km/hr sounds a short whistle at frequency
f 0  640 Hz .when it is 300 m away from the crossing. The speed of sound in air is 340 m/sec. A person
standing on a road perpendicular to the track through the corssing at a distance 400 m from the crossing,
hears frequency f1 value f1  f 0  10 x . Value of x is.
9. A guiter string is 90 cm long nad has a fundamental frequency of 124 Hz. It is pressed at a distance
10x cm from one end to produce a fundamental frequency of 186Hz. Value of x is.
10. A wire having a linear mass density 5.0  103 kg / m is streched between two rigid supports with a
tension of 450N. The wire resonates at a frequency of 420Hz. The next higher frequency at which the
same wire resonates is (420 + 10x) Hz. Length of wire is 2.1 m. Value of x is.

Page # 196
AVES ON STRING AND SOUND WAVES PHYSICS PART - IV

Questions asked in previous AIEEE / JEE MAINS


1. Tube A has both ends open while tube B has one end closed, otherwise they are identical. The ratio of
fundamental frequency of tubes A and B is : [AIEEE 2002]
(A) 1 : 2 (B) 1: 4 (C) 2 : I (D) 4 : I
2. A tuning fork arrangement (pair) produces 4 beats/sec with one fork of frequency 288 cps. A little Wax
is placed on the unknown fork and it then produces 2 beats/sec. The frequency of the unknown fork is :
[AIEEE 2002]

(A) 286 cps (B) 292 cps (C) 294 cps (D) 288 cps
3. When temperature increases, the frequency of a tuning fork : [AIEEE 2002]
(A) increases (B) decreases
(C) remain same (D) increases or decreases depending on the material
4. A tuning fork of known frequency 256 Hz. makes 5 betas per second with the vibrating string of a piano.
The beat frequency decreases to 2 beats per second when the tension in the piano string is slightly
increased. The frequency of the piano string before increasing the tension was : [AIEEE 2003]
(A) (256 + 2) Hz. (B) (256 – 2) Hz (C) (256 – 5) Hz (D) (256 + 5) Hz
5. When two tuning forks (fork 1 and fork 2) are sounded simultaneously, 4 beats per second are heard.
Now, some tape is attached on the prong of the fork 2. When the tuning forks are sounded again, 6 beats
per second are heard. If the frequency of fork 1 is 200 Hz, then what was the original frequency of fork
2? [AIEEE 2005]
(A) 200 Hz (B) 202 Hz (C) 196 Hz (D) 204 Hz
6. An observer moves towards a stationary source of sound, with a velocity one-fifth of the velocity of
sound. What is the percentage increase in the apparent frequency? [AIEEE 2005]
(A) Zero (B) 0.5% (C) 5% (D) 20%
7. A whistle producing sound waves of frequencies 9500 Hz and above is approaching a stationary person with
speed v ms–1. The velocity of sound in air is 300 ms–1. If the person can hear frequencies upto a maximum of
10,000 Hz, the maximum value of v upto which he can hear the whistle is: [AIEEE 2006]
(A) 30 ms–1 (B) 15 2 ms–1 (C) 15 2 ms–1 (D) 15 ms–1

8. A sound absorber attenuates the sound level by 20 dB. The intensity decreases by a factor of :
[AIEEE 2007]
(A) 1000 (B) 10000 (C) 10 (D) 100
9. The speed of sound in oxygen (O2) at a certain temperature is 460 ms–1. The speed of sound in helium
(He) at the same temperature will be (assume both gases to be ideal) : [AIEEE 2008]
(A) 500 ms–1 (B) 650 ms–1 (C) 330 ms–1 (D) 1419 m/s
10. While measuring the speed of sound by performing a resonance column experiment, a student gets the
first resonance condition at a column length of 18 cm during winter. Repeating the same experiment
during summer, she measures the column length to be x cm for the second resonance. Then
[AIEEE 2008]
(A) x > 54 (B) 54 > x > 36 (C) 36 > x > 18 (D) 18 > x
11. A motor cycle starts from rest and accelerates along a straight path at 2 m/s2. At the starting point of the
motor cycle there is a stationary electric siren. How far has the motor cycle gone when the driver hears
the frequency of the siren at 94% of its value when the motor cycle was at rest? (Speed of sound = 330
ms–1) [AIEEE 2009]
(A) 98 m (B) 147 m (C) 196 m (D) 49 m

Page # 197
AVES ON STRING AND SOUND WAVES PHYSICS PART - IV

1. A wave y = a sin (t - kx) on a string meets with another wave producing a node at x = O. Then the equation
of the unknown wave is : [AIEEE - 2002]
(A) y = a sin (t + kx) (B) y = -a sin (t + kx) (C) y = a sin (t - kx) (D) y = -a sin (t - kx)
2. Length of a string tied to two rigid supports is 40 cm. Maximum length (wavelength in cm) of a stationary
wave produced on it, is - [AIEEE - 2002]
(A) 20 (B) 80 (C) 40 (4) 120
3. The displacement y of a wave travelling in the x-direction is given by [AIEEE 2003]

 
y = 10–4 sin  600 t  2x  3  metre,
 

where x is expressed in metres and t in seconds. The speed of the wave-motion, in ms–1 is :
(A) 300 (B) 600 (C) 1200 (D) 200
4. A metal wire of linear mass density of 9.8 g/m is stretched with a tension of 10 kg-wt between two rigid
supports 1 metre apart. The wire passes at its middle point between the poles of a permanent magnet and
it vibrates in resonance when carrying an alternating current of frequency n. The frequency n of the
alternating source is : [AIEEE 2003]
(A) 50 Hz (B) 100 Hz (C) 200 Hz (D) 25 Hz
5. The displacement y of a particle in a medium can be expressed as : [AIEEE 2004]


y = 10–6 sin (100t + 20x + )m, where t is in second and x in metre. The speed of the wave is:
4
(A) 2000 m/s (B) 5 m/s (C) 20 m/s (D) 5 m/s
6. A string is stretched between fixed points separated by 75 cm. It is observed to have resonant frequen-
cies of 420 Hz and 315 Hz. There are no other resonant frequencies between these two. Then, the lowest
resonant frequency for this string is [AIEEE 2006]
(A) 10.5 Hz (B) 105 Hz (C) 1.05 Hz (D) 1050 Hz
7. A wave travelling along the x- axis is described by the equation y(x,t) = 0.005 cos (ax –bt). If the
wavelength and the time period of the wave are 0.08 m and 2.0 s, respectively, then a and b in appropriate
units are [AIEEE 2008]

0.08 2.0 0.04 1.0


(A) a  ,b  (B) a  ,b 
   


(C) a  12.50 , b  (D) a = 25.00 p, b = 
2.0
8. The equation of a wave on a string of linear mass density 0.04 kg m–1 is given by y = 0.02 (m) sin
  t x 
2π    . The tension in the string is : [AIEEE 2010]
  0.04(s) 0.50(m) 

(A) 4.0 N (B) 12.5 N (C) 0.5 N (D) 6.25 N


9. The transverse displacement y(x,t) of a wave on a string is given by

y( x, t )  e  ax  This represents a :


2
 bt 2  2 ab xt [AIEEE - 2011]

a b
(A) wave moving in +x–direction with speed (B) wave moving in –x-direction with speed
b a

1
(C) standing wave of frequency b (D) standing wave of frequency
b

Page # 198
AVES ON STRING AND SOUND WAVES PHYSICS PART - IV

10. A travelling wave represented by y = A sin (t – kx) is superimposed on another wave represented by y
= A sin (t + kx). The resultant is : [AIEEE 2011]
(A) A wave travelling along + x direction
(B) A wave travelling along – x direction
n
(C) A standing wave having nodes at x = , n = 0, 1, 2 ....
2
 1 
(D) A standing wave having nodes at x =  n  2  2 ; n = 0, 1, 2 .....
 
11. Statement - 1 : [AIEEE 2011]
Two longitudinal waves given by equations : y1 (x, t) = 2a sin (t – kx) and y2 (x, t) = a sin (2t – 2kx) will
have equal intensity.
Statement - 2:
Intensity of waves of given frequency in same medium is proportional to square of amplitude only.
(A) Statement-1 is true, statement-2 is false.
(B) Statement-1 is true, statement-2 is true, statement-2 is the correct explanation of statment-1
(C) Statement-1 is true, statement-2 is true, statement-2 is not the correct explanation of statement-1
(D) Statement-1 is false, statement-2 is true.
12. A sonometer wire of length 1.5 m is made of steel. The tension in it produces an elastic strain of 1 %. What is
the fundamental frequency of steel if density and elasticity of steel are 7.7 × 103 kg/m 3 and 2.2 × 1011N/m 2
respectively? [JEE Mains - 2013]
(A) 178.2 Hz (B) 200.5 Hz (C) 770 Hz (D) 188.5 Hz
13. A pipe of length 85 cm is closed from one end. Find the number of possible natural oscillations of air
column in the pipe whose frequencies lie below 1250 Hz. The velocity of sound in air is 340 m/s.
[JEE Mains - 2014]
(A) 6 (B) 4 (C) 12 (D) 8
14. A train is moving on a straight track with speed 20 ms–1. It is blowing its whistle at the frequency of 1000
Hz. The percentage change in the frequency heard by a person standing near the track as the train passes
him is (speed of sound = 320 ms–1) close to : [JEE Mains - 2015]
(A) 12% (B) 18% (C) 24% (D) 6%
Questions asked in previous IIT JEE/JEE ADVANCED
1. As a wave propagates :
(A) the wave intensity remains constant for a plane wave
(B) the wave intensity decreases as the inverse of the distance from the source for a spherical wave
(C) the wave intensity decreases as the inverse square of the distance from the source for a spherical
wave
(D) total power of the spherical wave over the spherical surface centered at the source remains constant
at all times . [JEE - 99]
2. In a wave motion y = a sin (kx t), y can represent : [JEE - 99]
(A) electric field (B) magnetic field (C) displacement (D) pressure change
3. The ratio of speed of sound in nitrogen gas to that in helium gas at 300 K is [JEE - 99]

(A) 2/7 (B) 1/ 7 (C) 3 / 5 (D) 6/5

Page # 199
AVES ON STRING AND SOUND WAVES PHYSICS PART - IV

4. A closed pipe and an open pipe have their first overtones identical in frequency. Their lengths are in the
ratio
(A) 1 : 2 (B) 2 : 3 (C) 3 : 4 (D) 4 : 5
5. Two monoatomic ideal gases 1 and 2 of molecular masses m 1 and m2 respectively are enclosed in
separate containers kept at the same temperature. The ratio of the speed of sound in gas 1 to that in
gas 2 is given by [JEE (Scr) - 2000]

m1 m2 m1 m2
(A) m2 (B) m1 (C) m (D) m
2 1

6. A train moves towards a stationary observer with speed 34m/s. The train sounds a whistle and its
frequency registered by the observer is f 1. If the train’s speed is reduced to 17m/s, the frequency
registered is f2. If the speed of sound is 340m/s then the ratio f 1/f2 is [JEE - 2000]
(A) 18/19 (B) 1/2 (C) 2 (D) 19/18
7. A 3.6 m long vertical pipe resonates with a source of frequency 212.5 Hz when water level is at
certain heights in the pipe. Find the heights of water level (from the bottom of the pipe) at which
resonances occur. Neglect end correction. Now, the pipe is filled to a height H (~ 3.6 m). A small hole
is drilled very close to its bottom and water is allowed to leak. Obtain an expression for the rate of
fall of water level in the pipe as a function of x . If the radii of the pipe and the hole are 2  102 m and
1  103 m respectively, calculate the time interval between the occurrence of first two resonances.
Speed of sound in air is 340 m/s and g = 10 m/s 2 . [JEE - 2000 Mains]
8. A source of sound revolving in a circle of radius 15 m is emitting a signal of frequency 200 Hz. It
completes one revolution in 3 seconds . Calculate the maximum and minimum frequencies of the
signal heard at a point 30 m from the centre of the circle . [REE - 2000 Mains]
9. It is desired to increase the fundamental resonance frequency in a tube which is closed at one end. This
can be achieved by [REE - 2000]
(A) replacing the air in the tube by hydrogen gas (B) increasing the length of the tube
(C) decreasing the length of the tube (D) opening the closed end of the tube
10. A boat is travelling in a river with a speed 10 m/sec along the stream flowing with a speed
2 m/sec . From this boat, a sound transmitter is lowered into the river through a rigid support. The
wavelength of the sound emitted from the transmitter inside the water is 14.45 mm. Assume that
attenuation of sound in water and air is negligible ,
(a) What will be the frequency detected by a receiver kept inside the river downstream ?
(b) The transmitter and the receiver are now pulled up into air . The air is blowing with a speed 5
m/sec in the direction opposite the river stream. Determine the frequency of the sound detected by
the receiver.
[Temperature of the air and water = 20º C ;
Density of river water = 103 kg/m3
Bulk modulus of the water = 2.088  109 Pa ;
Gas constant R = 8.31 J/mol-K
Mean molecular mass of air = 28.8  10 3 Kg/mol ;
CP/CV for air = 1.4 ]
[ velocity of boat was assumed to be w.r.t. ground ] [JEE - 2001 (Mains)]
11. A source S emitting sound of 300 Hz is fixed on block A which is attached to the free end of a spring
SA as shown in figure. The detector D fixed on block B attached to free end of spring S B detects this
sound. The blocks A and B are simultaneously displaced towards each other through a distance of
1.0m and then left to vibrate. Find the maximum and minimum frequencies of sound detected by D
if the vibrational frequencies of each block is 2 Hz. speed of sound in air = 340 m/s
[REE - 2001]
Page # 200
AVES ON STRING AND SOUND WAVES PHYSICS PART - IV

12. A siren placed at a railway platform is emitting sound of frequency 5 kHz. A passenger sitting in a
moving train A records a frequency of 5.5 kHz, while the train approaches the siren. During his
return journey in a different train B he records a frequency of 6.0 kHz while approaching the same
siren. The ratio of the velocity of train B to that of train A is [JEE - 2002]

242 5 11
(A) (B) 2 (C) (D)
252 6 6
13. Two narrow cylindrical pipes A and B have the same length. Pipe A is open at both ends and is filled
w it h a monoat omic gas of molar mass M A. Pipe B is open at one end and closed at the other end , and
is filled with a diatomic gas of molar mass M B . Both gases are at the same temperature.
(a) If the frequency of the second harmonic of the fundamental mode in pipe A is equal to the
MA
frequency of the third harmonic of the fundamental mode in pipe B, determine the value of M .
B

(b) Now the open end of pipe B is also closed (so that the pipe is closed at both ends). Find the
ratio of the fundamental frequency in pipe A to that in pipe B. [JEE - 2002 (Mains)]
14. A police car moving at 22 m/s, chases a motorcyclist. The police man sounds his horn at 176 Hz, while
both of them move towards a stationary siren of frequency 165 Hz. Calculate the speed of the motorcycle,
if it is given that he does not observe any beats. [JEE-2003]
(A) 33 m/s (B) 22 m/s (C) zero (D) 11 m/s
15. In the experiment for the determination of the speed of sound in air using the resonance column method,
the length of the air column that resonates in the fundamental mode, with a tuning fork is 0.1 m. When this
length is changed to 0.35 m, the same tuning fork resonates with first overtone. Calculate the end correc-
tion. [JEE- 2003 (Screening)]
(A) 0.012 m (B) 0.025 m (C) 0.05 m (D) 0.024 m
16. In a resonance tube experiment to determine the speed of sound in air, a pipe of diameter 5 cm is used.
The air column in pipe resonates with a tuning fork of frequency 480 Hz when the minimum length of the
air column is 16 cm. Find the speed of sound in air at room temperature. [JEE - 2003]
17. A source of sound of frequency 600 Hz is placed inside water. The speed of sound in water is 1500 m/s
and in air it is 300 m/s. The frequency of sound recorded by an observer who is standing in air is :
[JEE- 2004 (screening)]
(A) 200 Hz (B) 3000 Hz (C) 120 Hz (D*) 600 Hz
18. A closed organ pipe of length L and an open organ pipe contain gases of densities 1 and 2 respectively.
The compressibility of gases are equal in both the pipe. Both the pipes are vibrating in their first overtone
with same frequency. The length of the open organ pipe is : [JEE- 2004 (screening)]

L 4L 4L 1 4L  2
(A) (B) (C*) 3 2 (D) 3 1
3 3

19. A train is passing a stationary observer at station with constant velocity. If the frequency observed by the
person during its approach and recession are 2.2 kHz and 1.8 kHz respectively. Then find the velocity of
train if the velocity of sound in air is 300 m/s. [JEE 2005 (Main)]

Page # 201
AVES ON STRING AND SOUND WAVES PHYSICS PART - IV

20. An open organ pipe is in resonance in its 2nd harmonic with a tuning fork of frequency f1. Now it is closed
at one end. If the frequency of the tuning fork is increased slowly from f1 then again a resonance is
obtained when the frequency is f2. If in this case the pipe vibrates in nth harmonic then
[JEE- 2005 (Screening)]

3 5 5 3
(A) n = 3, f2 = f (B) n = 3, f2 = f (C) n = 5, f2 = f (D) n = 5, f2 = f
4 1 4 1 4 2 4 1
Paragraph for question Nos. 21 to 23 :
Two plane harmonic sound waves are expressed by the equations. [JEE' 2006]
y1(x, t) = A cos (0.5 x – 100 t)
y2(x, t) = A cos (0.46 x – 92 t)
(All parameters are in MKS) :
21. How many times does an observer hear maximum intensity in one second ?
(A) 4 (B) 10 (C) 6 (D) 8
22. What is the speed of the sound ?
(A) 200 m/s (B) 180 m/s (C) 192 m/s (D) 96 m/s
23. At x = 0 how many times y1 + y2 is zero in one second ?
(A) 192 (B) 48 (C) 100 (D) 96

Paragraph for Question Nos. 24 to 26 [JEE' 2007]


Two trains A and B are moving with speeds 20 m/s and 30 m/s respectively in the same direction on the
same straight track, with B ahead of A. The engines are at the front ends. The engines of train A blows a
long whistle. Assume that the sound of the whistle is composed of components varying in frequency
from f1 = 800 Hz to f2 = 1120 Hz, as shown in the figure. The spread in the frequency (highest frequency
– lowest frequency) is thus
320 Hz. The speed of sound in still air is 340 m/s.
Intensity

24. The speed of sound of the whistle is


(A) 340 m/s for passengers in A and 310 m/s for passengers in B
(B) 360 m/s for passengers in A and 310 m/s for passengers in B f1 f2 frequency
(C) 310 m/s for passengers in A and 360 m/s for passengers in B
(D) 340 m/s for passengers in both the trains
25. The distribution of the sound intensity of the whistle as observed by the passengers in train A is best
represented by
Intensity
Intensity

(A) (B)

f1 f2 frequency f1 f2 frequency
Intensity
Intensity

(C) (D)
f1 f2 frequency f1 f2 frequency

Page # 202
AVES ON STRING AND SOUND WAVES PHYSICS PART - IV

26. The spread of frequency as observed by the passengers in train B is


(A) 310 Hz (B) 330 Hz (C) 350 Hz (D) 290 Hz
27. A vibrating string of certain length  under a tension T resonates with a mode corresponding to the first
overtone (third harmonic) of an air column of length 75 cm inside a tube closed at one end. The string also
generates 4 beats per second when excited along with a tuning fork of frequency n. Now when the
tension of the string is slightly increased the number of beats reduces to 2 per second. Assuming the
velocity of sound in air to be 340 m/s, the frequency n of the tuning fork in Hz is
[JEE' 2008]
(A) 344 (B) 336 (C) 117.3 (D) 109.3
28. A student performed the experiment to measure the speed of sound in air using resonance air-column
method. Two resonances in the air-column were obtained by lowering the water level. The resonance
with the shorter air-column is the first resonance and that with the longer air-column is the second
resonance. Then, [JEE' 2009]
(A) the intensity of the sound heard at the first resonance was more than that at the second resonance
(B) the prongs of the tuning fork were kept in a horizontal plane above the resonance tube
(C) the amplitude of vibration of the ends of the prongs is typically around 1 cm
(D) the length of the air-column at the first resonance was somewhat shorter than 1/4th of the wave-
length of the sound in air.

29. A stationary source is emitting sound at a fixed frequency f0, which is reflected by two cars approaching
the source. The difference between the frequencies of sound reflected from the cars is 1.2% of f0. What
is the difference in the speeds of the cars (in km per hour) to the nearest integer ? The cars are moving at
constant speeds much smaller than the speed of sound which is 330 ms–1. [JEE' 2010]
30. A hollow pipe of length 0.8 m is closed at one end. At its open end a 0.5 m long uniform string is vibrating
in its second harmonic and it resonates with the fundamental frequency of the pipe. If the tension in the
wire is 50 N and the speed of sound is 320 ms–1, the mass of the string is : [JEE' 2010]
(A) 5 grams (B) 10 grams (C) 20 grams (D) 40 grams
31. A police car with a siren of frequency 8 kHz is moving with uniform velocity 36 km/hr towards a tall
building which reflects the sound waves. The speed of sound in air is 320 m/s. The frequency of the siren
heard by the car driver is [JEE' 2011]
(A) 8.50 kHz (B) 8.25 kHz (C) 7.75 kHz (D) 7.50 kHz
32. Column I shows four systems, each of the same length L, for producing standing waves. ‘ The lowest
possible natural frequency of a system is called its fundamental frequency, whose wavelength is denoted
as f. Match each system with statements given in Column II describing the nature and wavelength of the
standing waves. [JEE' 2011]
Column I Column II
(A) Pipe closed at one end (p) Longitudinal waves

(B) Pipe open at both ends (q) Transverse waves

(C) Stretched wire clamped at both ends (r) f = L

Page # 203
AVES ON STRING AND SOUND WAVES PHYSICS PART - IV

(D) Stretched wire clamped at both ends (s) f = 2L


and at mid-point

33. Two vehicles, each moving with speed u on the same horizontal straight road, are approaching each
other. Wind blows along the road with velocity w. One of these vehicles blows a whistle of frequency f1
. An observer in the other vehicle hears the frequency of the whistle to be f2 . The speed of sound in still
air is V. The correct statement(s) is (are) [JEE Advanced-2013]
(A) If the wind blows from the observer to the source, f2 > f1 .
(B) If the wind blows from the source to the observer, f2 > f1.
(C) If the wind blows from observer to the source, f2 < f1 .
(D) If the wind blows from the source to the observer f2 < f1 .

34. Using the expression 2d sin    , one calculates the values of d by measuring the corresponding angles
 in the range 0 to 90°. The wavelength  is exactly known and the error in  is constant for all values
ues
of  . As  increases from 0°, [JEE Advanced-2013]
(A) the absolute error in d remains constant. (B) the absolute error in d increases
(C) the fractional error in d remains constant. (D) the fractional error in d decreases.
35. A person blows into open-end of a long pipe. As a result, a high pressure pulse of air travels down the pipe.
When this pulse reaches the other end of the pipe, [JEE Advanced-2013]
(A) a high-pressure pulse starts travelling up the pipe, if the other end of the pipe is open.
(B) a low-pressure pulse starts travelling up the pipe, if the other end of the pipe is open.
(C) a low-pressure pulse starts travelling up the pipe, if the other end of the pipe is closed.
(D) a high-pressure pulse starts travelling up the pipe, if the other end of the pipe is closed.
36. A student is performing the experiment of resonance Column. The diameter of the column tube is 4 cm.
The frequency of the tuning fork is 512 Hz. The air temperature is 38C in which the speed of sound is
336 m/s. The zero of the meter scale coincides with the top end of the Resonance Column tube. When the
first resonance occurs, the reading of the water level in the column is [JEE Advanced-2013]
(A) 14.0 cm (B) 15.2 cm (C) 16.4 cm (D) 17.6 cm

Wave On A String
1. y (x, t) = 0.8/[(4x + 5t)2 + 5] represents a moving pulse, where x & y are in meter and t in second. Then :
[JEE - 99]
(A) pulse is moving in +x direction (B) in 2s it will travel a distance of 2.5 m
(C) its maximum displacement is 0.16 m (D) it is a symmetric pulse

2. Standing waves can be produced : [JEE - 99]


(A) on a string clamped at both the ends
(B) on a string clamped at one end and free at the other
(C) when incident wave gets reflected from a wall
(D) when two identical waves with a phase difference of  are moving in same direction

Page # 204
AVES ON STRING AND SOUND WAVES PHYSICS PART - IV

3. A long wire PQR is made by joining two wires PQ and QR of equal radii. PQ has length 4.8 m and mass 0.06
kg. QR has length 2.56 m and mass 0.2kg. The wire PQR is under a tension of 80N. A sinusoidal wavepulse
of amplitude 3.5cm is sent along the wire PQ from the end P. No power is dissipated during the propagation
of the wavepulse. Calculate [JEE - 99
mains]
(a) the time taken by the wavepulse to reach the other end R of the wire, and
(b) the amplitude of the reflected and transmitted wavepulses after the incident wavepulse crosses the
joint Q.
4. Two metallic strings A and B of different materials are connected in series forming a joint. The strings
have similar crosssectional area. The length of A is A = 0.3m and that B is B = 0.75m. One end of the
combined string is tied with a support rigidly and the other end is loaded with a block of mass m passing
over a frictionless pulley. Transverse waves are set up in the combined string using an external source of
variable frequency. Calculate [REE - 99]
(i) the lowest frequency for which standing waves are observed such that the joint is a node and
(ii) the total number of antinodes at this frequency. The densities of A & B are 6.3 x 103kg m3 and 2.8 x
103kg m3 respectively.
5. Two vibrating strings of the same material but lengths L & 2 L have radii 2 r and r respectively. They
are stretched under the same tension. Both the strings vibrate in their fundamental modes, the one
of length L with frequency f 1 and the other with frequency f 2. The ratio f1/f2 is given by :
[JEE - 2000 Screening]
(A) 2 (B) 4 (C) 8 (D) 1
6. A wave pulse starts propagating in the + x direction along a non uniform wire of length 10 m with
mass per unit length given by  = 0 +  x and under a tension of 100 N . Find the time taken by the
pulse to travel from the lighter end (x = 0) to the heavier end. ( 0 = 10 2 kg/m and  = 9  10 3 kg/
m 2) [REE - 2000 Mains, 6]
7. Two sinusoidal waves with same wavelengths and amplitude travel in opposite directions along a string
with a speed 10 ms–1. If the minimum time interval between instants when the string is flat is 0.5s, the
wavelength of the waves is : [REE - 2000]
(A) 25 m (B) 20 m (C) 15 m (D) 10 m
8. A longitudinal travelling wave transports [REE - 2000]
(A) energy and linear momentum (B) energy and angular momentum
(C) energy and torque (D) angular momentum and torque
9. The intensity of a progressing plane wave in loss-free medium is [REE - 2000]
(A) directly proportional to the square of amplitude of the wave
(B) directly proportional to the velocity of the wave
(C) directly proportional to the square of frequency of the wave
(D) inversely proportional to the density of the medium.
10. The ends of a stretched wire of length L are fixed at x = 0 & x = L . In one experiment the displacement
of the wire is y1 = A sin ( x/L) sin  t & energy is E1 and in other experiment its displacement is
y2 = A sin (2 x/L) sin 2  t and energy is E2 . Then :
(A) E2 = E1 (B) E2 = 2 E1 (C) E2 = 4 E1 (D) E2 = 16 E1

11. Two symmetrical and identical pulses in a stretched string, whose centers are initially 8 cm apart,
are moving towards each other as shown in the figure. The speed of each pulse is 2 cm/s. After 2
seconds, the total energy of the pulses will be : [JEE - 2001 Screening,]

Page # 205
AVES ON STRING AND SOUND WAVES PHYSICS PART - IV

8 cm

(A) zero (B) purely kinetic


(C) purely potential (D) partly kinetic and partly potential
12. A sonometer wire resonates with a given tuning fork forming standing waves with five antinodes
between the two bridges when a mass of 9 kg is suspended from the wire . When this mass is
replaced by a mass M, the wire resonates with the same tuning fork forming three antinodes for the
same positions of the bridges. The value of M is [JEE - 2002 Screening]
(A) 25 kg (B) 5 kg (C) 12.5 kg (D) 1/25 kg
13. A string of mass ' m ' and length , fixed at both ends is vibrating in its fundamental mode.The maximum
amplitude is ' a ' and the tension in the string is ' T '. Find the energy of vibrations of the string.
[JEE - 2003_mains]
14. A transverse wave travelling in a string produces maximum transverse velocity of 3 m/s and maximum
transverse acceleration 90 m/s2 in a particle. If the velocity of wave in the string is 20 m/s. Determine the
equation of the wave ? [JEE - 2005_mains]

15. A massless rod BD is suspended by two identical massless strings AB and CD of equal lengths. A block of
mass ‘m’ is suspended point P such that BP is equal to ‘x’, if the fundamental frequency of the left wire is
twice the fundamental frequency of right wire, then the value of x is :

[JEE - 2006 mains ]

(A) l/5 (B) l/4 (C) 4l/5 (D) 3l/4


16. A transverse sinusoidal wave moves along a string in the positive x-direction at a speed of 10 cm/s. The
wavelength of the wave is 0.5 m and its amplitude is 10 cm. At a particular time t, the snap-shot of the
wave is shown in figure. The velocity of point P when its displacement is 5 cm is Figure :
y [JEE - 2008]

P
x

3 3 3 3
(A) ĵ m/s (B) – ĵ m/s (C) î m/s (D) – î m/s
50 50 50 50

Page # 206
AVES ON STRING AND SOUND WAVES PHYSICS PART - IV

17. A 20cm long string, having a mass of 1.0 g, is fixed at both the ends. The tension in the string is 0.5 N. The
string is set into vibrations using an external vibrator of frequency 100 Hz. Find the separation (in cm)
between the successive nodes on the string. [JEE - 2009]

 
18. When two progressive waves y1 = 4 sin (2x – 6t) and y2 = 3 sin  2x – 6t – 2  are superimposed, the
 
amplitude of the resultant wave is : [JEE - 2010]
19. A horizontal stretched string fixed at two ends, is vibrating in its fifth harmonic according to the equation
y(x, t) = 0.01m sin [(62.8m-1)x] cos[(628s-1)t]. Assuming  = 3.14, the correct statement(s) is (are)
[JEE Advanced-2013]
(A) The number of nodes is 5.
(B) the length of the string is 0.25 m.
(C) The maximum displacement of the midpoint of the string, from its equilibrium position is 0.01m.
(D) The fundamental frequency is 100 Hz.
20. A student is performing an experiment using a resonance column and a tuning fork of frequency 244 s-
1.He is told that the air in the tube has been replaced by another gas (assume that the column remains
filled with the gas). If the minimum height at which resonance occurs is (0.350 ± 0.005)m, the gas in the
tube is (Useful information: 167RT  640J1/2 mole 1/2 ; 140RT  590J1/2 mole 1/2 , the molar masses M in

10
grams are given in the options. Take the values of for each gas as given there.)
M
[JEE ADVANCED 2014]

 10 7   10 3 
(A) Neon  M  20, 20  10  (B) Nitrogen  M  28, 28  5 
   

 10 9   10 17 
(C) Oxygen  M  32, 32  16  (D) Argon  M  36, 36  32 
   

21. One end of a taut string of length 3m along the x axis is fixed at x = 0. The speed of the waves in the string
is 100 ms–1. The other end of the string is vibrating in the y direction so that stationary waves are set up
in the string. The possible waveform(s) of these stationary waves is (are) [JEE ADVANCED 2014]

x 50t x 100t
(A) y(t)  Asin cos (B) y(t)  A sin cos
6 3 6 3

5x 250t 5x


(C) y(t)  Asin cos (D) y(t)  A sin cos 250t
6 3 2

22. Four harmonic waves of equal frequencies and equal intensities I0 have phase angles 0,  / 3, 2 / 3 and  .
When they are superposed, the intensity of the resulting wave is nI0. The value of n is : [JEE ADV. 2015]
23. A block M hangs vertically at the bottom end of a uniform rope of constant mass per unit length. The top
end of the rope is attached to a fixed rigid support at O. A transverse wave pulse (Pulse 1) of wavelength
0 is produced at point O on the rope. The pulse takes time TOA to reach point A. If the wave pulse of
wavelength 0 is produced at point A (Pulse 2) without disturbing the position of M it takes time TAO to
reach point O. Which of the following options is/are correct ? [JEE ADV 2017]

Page # 207
AVES ON STRING AND SOUND WAVES PHYSICS PART - IV

(A) The time TAO  TOA


(B) The wavelength of Pulse 1 becomes longer when it reaches point A
(C) The velocity of any pulse along the rope is independent of its frequency and wavelength
(D) The velocities of the two pulses (Pulse 1 and Pulse 2) are the same at the midpoint of rope
24. A stationary source emits sound of frequency f0 = 492 Hz. The sound is reflected by a large car
approaching the source with a speed of 2 ms-1. The reflected signal is received by the source and
superposed with the original. What will be the beat frequency of the resulting signal in Hz? (Given that
the speed of sound in air is 330 ms-1 and the car reflects the sound at the frequency it has received).
[JEE ADV 2017]

Page # 208
AVES ON STRING AND SOUND WAVES PHYSICS PART - IV

ANSWER KEY
TRY YOURSELF

(1) (D) (2) (C) (3) (A) (4) (D) (5) (C) (6) (B)
(7) (C) (8) (C) (9) (C) (10) (C) (11) (B) (12) (A)
(13) (C) (14) (A) (15) (A) (16) (B) (17) (A) (18) (C)
(19) (B) (20) (C) (21) (B) (22) (A) (23) (B) (24) (A)
(25) (D) (26) (D) (27) (B) (28) (D) (29) (D) (30) (D)
(31) (A) (32) (C) (33) (A) (34) (A) (35) (C) (36) (D)
(37) (B) (38) (B) (39) (A) (40) (D) (41) (B)

ONLY ONE CHOICE IS CORRECT

1. (D) 2. (A) 3. (C) 4. (D) 5. (D) 6. (A)


7. (B) 8. (B) 9. (D) 10. (D) 11. (A) 12. (D)
13. (D) 14. (C) 15. (D) 16. (C) 17. (A) 18. (B)
19. (A) 20. (C) 21. (D) 22. (D) 23. (C) 24. (C)
25. (C) 26. (A) 27. (D) 28. (A) 29. (D) 30. (B)
31. (D) 32. (A) 33. (B) 34. (C) 35. (D) 36. (B)
37. (D) 38. (C) 39. (A) 40. (B) 41. (D) 42. (C)
43. (B) 44. (B) 45. (C) 46. (B) 47. (C) 48. (D)
49. (A) 50. (D) 51. (C) 52. (C) 53. (B) 54. (C)
55. (C) 56. (C) 57. (A) 58. (A) 59. (B) 60. (C)
61. (A) 62. (C) 63. (A) 64. (A)

ONE OR MORE THAN ONE CHOICE MAY BE CORRECT

1. (A), (B), (C), (D)2. (B), (C), (D) 3. (B), (C)


4. (A), (D) 5. (B) 6. (A, C)
7. (A, D) 8. (C, D) 9. (B, D)
10. (C, D) 11. (B, C, D) 12. (A), (C)
13. (A), (B), (D) 14. (A), (D) 15. (B, C, D)
16. (C, D) 17. (A), (B), (C) 18. (C), (D)
19. (A), (B), (C), (D)20. (A), (C), (D)
21. (A), (C), (D) 22. (B), (C), (D) 23. (B), (C)
24. (A) 25. (A), (B), (C),(D) 26. (A), (B), (D)
27. (B), (C) 28. (C), (D) 29. (A), (B)
30. (B), (D) 31. (C), (D) 32. (B), (D)
33. (A), (C)

PASSAGE BASED QUESTIONS

1. (A) 2. (B) 3. (C) 4. (D) 5. (A) 6. (B)


7. (A) 8. (A) 9. (B)

Page # 209
AVES ON STRING AND SOUND WAVES PHYSICS PART - IV

MATCH THE COLUMN TYPE QUESTIONS

1. (A) – (r), (B) – (p), (C) – (q), (D) – (s)


2. (A) – (p), (q), (r), (B) – (p), (q), (r), (C) – (q), (D) – (p), (q), (r)
3. (A) – (q, r), (B) – (q, r), (C) – (p, s), (D) – (q, r)

INTEGER TYPE QUESTIONS

1. 1 2. 2 3. 9 4. 3 5. 1 6. 5
7. 3 8. 4 9. 6 10. 7

Questions asked in previous AIEEE / JEE MAINS


Wave:
1. (C) 2. (B) 3. (B) 4. (C) 5. (C) 6. (D)
7. (D) 8. (D) 9. (D) 10. (A) 11. (A)
Wave on a String:
1. (B) 2. (B) 3. (A) 4. (B) 5. (B) 6. (B)
7. (D) 8. (D) 9. (B) 10. (D) 11. (A) 12. (A)
13. (A) 14. (A)

Questions asked in previous IIT JEE/JEE ADVANCED

1. (ACD) 2. (ABCD) 3. (C) 4. (c) 5. (B) 6. (D)

7. x = 3.2, 2.4, 1.6, 0.8, 0 ; = 5 × 10–3 5 x ; D t = 80 (4  2 3)

 330 
8. 200   i.e. 221.0 Hz, 182.6 Hz.
 330 10 
9. (ACD)
 1447  5  339  5
10. (a) f ¢ =  1437  10 Hz (b) f ¢¢ =  329  10 Hz
   

340  12.56
11. fmax =  300  323Hz ,
340 – 12.56

 340  12.56 
fmin =   × 300 = 278.6 Hz
 340  12.56 
12. (B)
MA 400 fA 3
13. (a) M = (b) f =
B 189 B 4
14. (B)
15. (B)
16. 336 m/s
17. (D) 18. (C) 19. Vs = 30 m/s 20. (C)
21. (A) 22. (A) 23. (C) 24. (B) 25. (A) 26. (A)
27. (A) 28. (AD) 29. (7) 30. (B) 31. (A)
32. (A) – p,t , (B) – p,s, (C) – q,s, (D) – q, r
33. (AB) 34. (D) 35. (BD) 36. (B)

Page # 210
AVES ON STRING AND SOUND WAVES PHYSICS PART - IV

Wave On A String

1. (BCD) 2. (ABC)
– v1 2v 2
3. (a) Time = 140 ms (b) Ar = vv 2  v Ai = 1.5 cm ; At = v  v Ai = 2 cm
2 1 1 2

5 m
4. (i) , where S = area of cross section of wire (ii) 8
3 70 S
5. (D)
1 10 10  1
6. [(m0 + a l)3/2 - (m0)3/2] = s
15  135
7. (D) 8. (A) 9. (ABC) 10. (C) 11. (B) 12. (A)

2 a 2 T
13.
4L

 3 
14. Equation of wave in string y = 0.1 sin  30 t  2 x    [where f is initial phase]
 
15. (A) 16. (A) 17. (5) 18. (5) 19. (BC) 20. (D)
21. A, C, D 22. (3) 23. (ACD) 24. (6)

Page # 211

Potrebbero piacerti anche